SlideShare uma empresa Scribd logo
1 de 252
Question 1: They offered her the job because she was
very _______ on the design front.
A. creatively B. creative C. create D. Creation
1 (TH)
Kiến thức: Từ loại
Giải thích:
A. creatively (adv): một cách sáng tạo
B. creative (adj): sáng tạo
C. create (v): tạo ra
D. creation (n): sự sáng tạo
Sau động từ “was” và trạng từ “very” cần điền tính từ.
Dấu hiệu: đuôi –tion thường là danh từ, -ly thường là trạng từ, -ive
thường là tính từ.
Tạm dịch: Họ đề nghị cô ấy làm việc vì cô ấy rất sáng tạo trong lĩnh
vực thiết kế.
Question 2: She’ll take the flight for the early meeting,
______?
A. won’t she B. hasn’t she C. didn’t she D. doesn’t she
(NB) Kiến thức: Câu hỏi đuôi
Giải thích:
Vế trước câu hỏi đuôi dạng khẳng định => câu hỏi đuôi
dạng phủ định
Vế trước: She'll take => câu hỏi đuôi: won’t she?
Tạm dịch: Cô ấy sẽ đáp chuyến bay cho cuộc họp sớm
phải khôn
Question 3: The letters he would write were full of doom
and _______. We still do not know what made him so
depressed.
A. drib B. gloom C. duck D. Tuck
3 (VDC)
Kiến thức: Thành ngữ
Giải thích:
A. drib (không tồn tại từ này) B. gloom (n): u ám, buồn
bã
C. duck (n): con vịt D. tuck (n): nếp gấp (ở quần áo)
=> doom and bloom: cảm giác tuyệt vọng, bi quan
Tạm dịch: Những lá thư anh ấy viết chứa đầy cảm giác
tuyệt vọng. Chúng tôi vẫn không biết điều gì đã khiến anh
ấy chán nản như vậy.
Question 4: The restaurant is well-known ________
its friendly atmosphere and excellent service.
A. for B. on C. in D. Off
4(TH)
Kiến thức: Giới từ
Giải thích:
A. for: cho B. on: trên C. in: trong D. off: rời, nghỉ
=> be well-known for something: nổi tiếng về cái gì
Tạm dịch: Nhà hàng nổi tiếng với bầu không khí thân
thiện và dịch vụ tuyệt vời.
Question 5: The course was so difficult that I didn’t
………any progress at all.
A. do B. make C. produce D. create
5 (TH)
Make progress ~ improve: tiến bộ.
Dịch: Khóa học này quá khó đến nỗi tôi không tiến bộ
lên được chút nào
Question 6: John crashed his truck _________ he
was driving too fast.
A. despite B. although C. because D. because of
6(TH)
Kiến thức: Liên từ
Giải thích:
A. despite + cụm danh từ/ V_ing: mặc dù … B.
although + S + V: mặc dù …
C. because + S + V: bởi vì … D. because of + cụm danh
từ/ V_ing: bởi vì …
Sau chỗ trống là một mệnh đề => loại A, D.
Tạm dịch: John đã gặp va chạm với xe tải của mình vì
anh ta lái xe quá nhanh.
Question 7: She bought a _________ jacket as a
present for her daughter.
A. new Italian leather B. new leather Italian
C. leather new Italian D. leather Italian new
7 (TH)
Kiến thức: Trật tự tính từ
Giải thích:
Khi có nhiều tính từ cùng đứng trước 1 danh từ, sắp xếp chúng theo thứ tự:
OSASCOMP + N. Trong đó:
O – opinion:quan điểm S – size:kích thước A – age: độ tuổi
S – shape: hình dạng C – colour:màu sắc O – origin: nguồngốc
M – material: chất liệu P – purpose:mục đích N – noun: danh từ
Nếu có số thứ tự => đứng trước tính từ & danh từ
=> Trật tự tính từ: “new” – chỉ độ tuổi + “Italian”– chỉ nguồn gốc + “leather”–
chỉ chất liệu
Tạm dịch: Cô ấy đã mua một chiếc áo khoác da mới của Ý để làm quà cho con gái.
Question 8: Unless we do something now, hundreds
of plant and animal species will _______.
A. point outB. die out C. cut up D. make up
8 (TH)Kiến thức: Cụm động từ
Giải thích:
A. point out: chỉ ra B. die out: chết đi
C. cut up: cắt nhỏ D. make up: chiếm (phần trăm), bịa
(chuyện), …
Tạm dịch: Nếu chúng ta không làm điều gì đó ngay bây
giờ, hàng trăm loài động thực vật sẽ chết.
Question 9: While she was taking her exams, she
_______ a terrible headache.
A. will haveB. has C. had D. is having
9(TH)
Kiến thức: Thì quá khứ đơn, phối hợp thì
Giải thích:
Cấu trúc: While + S + was/ were + V_ing, S + V_ed/ cột 2
(quá khứ đơn)
Cách dùng: diễn tả một hành động đang xảy ra trong quá
khứ (chia quá khứ tiếp diễn) thì một hành động khác xen
vào (chia quá khứ đơn).
Tạm dịch: Trong khi cô ấy đang làm bài thi thì cô ấy bị
đau đầu kinh khủng.
Chọn C.
Question 10: I hope these machines will have worked very well
______.
A. as soon as you would come back next month
B. when you came back next month
C. by time you come back next month
D. after you were coming back next month
10 (TH)
Kiến thức: Sự kết hợp thì
Giải thích:
Dấu hiệu: “next month” => chia các thì tương lai
Cấu trúc: S + will have P2 + by the time + S + V_(s/es): … vào
thời điểm …
Động từ trong mệnh đề thời gian chia thì hiện tại, động từ trong
mệnh đề chính chia tương lai. => loại A, B, D.
Tạm dịch: Tôi hy vọng những chiếc máy này sẽ hoạt động rất
tốt vào thời điểm bạn quay lại vào tháng sau.
Question 11: The popular press often contains a lot
more _______ than hard facts.
A. tolerance B. influence C. speculation D. Realism
11 (VD)
Kiến thức: Từ vựng
Giải thích:
A. tolerance (n): khoan dung B. influence (n): ảnh
hưởng
C. speculation (n): sự đồn đoán, đầu cơ D. realism
(n): chủ nghĩa hiện thực
Tạm dịch: Báo chí phổ thông thường chứa đựng
nhiều thông tin đồn đoán hơn là những sự thật trần
trụi.
Question 12: Researchers have ________ to the
conclusion that personality is affected by your genes.
A. gotB. reached C. arrived D. Come
12 (VD)
Kiến thức: Từ vựng, sự kết hợp từ
Giải thích:
A. got (P2): nhận được
B. . reached (P2): chạm tới, đạt tới
C. arrived (P2): đến
D. come (P2): tới, về
=> come to the conclusion: đưa ra kết luận
Tạm dịch: Các nhà nghiên cứu đã đưa ra kết luận
rằng tính cách bị ảnh hưởng bởi gen của bạn.
Question 13: The flood victims ________ with food and clean water
by the volunteers
A. provided B. were provided C. were providing D. provide
13 (TH)
Kiến thức: Bị động thì quá khứ đơn
Giải thích:
Chủ ngữ là “The flood victims” – “Những nạn nhân lũ lụt”,động
từ “provide” – “cung cấp” => câu mang nghĩa bị động
Cấu trúc câu bị động thì quá khứ đơn: S + was/ were + Vp2 (by +
O).
Tạm dịch: Những nạn nhân lũ lụt được cung cấp đồ ăn và nước
sạch bởi những tình nguyện viên.
Chọn B
Question 14: _______ the promotion, I began to search for
other goals.
A. Having attained B. To attain C. Being attained D.
Attained
14 (VDC)
Kiến thức: Rút gọn mệnh đề trạng ngữ
Giải thích:
Khi mệnh đề trạng ngữ và mệnh đề chính có cùng một chủ ngữ, có thể lược
bỏ chủ ngữ ở mệnh đề trạng ngữ và đưa động từ về dạng V_ing khi mệnh
đề ở dạng chủ động
Để nhấn mạnh hành động ở mệnh đề trạng ngữ xảy ra trướchành động ở
mệnh đề chính ta dùng dạng: having+ P2.
Câu đầy đủ: I had attainedthe promotion, I began to searchfor other goals.
Câu rút gọn: Having attainedthe promotion,I beganto searchfor other goals.
Tạm dịch: Sau khiđạt được sự thăng tiến, tôi bắt đầu tìm kiếm các mục tiêu
khác.
Question 15: _______ the person is, the more privilege he enjoys.
A. The rich B. The richest C. As rich as D. The richer
15(TH)
Kiến thức: So sánh lũy tiến/ so sánh kép
Giải thích:
Cấu trúc so sánh lũy tiến: The + so sánh hơn + S + V, the
+ so sánh hơn + S + V
Dạng so sánh hơn của tính từ “rich” là “richer”.
Tạm dịch: Người càng giàu thì càng được hưởng nhiều
đặc quyền.
Question 16: Peter is talking to Mary about eating habits.
- Peter: “You should eat more fruits and vegetables.” – Mary:
“___________”
A. Yes, I will, Thanks B. No, thanks
C. My pleasure D. You’re welcome
16 (TH)
Kiến thức: Ngôn ngữ giao tiếp
Giải thích:
Peter đang nói chuyện với Mary về thói quen ăn uống.
Peter: “Bạn nên ăn nhiều trái cây và rau quả hơn.”
Mary: “ ________ ”
A. Ừ, tớ sẽ làm vậy. Cảm ơn nhé.
B. Không, cảm ơn.
C. Rất hân hạnh.
D. Không có chi.
Question 17: Mrs Brown and Mr Smith are talking about
teaching soft skills at school.
- Mrs Brown: “Some soft skills should be taught to children.”
- Mr Smith: “______. They are necessary for them.”
A. I don’t either B. I agree with you
C. You’re quite wrong D. You’re welcome
17. (TH)
Kiến thức: Ngôn ngữ giao tiếp
Giải thích:
Bà Brown và ông Smith đang nói về việc dạy các kỹ năng
mềm ở trường.
Bà Brown: “Một số kỹ năng mềm nên được dạy cho trẻ em.”
Ông Smith: “ ________ . Chúng cần thiết cho họ.”
A. Tôi cũng không B. Tôi đồng ý với bạn
C. Bạn khá sai rồi D. Không có chi
Question 18
A. document B. holiday C. location D.
journalist
18 (NB)
Kiến thức: Trọng âm từ có 3 âm tiết
Giải thích:
A. document /ˈdɒkjumənt/ B. holiday /ˈhɒlədeɪ/
C. location /ləʊˈkeɪʃn/ D. journalist /ˈdʒɜːnəlɪst/
Question 19
A. listen B. agree C. escape D. deny
19 (NB)
Kiến thức: Trọng âm từ có 2 âm tiết
Giải thích:
A. listen /ˈlɪsn/ B. agree /əˈɡriː/
C. escape /ɪˈskeɪp/ D. deny /dɪˈnaɪ/
Phương án A có trọng âm rơi vào âm tiết 1, còn lại là
âm tiết 2.
Chọn A.
Question 20:
A. concerned B. devoted C. renewed D. improved
20 (NB)
Kiến thức: Cách phát âm đuôi “ed”
Giải thích:
A. concerned /kənˈsɜːnd/ B. devoted /dɪˈvəʊtɪd/
C. renewed /rɪˈnjuːd/ D. improved /ɪmˈpruːvd/
Quy tắc phát âm động từ đuôi –ed dựa vào chữ cái kết thúc:
Những động từ có chữ cái kết thúc tận cùng là:
TH 1: t, d => đuôi –ed được phát âm là /id/
TH 2: p, pe; k, ke; ff, ph, gh; ss, ce, se, x; ch; sh => đuôi –ed được
phát âm là /t/
TH 3: còn lại => đuôi –ed được phát âm là /d/
Phần gạch chân phương án B được phát âm là /ɪd/, còn lại phát
âm là /d/.
Question 21:
A. thrill B. hide C. prize D. crime
Kiến thức: Cách phát âm “i”
Giải thích:
A. thrill /θrɪl/ B. hide /haɪd/ C. prize /praɪz/ D.
crime /kraɪm/
Phần gạch chân phương án A được phát âm là /ɪ/, còn
lại phát âm là /aɪ/.
Chọn A.
Question 22: Please give me some recommendation
to buy suitable books for my ten-year-old girl
A. advice B. defense C. interest D. question
22 (TH)
Kiến thức: Từ vựng
Giải thích:
recommendation (n): sự giới thiệu, tiến cử
A. advice (n): lời khuyênB. defense (n): sự che chở,
phòng thủ
C. interest (n): mối quan tâm D. question (n): câu hỏi
=> recommendation = advice
Tạm dịch: Xin hãy cho tôi một vài giới thiệu để mua
những cuốn sách phù hợp cho con gái mười tuổi của
tôi.
Question 23: He has been jobless and often has to ask his
parents for money
A. out of fashion B. out of practice C. out of order
D. out of work
23 (TH)
Kiến thức: Từ đồng nghĩa
Giải thích:
jobless (adj): không có việc làm, thất nghiệp
A. out of fashion: lỗi thời B. out of practice: không
rèn luyện
C. out of order: bị hỏng (máy móc) D. out of work:
thất nghiệp
=> jobless = out of work
Tạm dịch: Anh ấy thất nghiệp và thường xuyên phải
xin tiền bố mẹ.
Question 24: Aren’t you putting the cart before the horse by
deciding what to wear for the wedding before you’re invited to it?
A. knowing the horse cart
B. do things in the right order
C. doing things in the wrong order
D. upsetting the horse cart
Kiến thức: Thành ngữ
Giải thích:
putting the cart before the horse: làm việc sai trình tự
A. knowing the horse cart: biết đến xe ngựa
B. do things in the right order: làm mọi việc theo đúng trình tự
C. doing things in the wrong order: làm những việc không đúng
trình tự
D. upsetting the horse cart: làm đổ xe ngựa
=> putting the cart before the horse >< do things in the right
order
Tạm dịch: Bạn có đang làm sai trình tự khi quyết định mặc gì
cho đám cưới trước khi bạn được mời đến dự không đó?
Question 25: He is writing a letter of acceptance to the
employer with the hope to get his favorite position in the
company.
A. confirmation B. refusal C. agree D. admission
25 (TH)
Kiến thức: Từ vựng
Giải thích:
acceptance (n): sự chấp nhận
A. confirmation (n): sự xác nhận B. refusal (n): sự từ chối
C. agree (v): đồng ý D. admission (n): sự thu nạp, kết nạp
=> acceptance >< refusal
Tạm dịch: Anh ấy đang viết một lá thư chấp nhận cho nhà tuyển
dụng với hy vọng sẽ có được vị trí yêu thích trong công ty.
Question 26: The book was interesting. I’ve read it three times.
A. Such was the interesting book that I have read it three times.
B. So interesting was the book that I have no time to read it.
C. Only if it is an interesting book have I read it three times.
D. Should the book be interesting, I have read it three times.
26 (VDC)
Kiến thức: Đảo ngữ
Giải thích:
Đảo ngữ với “such … that …”: Such + be + cụm danh từ + that + S + V
Đảo ngữ với “so … that …”: So + adj + be + S1 + that + S2 + V2
Đảo ngữ với “only if”: Only if + S1 + V1 + trợ động từ + S2 + V2
Đảo ngữ câu điều kiện loại 1: Should + S + V_infinitive, S + will/ can/ may
+ V_infinitive
Tạm dịch: Cuốn sách thật thú vị. Tôi đã đọc nó ba lần.
A. Đó là cuốn sách thú vị đến mức tôi đã đọc nó ba lần.
B. Cuốn sách thú vị đến mức tôi không có thời gian để đọc nó. => sai
nghĩa
C. Chỉ khi nó là một cuốn sách thú vị, tôi mới đọc nó ba lần. => sai nghĩa
D. sai ngữ pháp, sai nghĩa
Question 27: She bought an old TV. She has regrets about it
now.
A. Provided she bought an old TV. She wouldn’t have regrets.
B. If she hadn’t bought an old TV. She would have regrets.
C. If only she had bought an old TV
D. She wishes she hadn’t bought an old TV.
Kiến thức: Câu ước
Giải thích:
Provided that + S + V_(s/es), S + will/ can/ may + V_infinitive: Miễn là
Câu điều kiện hỗn hợp 3 và 2: If + S + had + P2, S + would/ could/ might +
V_infinitive
Diễn tả 1 điều kiện trái với quá khứ, dẫn đến 1 kết quả trái với hiện tại
Cấu trúc: If only + S + had + P2: Giá mà ai đó đã làm gì trong quá khứ
(nhưng đã không làm)
Câu ước trái với quá khứ: S + wish(es) + S + had + P2
Tạm dịch: Cô ấy đã mua một chiếc TV cũ. Cô ấy hối hận về điều đó bây giờ.
A. sai ngữ pháp: “provided” dùng cho câu điều kiện loại 1
B. Nếu cô ấy không mua một chiếc TV cũ, cô ấy sẽ hối tiếc. => sai nghĩa
C. Giá mà cô ấy đã mua một chiếc TV cũ. => sai nghĩa
D. Cô ấy ước cô ấy đã không mua một chiếc TV cũ.
Question 28: Last night, she stays up so late to study for her exams.
A B C D
28(NB)
Kiến thức: Thì quá khứ đơn
Giải thích:
Dấu hiệu: “Last night” – tối hôm qua => chia thì quá khứ
đơn
Thì quá khứ đơn: S + V_ed/ cột 2
Diễn tả một hành động đã xảy trong quá khứ có thời
điểm xác định
Sửa: stays => stayed
Tạm dịch: Đêm qua, cô ấy thức rất khuya để học bài
cho kỳ thi.
Question 29: She is a confidential
and practiced speaker who always
impresses her audience.
Kiến thức: Từ vựng
Giải thích:
confidential (adj): bí mật, thân tín confident (adj):
tự tin
Sửa: confidential => confident
Tạm dịch: Cô ấy là một diễn giả tự tin và có kinh
nghiệm, người luôn gây ấn tượng với khán giả của
mình.
Chọn A.
Question 30: My sister finally got his
own favorite piano to practice every
day.
Kiến thức: Tính từ sở hữu
Giải thích:
Chủ ngữ “My sister” – danh từ chỉ người là phái nữ
=> dùng tính từ sở hữu “her”
Sửa: his => her
Tạm dịch: Em gái tôi cuối cùng đã có được cây
đàn piano yêu thích của riêng mình để luyện tập
mỗi ngày.
Chọn B.
Question 31: Every student is required to write an essay on
the topic.
A. Every student might write an essay on the topic.
B. Every student must write an essay on the topic.
C. Every student can’t write an essay on the topic.
D. Every student needn’t write an essay on the topic.
Kiến thức: Động từ khuyết thiếu
Giải thích:
be required to: được yêu cầu phải làm gì
S + might + V_infinitive: Ai đó có thể làm gì
S + must + V_infinitive: Ai đó phải làm gì
S + can’t + V_infinitive: Ai đó không thể làm gì
S + needn’t + V_infinitive: Ai đó không cần làm gì
Tạm dịch: Mỗi học sinh được yêu cầu viết một bài luận về chủ đề này.
A. Mỗi học sinh có thể viết một bài luận về chủ đề này. => sai nghĩa
B. Mỗi học sinh phải viết một bài luận về chủ đề này.
C. Mọi học sinh không thể viết một bài luận về chủ đề này. => sai nghĩa
D. Mọi học sinh không cần phải viết một bài luận về chủ đề này. => sai
nghĩa
Question 32: “If I were you, I would rent anotherroom” saidmy friend.
A. My friend threatenedme to rent anotherroom.
B. My friend was thinking about renting another room.
C. My friend insistedon renting anotherroom.
D. My friend advisedme to rent anotherroom.
Kiến thức: Câu tường thuật
Giải thích:
S + threatened + O + to V_infinitive: Ai đó đe dọa ai làm gì
S + insisted on + V_ing: Ai đó nài nỉ làm gì
S + advised + O + to V_infinitive: Ai đó khuyên ai làm gì
Tạm dịch: “Nếu tôi là bạn, tôi sẽ thuê một phòng khác” bạn
tôi nói.
A. Bạn tôi đe dọa tôi phải thuê phòng khác. => sai nghĩa
B. Bạn tôi đã nghĩ đến việc thuê một căn phòng khác. => sai
nghĩa
C. Bạn tôi đòi thuê phòng khác. => sai nghĩa
D. Bạn tôi khuyên tôi nên thuê phòng khác
Question 33: He last cooked for the whole family five months
ago.
A. He didn’t cook for the whole family five months ago.
B. He has cooked for the whole family for five months.
C. He hasn’t cooked for the whole family for five months.
D. He would cook for the whole family in five months.
Kiến thức: Thì hiện tại hoàn thành
Giải thích:
Cấu trúc: S + last + V_ed + khoảng thời gian + ago
= S + have/ has + (not) + P2 + for + khoảng thời gian
Tạm dịch: Lần cuối cùng anh ấy nấu cho cả gia đình là cách đây 5 tháng.
A. Anh ấy đã không nấu ăn cho cả gia đình năm tháng trước. => sai nghĩa
B. Anh ấy đã nấu ăn cho cả gia đình trong năm tháng. => sai nghĩa
C. Anh ấy đã không nấu ăn cho cả gia đình trong năm tháng.
D. Anh ấy sẽ nấu ăn cho cả gia đình trong năm tháng. => sai nghĩa
Question 34:
Read the following passage and mark the letter A, B, C or D on
your answer sheet to indicate the correct word or phrase that
best fits each of the numbered blanks from 1 to 5.
A stinky gym bug in your kitchen? Who will be more upset
by the smell - the men or the women in your family? (34)
_______ scientists suggest that women not only smell, but feel,
taste, and hear more accurately than men.
A. Each B. Some C. Another D. Every
Kiến thức: Lượng từ
Giải thích:
A. Each + danh từ đếm được dạng số ít: Mỗi …
B. Some + danh từ đếm được dạng số nhiều: Một vài …
C. Another + danh từ đếm được dạng số ít/ of + danh từ đếm
được dạng số nhiều: … khác
D. Every + danh từ đếm được dạng số ít: Mỗi …
Sau chỗ trống là danh từ số nhiều, đếm được => loại A, C, D.
Question 35:
Take colors, for example. One study suggests that men are not
as good as women at distinguishing between (35) _______ of
color, although they focus well on rapidly changing images.
A. smells B. senses C. shadows D. shades
Kiến thức: Từ vựng, sự kết hợp từ
Giải thích:
A. smells (n): mùi B. senses (n): giác quan
C. shadows (n): bóng tối D. shades (n): sắc thái
=> shades of color: sắc thái màu sắc
Tạm dịch: Một nghiên cứu cho thấy rằng đàn ông không giỏi
bằng phụ nữ trong việc phân biệt giữa các sắc thái màu sắc,
mặc dù họ tập trung tốt vào những hình ảnh thay đổi nhanh
chóng.
Question 36: Evidence shows that boys hear as well as girls
at birth, but with age, a man's hearing may soon deteriorate.
………………, environmental factors could play a role in this.
A. Moreover B. ThereforeC. However D. Due to
36 (TH)
Kiến thức: Liên từ
Giải thích:
A. Moreover + S + V, …: Hơn thế nữa … (thêm ý)
B. Therefore, …: do đó … (chỉ kết quả)
C. However, …: tuy nhiên … (chỉ ý đối lập)
D. Due to + cụm danh từ/ V_ing, …: vì …(chỉ nguyên nhân)
Tạm dịch: Bằng chứng cho thấy rằng các bé trai nghe tốt như các
bé gaí khi mới sinh, nhưng theo tuổi tác, thính lực của đàn ông có
thể sớm bị suy giảm. Tuy nhiên, các yếu tố môi trường có thể
đóng vai trò trong việc này.
Question 37
Women may also be better at identifying different flavors ……………..
need both taste and smell to experience, as studies have shown
that they have more taste buds on their tongue.
A. who B. which C. where D. whom
Kiến thức: Đại từ quan hệ
Giải thích:
A. who + V: người mà … => bổ sung thông tin cho từ chỉ người trước nó
B. which + V: cái mà … => bổ sung thông tin cho từ chỉ vật trước nó
C. where + S + V: khi mà … => bổ sung thông tin cho từ chỉ địa điểm
trước đó
D. whom + S + V: người mà … => bổ sung thông tin cho từ chỉ người
trước nó
“different flavors” – danh từ chỉ vật => loại A, C, D.
Tạm dịch: Phụ nữ cũng có thể giỏi hơn trong việc xác định các hương vị
khác nhau những thứ cần cả vị giác và khứu giác để trải nghiệm, vì các
nghiên cứu đã chỉ ra rằng họ có nhiều nụ vị giác hơn trên lưỡi.
Question 38:
A. taken B. sprung C. mazed D. freaked
Going back to the smelly gym bag - yes, it's likely that Mom will
be the most ……………..out by it.
Kiến thức: Từ vựng
Giải thích:
A. taken (P2): lấy B. sprung (P2): nhảy ra, bật lên
C. mazed (không tồn tại từ này) D. freaked (P2): sốc, sợ hãi,
ngạc nhiên
=> freaked out: sốc, sợ hãi, ngạc nhiên
Tạm dịch: Quay trở lại với chiếc túi tập thể dục bốc mùi - vâng,
có khả năng mẹ sẽ là người sốc nhất với nó.
Question39:Which of the followingbest states the main idea of the
reading?
A. No one knows for sure where dogs firstappearedwith humans.
B. There are two theories about the evolutionof dogs.
C. Evolutionand breeding have made dogs what they are today.
D. Human beings used dogs for many jobs, so dogs are the first tame
animals.
Kiến thức: Đọc hiểu
Giải thích:
Câu nào sau đây thể hiện đúng nhất ý chính của bài đọc?
A. Không ai biết chắc nơi đầu tiên loài chó xuất hiện cùng với con người.
B. Có hai giả thuyết về sự tiến hóa của loài chó.
C. Sự tiến hóa và lai tạo đã tạo nên những con chó
D. Loài người đã sử dụng chó vào nhiều công việc, vì vậy chó là con vật được
thuần hóa đầu tiên.
Thông tin:
- Dogs originally came from wolves.
- At some point, some wolves changed through evolution. Some of them were
smaller and gentler. These nicer wolves were the first dogs.
- Some scientists believe that dogs mixed with other animals from the Canidae
family. This includes coyotes, foxes and jackals. Most scientists also believe that
evolution and breeding influenced the variation.
Question 40: The word “they” in the first paragraph
refers to _______.
A. people B. years C. dogs D. wolves
Kiến thức: Đọc hiểu
Giải thích:
Từ “they” trong đoạn đầu tiên đề cập đến ________ .
A. people (n): người B. years (n): năm
C. dogs (n): những con chó D. wolves (n): những con
chó sói
Thông tin: Wolves look a lot like dogs, but they are bigger.
Tạm dịch: Chó sói trông rất giống chó, nhưng chó sói lớn
hơn.
Question 41: According to some scientists, when did the first tame
dogs appear with humans?
A. About 15,000years ago B. Between 15,000and 50,000years ago
C. Over 50.000years ago D. About 13,000years ago
Kiến thức: Đọc hiểu
Giải thích:
Theo một số nhà khoa học, những con chó thuần hóa đầu tiên
xuất hiện cùng với con người khi nào?
A. Khoảng 15.000 năm trước B. Từ 15.000 đến 50.000
năm trước
C. Hơn 50.000 năm trước D. Khoảng 13.000 năm trước
Thông tin: Scientists who study humans say the first tame
dogs appeared with humans about 13,000 years ago in the
Middle East.
Tạm dịch: Các nhà khoa học nghiên cứu con người cho biết
những con chó đầu tiên được thuần hóa đã xuất hiện cùng với
loài người vào khoảng 13.000 năm trước ở Trung Đông.
Question 42: The word “companions” in the second paragraphmostly means
______.
A. messengers B. co-workers C. travelingfriends D. hunting
animals
Kiến thức: Đọc hiểu
Giải thích:
Từ “companions” trong đoạn thứ hai chủ yếu có nghĩa là
________ .
companions: bạn đồng hành
A. messengers (n): sứ giả, người đưa tin B. co-workers
(n): đồng nghiệp
C. traveling friends (n): bạn cùng đi du lịch D. hunting
animals (n): săn bắn động vật
=> companions = traveling friends
Thông tin: They were also good companions.
Tạm dịch: Chúng cũng là những người bạn đồng hành tốt.
Question 43: According to the reading, which of the following is TRUE?
A. Most people like to name their dogs Reliable, Blacky, and Useless.
B. The Roman always used dogs as messengers.
C. The first tame dogs appeared in ancient Egypt.
D. According to some studies, dogs mixed with coyotes and jackals.
Kiến thức: Đọc hiểu
Giải thích:
Theo bài đọc, điều nào sau đây là ĐÚNG?
A. Hầu hết mọi người đều thích đặt tên cho những chú chó của họ là
Reliable, Blacky và Useless.
B. Người La Mã luôn dùng chó làm người đưa tin.
C. Những chú chó được thuần hóa đầu tiên xuất hiện ở Ai Cập cổ
đại.
D. Theo một số nghiên cứu, chó lai tạp với chó sói đồng cỏ và chó
rừng.
Thông tin: Some scientists believe that dogs mixed with other
animals from the Canidae family. This includes coyotes, foxes and
jackals.
Tạm dịch: Một số nhà khoa học tin rằng những loài chó đã phối ngẫu
với các động vật khác từ họ Canidae. Bao gồm sói đồng cỏ, cáo và
Question 44: Which of the following could be served as the best title
for the passage?
A. Urban cities - The new opportunity for communitydevelopment
B. Urbanization - Pros and cons
C. Urbanization - How people’s health is impacted?
D. Developing countries - The fastest urbanization
Kiến thức: Đọc hiểu
Giải thích:
Điều nào sau đây có thể được coi là tiêu đề hay nhất cho đoạn văn?
A. Thành phố đô thị - Cơ hội mới để phát triển cộng đồng
B. Đô thị hóa - Ưu và nhược điểm
C. Đô thị hóa - Sức khỏe của con người bị ảnh hưởng như thế nào?
D. Các nước đang phát triển - Đô thị hóa nhanh nhất
Thông tin: Despite its positive things, there are also negatives from
urbanization on the physical health of humans living.
Tạm dịch: Mặc dù có nhiều điều tích cực của việc này, nhưng cũng có
những tiêu cực từ việc đô thị hóa đối với sức khỏe thể chất của con
người.
Question 45: The word “agrarian” in paragraph 2 mostly
means ________.
A. farming B. industry C. city D. modernizing
Kiến thức: Đọc hiểu
Giải thích:
Từ “agrarian” trong đoạn 2 chủ yếu có nghĩa là ________ .
agrarian (n): nông nghiệp
A. farming (n): nông nghiệp B. industry (n): công nghiệp
C. city (n): thành phố D. modernizing (n): hiện đại hóa
=> agrarian = farming
Thông tin: China is a country that in the past 30-40 years
went from being an agrarian based society to a significant
industrialized country.
Tạm dịch: Trung Quốc là một quốc gia mà trong vòng 30 - 40
năm qua đã từ một xã hội nông nghiệp trở thành một nước
công nghiệp với sự phát triển đáng kể.
Question 46: The word “congested” in paragraph 3 is closest in
meaning to _______.
A. fresh B. overcrowded C. sparse D. contaminated
Kiến thức: Đọc hiểu
Giải thích:
Từ “congested” trong đoạn 3 gần nghĩa nhất với ________ .
congested (adj): đông đúc, mật độ giao thông cao
A. fresh (adj): tươi B. overcrowded (adj): đông đúc
C. sparse (adj): thưa thớt D. contaminated (adj): bị ô nhiễm
=> congested = overcrowded
Thông tin: One very common and fairly obvious negative
aspect of highly congested urban areas is air pollution.
Tạm dịch: Một khía cạnh tiêu cực khác rất phổ biến và khá rõ
ràng của các khu đô thị vô cùng đông đúc là ô nhiễm không
khí.
Question 47: The following are the air pollution sources
mentioned in paragraph 3, EXCEPT _______.
A. industrial plants B. sewage
C. refineries waste D. chemicals
Kiến thức: Đọc hiểu
Giải thích:
Sau đây là các nguồn ô nhiễm không khí được đề cập trong đoạn
3, NGOẠI TRỪ ________ .
A. industrial plants: nhà máy công nghiệp B. sewage: nước thải
C. refineries waste: chất thải nhà máy lọc dầu D. chemicals:
hóa chất
Thông tin: It could include particulate matter, most commonly
attributed to industrial plants and refineries waste, or chemicals
like CO2 or Methane.
Tạm dịch: Nó có thể bao gồm các chất dạng hạt thường được
cho là do chất thải của các nhà máy công nghiệp và nhà máy lọc
dầu, hoặc các chất hóa học như CO2 hoặc Metan.
Question 48: The word “it” in the last paragraph refers to
________?
A. quality B. sugar C. food D. sodium
Kiến thức: Đọc hiểu
Giải thích:
Từ “it” trong đoạn cuối đề cập đến ________ .
A. quality (n): chất lượng B. sugar (n): đường
C. food (n): thực phẩm D. sodium (n): natri
Thông tin: Because this food is so accessible, people tend to
eat it more.
Tạm dịch: Bởi vì thực phẩm này rất dễ tiếp cận, mọi người
thường có xu hướng ăn thực phẩm này nhiều hơn.
Question49:Why are urban populations easy to get diseasesfrom food,
accordingto the last paragraph?
A. Becauseof the change in people’s diet.
B. Because this food is so deliciousthat people have a tendency to eat more
than normal.
C. Becausethe way people get this food is ratheraccessible, quick and easy.
D. Becauseof the low qualityand the high proportionof sodiumand sugar in
this food.
Kiến thức: Đọc hiểu
Giải thích:
Tại sao dân thành thị dễ mắc bệnh từ thức ăn, theo đoạn cuối?
A. Do chế độ ăn của con người thay đổi.
B. Vì thực phẩm này rất ngon nên mọi người có xu hướng ăn nhiều hơn
bình thường.
C. Vì cách mọi người có được thực phẩm này khá dễ tiếp cận, nhanh chóng
và dễ dàng.
D. Vì thực phẩm này có chất lượng thấp và tỷ lệ natri và đường cao.
Thông tin: This food is also more than likely not as high quality as well as
contains a large amount of sodium and sugar.
Tạm dịch: Các loại thực phẩm này nhiều khả năng không có chất lượng
cao cũng như chứa một lượng lớn natri và đường.
Question 50: What can be inferred fromthe passage?
A. One of the negativehealth effects comes from the overpopulationin
industrializedcountries.
B. Living in urban areas for a long time will certainlymake the life expectancy
of inhabitantsshorten.
C. Peoplein developedcountriessuffer less harmful health effects from
urbanizationthan thosein developingnations.
D. The bad health effects from urbanizationare not greater than the benefits it
brings to people in
Kiến thức: Đọc hiểu - Điều gì có thể được suy ra từ đoạn văn?
A. Một trong những ảnh hưởng tiêu cực đến sức khỏe là do dân số quá đông ở
các nước công nghiệp phát triển.
B. Sống ở thành thị trong thời gian dài chắc chắn sẽ làm cho tuổi thọ của cư dân
bị rút ngắn.
C. Người dân ở các nước phát triển ít bị ảnh hưởng sức khỏe do đô thị hóa hơn
người dân ở các nước đang phát triển.
D. Những ảnh hưởng xấu đến sức khoẻ do đô thị hoá không lớn hơn những lợi
ích mà nó mang lại cho người dân ở các thành phố đô thị.
Thông tin: As it would be expected, developing countries tend to see more
negative physical health effects than modern countries in regard to urbanization.
Tạm dịch: Đúng như dự đoán, các nước đang phát triển đều có xu hướng nhận
thức được việc xuất hiện nhiều ảnh hưởng tiêu cực đến sức khỏe thể chất con
người hơn là các nước hiện đại liên quan đến quá trình đô thị hóa.
Question 1. You may find doing this job very__________. Try it!
A. relaxing B. relaxed C. relax D. relaxation
Kiến thức: Từ loại
Giải thích:
A. relaxing (a): thoải mái, dễ chịu (tính từ đuôi ing, có yếu tố chủ
động, thể hiện bản chất, đặc điểm của sự vật, sự việc, tự thân nó có)
B. relaxed (a): thoải mái, dễ chịu (tính từ đuôi ed có yếu tố bị động,
nó không tự có tính chất này mà là bên ngoài tác động vào khiến nó
có cảm giác như vậy)
C. relax (v): thư giãn
D. relaxation (n): sự thư giãn, thoải mái
Ta có cấu trúc “Find sb/ sth ADJ’: thấy ai/ cái gì/ việc gì như thế nào.
Vì thế vị trí chỗ trống ta cần một tính từ. Và tính từ này để nói rõ tính
chất của “sb/sth” phía trước nên phải dùng tính từ đuôi ing..
Tạm dịch: Có lẽ bạn sẽ thấy công việc này rất thoải mái. Hãy thử nó
đi.
Question 2. No one on the plane was alive in the
accident last night, __________?
A. wasn’t he B. weren’t they C. were they D. was he
Kiến thức: Câu hỏi đuôi Giải thích:
Chủ ngữ là “No one” thì ở câu hỏi đuôi ta dùng đại từ
“they”.
Các đại từ bất định chỉ người như “everyone, everybody,
someone, somebody,anyone, anybody, no one, nobody” ta dùng
“they” cho vế câu hỏi đuôi.
Các đại từ bất định chỉ vật như “everything, something,
anything, nothing” ta dùng “it” cho vế câu hỏi đuôi.
Vế trước chủ ngữ “no one”: không một ai, chứa yếu tố phủ
định, nên câu hỏi đuôi sẽ ở thể khẳng định, do đó đáp án
phù hợp là “were they”. Chọn C.
Tạm dịch: Không ai trên máy bay còn sống trong vụ tai nạn
đêm qua, phải không?
Question 3. We had better keep on our__________
while were walking along the dark portions of this
street.
A. figures B. nails C. toes D. knees
Kiến thức: Idiom
Giải thích: Thành ngữ: keep on our toes: thận trọng,
đề phòng Chọn C
Tạm dịch: Chúng ta nên cảnh giác đề phòng khi
chúng ta đi bộ ở những đoạn đường tối ở thành
phố này.
Question 4. He regretted spending too much time _____
computer games.
A. to B. for C. in D. on
Kiến thức: Giới từ Giải thích:
Ta dùng cấu trúc: S + spend time/money on
sth: tiêu xài thời gian/ tiền bạc vào cái gì. Chọn D.
Ta cũng có: S + spend time/ money on sth: tiêu xài thời gian/ tiền
bạc vào cái gì.
Chọn D.
Ta cũng có:
S + spend time/ money Ving: tiêu xài thời gian/ tiền bạc vào
việc gì.
Tạm dịch: Anh ta thấy hối hận vì đã dùng quá nhiều thời
gian chơi điện tử.
Question 5. He looks for any excuse he can to blow off
his__________ to do housework.
A commitment B. obligation
C. assignment D. responsibility
Kiến thức: Từ vựng cùng trường nghĩa
Giải thích:
A. commitment (n): lời cam kết
B. obligation (n): nghĩa vụ
C. assignment (n): nhiệm vụ được giao
D. responsibility (n): trách nhiệm
Ta có cụm thường gặp “blow off the responsibility”: rũ bỏ
trách nhiệm. chọn D
Tạm dịch: Anh ta tìm mọi lý do có thể để rũ bỏ trách nhiệm
phải làm việc nhà
Question 6. __________ what she prepared for the job interview,
Megan didn’t pass it.
A. Despite of B. In spite of C. Though D. However
Kiến thức: Liên từ Giải thích:
Despite/ In spite of + N/ Ving = Though + S + V: mặc dù
However S+V: Tuy nhiên
Ta có: “What + s + V” là mệnh đề danh từ → Dùng “Despite/
In spite of”.
Loại A do despite không theo cùng với of, đồng thời loại C
và D vì sau Though là một mệnh đề và However không
đứng đầu để nối hai mệnh đề với nghĩa “tuy nhiên” như
này, However chỉ nối câu với câu hoặc đoạn với đoạn hoặc
đứng ở giữa được ngăn cách bởi dấu chấm phảy và dấu
phảy.
Tạm dịch: Bất chấp những gì cô ấy chuẩn bị cho buổi
phỏng vấn xin việc, Megan đã không vượt qua nó.
Question 7. There is __________ table in my bedroom.
A. an old square wooden B. a square wooden old
C. an old wooden square D. a wooden old square
Kiến thức: Trật tự tính từ
Giải thích:
Trật tự tính từ: OSASCOMP
1. Opinion and general description (Ý kiến hoặc miêu tả chung) Ví dụ: nice, awesome,
lovely...
2. Size / Weight(Kích cỡ, cân nặng) Ví dụ: big, small, heavy...
3. Age (Tuổi, cũ-mới) Ví dụ: old, new, young, ancient...
4. Shape (Hình dạng) Ví dụ: round, square, oval...
5. Color (Màu sắc) Ví dụ: green, red, blue,...
6. Origin (Xuất xứ) Ví dụ: Swiss, Italian, English....
7. Material (Chất liệu) Ví dụ: woolly, cotton, plastic...
8. Purpose (Mục đích) Ví dụ: walking (socks), running (shoes)....
Ta có sắp xếp đúng: old (cũ) - age → square (hình vuông) - shape → wooden(bằng
gỗ) – material
Chọn đáp án A
Tạm dịch: Có một chiếc bàn gỗ cũ hình vuông trong phòng của tôi.
Question 8. The new airport has__________ a lot of
changes on this island.
A. brought about B. taken to C. counted in D. turned up
Kiến thức: Phrasal verbs
Giải thích:
A. bring about: mang lại
B. take to: bắt đầu thích
C. count in: hoạt động cùng ai
D. turn up: xuất hiện
Tạm dịch: Sân bay mới đã mang lại nhiều thay đổi trên
hòn đảo này.
Question 9. They__________ the bridge by the time
you come back.
A. will finish B. will have finished
C. will be finished D. have finished
Kiến thức: sự phối hợp thì
Giải thích:
Thì tương lai hoàn thành dùng để diễn tả một hành động hoặc một sự
việc xảy ra và hoàn thành trước một hành động hoặc một thời điểm
khác trong tương lai.
Công thức: S + will + have + P2 by the time + S + V(s/es).
Chú ý: By the time là dấu hiệu để chia thì tương lai hoàn thành nếu sau
“by the time” có yếu tố hiện tại hoặc tương lai, còn by the time sẽ là
dấu hiệu chia quá khứ hoàn thành nếu sau cụm này có yếu tố của quá
khứ. Câu này có yếu tố tương lai nên chọn B.
Tạm dịch: Họ sẽ hoàn thành xong cây cầu trước khi anh quay lại.
Question 10. She won’t come home __________.
A. as soon as she had finished all the paperwork
B. until she has finished all the paperwork
C. by the time she finished all the paperwork
D. when she was finishing all the paperwork
Kiến thức: Mệnh để trạng ngữ
Giải thích:
Ta sử dụng kiến thức phổi hợp thì và kiến thức về liên từ
để giải quyết.
Mệnh đề cho trước đang là tương lai đơn nên không thể
dùng A (quá khứ hoàn thành); không dùng C (quá khứ
đơn), và không dùng D (quá khứ tiếp diễn) do tương lai
không kết hợp với quá khứ.
Question 11. The Youth Union in our school has decided to
launch a/an__________ to raise funds for local charities.
A. activity B. announcement C. campaign D. decision
Kiến thức: Từ vựng
A. activity: hoạt động
B. announcement:thông báo
C. campaign: chiến dịch, phong trào
D. decision: quyết định
Ta có: launch a campaign: phát động một phong trào
Tạm dịch: Hội thanh niên trường tôi đã phát động phong
trào để gây quỹ cho các tổ chức từ thiện địa phương.
Question 12. Being helpful is good, but don’t allow
others to__________ advantage of your generosity.
A. getB. take C. useD. make
Kiến thức: Cụm từ
Giải thích: Ta có: take advantage of: tận dụng/ lợi dụng =
make use of: sử dụng/ lợi dụng. Chọn B.
Tạm dịch: Có ích thì tốt, nhưng đừng để người khác lợi
dụng sự hào phóng của bạn.
Question 13. I demand to know how this vase __________,
and no one is leaving till I find out.
A. got broken B. was breaking C. has broken D. is broke
Kiến thức: Câu bị động.
Giải thích: Ngoài cấu trúc bị động quen thuộc là be + P2 thì ta
còn có cách diễn đạt bị động khác nữa với cấu trúc: Get + P2.
Loại B và C là 2 phương án viết ở chủ động, loại D do cách thể
hiện “is broke” sai ngữ pháp.
Question 14. Although __________ by the bravery of his fellow
soldiers, Bloch had harsh words for the army leadership.
A. was impressed B. impressed
C. having impressed D. impressing
Kiến thức: Giản lược mệnh đề cùng chủ ngữ:
Giải thích: Though/ Although + S + V, S + V: Mặc dù ...
Nếu chủ ngữ của 2 mệnh đề giống nhau thì ta có thể rút gọn
mệnh đề nhượng bộ theo cấu trúc:
Although+ ADJ/ P2, S + V.
Câu này cùng chủ ngữ là Bloch, có thể lược bỏ chủ ngữ ở
mệnh đề đầu tiên và giữ lại phần P2 (vì là câu bị động). Ta
chọn B.
Tạm dịch: Mặc dù bị ấn tượng bởi lòng dũng cảm của những
người lính, Bloch vẫn có những từ ngữ khá khắc nghiệt cho
việc lãnh đạo quân đội.
Tạm dịch: Tôi yêu cầu được biết làm sao mà cái lọ hoa này
lại bị vỡ, và sẽ không ai rời đi cho đến khi nào tôi tìm ra.
Question 15. The more you practice speaking in public, ____.
A. the more you become confident
B. the more you become confidently
C. the greater confidence you become
D. the more confident you become
Kiến thức: Câu so sánh đồng tiên: Càng... càng
Cấu trúc: The ...er/ more...S + V, the...er/ more...S + V.
Đáp án D đúng cấu trúc yêu cầu. Các phương án khác sai cấu trúc nên
loại.
Chọn D.
Tạm dịch: Bạn càng chịu khó luyện tập nhiều trước đám đông thì bạn
càng trở nên tự tin hơn.
Đáp án B sử dụng hiện tại hoàn thành kết hợp hoàn hảo với thì
tương lai như câu đã cho.
Tạm dịch: Cô ta sẽ không quay trở về nhà cho tới khi nào cô ấy hoàn
thành xong tất cả các công việc giấy tờ.
Question 16: Tom is inviting Linda to his birthday party.
Tom: “Would you like to come to my birthday party next week?"
Linda: “……………………..’’
A. Why not? B. Yes, I'd love to. C. I don’t think so. D. N 0, I‘d love to.
Tình huống giao tiếp
Tạm dịch: Tom đang mời Linda tới bữa tiệc sinh nhật của mình.
Tom: “cậu có muốn tham gia bữa tiệc sinh nhật vào tuần sau của tớ
không?" Linda: " ."
Tại sao không nhỉ? (đáp lại khi ai đó đưa ra ý kiến)
Có, tớ rất muốn đi (đáp lại lời mời)
Tớ không nghĩ vậy đâu
Không, tớ rất muốn
Question 17: Laura is asking Tom for his idea about a
vacation at the beach.
Laura: “Do you think a vacation at the beach will do me good?”
Tom: " ……………"
A. Sure. Have a good time there. B. Yes, I think.
C. Could you bye me something? D. Yes, it does.
Tình huống giao tiếp
Tạm dịch: Laura đang hỏi Tom về ý tưởng của anh ấy cho kì nghỉ
ở bãi biển.
Laura: "Cậu có nghĩa rằng đi nghỉ ở biển là tốt cho tớ không?"
Tom : “”
A. chắc chắn rồi. Chúc đi vui vẻ nha. B. có. Tớ nghĩ vậy
C. cậu có thể mua cho tớ vài thứ được không? D. có. Nó sẽ
như vậy mà.
Question 18.
A. equip B. secure C. vacant D. oblige
Đáp án C đúng vì đáp án C có trọng âm rơi vào âm thứ nhất. Các
phương án còn lại có trọng âm rơi vào âm tiết thứ hai.
equip (v) /ɪˈkwɪp/: trang bị
secure (adj) /sɪˈkjʊə/: an toàn
vacant (adj) /ˈveɪkənt/: còn trống
oblige (v) /əˈblaɪʤ/: bắt buộc
Question19.
A. encounter B. agency C. influence D. memory
Đáp án A đúng vì đáp án A có trọng âm rơi vào
âm thứ hai. Các phương án còn lại có trọng âm
rơi vào âm tiết thứ nhất.
encounter (v) /ɪnˈkaʊntə/: đương đầu
agency (n) /ˈeɪʤənsi/: đơn vị, đại lí
influence (v/n) /ˈɪnflʊəns/: ảnh hưởng
memory (n) /ˈmɛməri/: kí ức, trí nhớ
Question 20. A. asked B. danced C. cashed D. studied
Kiến thức: Cách phát âm đuôi “ed”
Giải thích:
- Đuôi “ed” được phát âm là /id/: Khi động từ có phát âm kết thúc là /t/ hay /d/
VD: wanted, needed, invited, decided
- Đuôi “ed” được phát âm là /t/: Khi động từ có phát âm kết thúc là: /s/: CE,
X, SS; /f/: GH, PH, /p/, /ʃ/:SH, /tʃ/: CH, /k/
VD: danced, fixed, crossed, laughed,
photographed, washed, watched, booked,...
- Đuôi “ed” được phát âm là /d/ với những trường hợp còn lại.
VD: considered, received, stayed,...
Đáp án D đúng vì phần gạch chân của đáp án D được đọc là /d/. Các
phương án còn lại phẩn gạch chân được đọc là /t/.
A. asked /ɑːskt/: hỏi
B. danced /dɑːnst/: nhảy, khiêu vũ
C. cashed /kæʃt/: đổi tiền mặt
D. studied /ˈstʌdɪd/: học
Question 21.
A. profile B. stomach C. postpone D. cyclone
Kiến thức: Cách phát âm nguyên âm “o”
Giải thích:
Đáp án B đúng vì phần gạch chân của đáp án B được đọc là
/ʌ/. Các phương án còn lại phần gạch chân được đọc là /əʊ/.
profile (n) /ˈprəʊfaɪl/: hồ sơ, sơ yếu lí lịch
stomach (n) /ˈstʌmək/: dạ dày, bụng
postpone (v) /pəʊstˈpəʊn/: trì hoãn
cyclone (n) /ˈsaɪkləʊn/: lốc. gió xoáy
Question 22: Such problems as haste and inexperience are a
universal feature of youth.
A. marked B. shared C. hidden D. separated
Kiến thức: Từ đồng nghĩa
Giải thích:
universal (adj): phổ biến, chung
shared: được chia sẻ, chung marked (adj): rõ rệt
hidden: ẩn, bị giấu đi separated (adj): ly thân
=> universal = shared
Tạm dịch: Những vấn đề như sự vội vàng và thiếu kinh nghiệm là
một đặc điểm chung của giới trẻ.
Question 23: If that was done on a national scale, we would
wipe out this infectious disease.
A. establish B. retain C. maintain D. eliminate
Question 24: Fee-paying schools, often called “independent
schools”, "private schools" or "public schools“
A. college B. primary schools
C. secondary schools D. state schools
Từ trái nghĩa
college /'kɔliddʒ/ (n): trường Đại học, Cao đẳng
primary school (n): trường Tiểu học
secondary school (n): trường Trung học phổ thông
state school (n): trường công lập
Tạm dịch: Những trường học phải trả phí thường được
gọi là trường dân lập hay là trường tư thục.
=> independent schools >< state schools
Question 25: Each time you turn it on, with
appropriate hardware and software, it is capable of
doing almost anything you ask.
A. unsuitable B. unimportant
C. ill-prepared D. irregular
Từ trái nghĩa
unsuitable /ʌn’su:təbl/ (a): không phù hợp, không thích hợp
unimportant /,ʌnim'pɔ:tənt/ (a): không quan trọng
ill-prepared (a): thiếu sự chuẩn bị
irregular /i'regjulə/ (a): không đều, bất quy tắc
Tạm dịch: Mỗi khi bạn bật nó lên, với phần cứng và phần mềm phù
hợp, nó có thể làm gần như bất cứ thứ gì mà bạn yêu cầu.
=> appropriate >< unsuitable
Question26. Hans told US about his investingin the company. He did it
on his arrivalat the meeting.
A. Only after investing in the company did Hans inform US of his
arrival at the meeting.
B. Not until Hans told US that he would invest in the company did he
arrive at the meeting.
C. Hardly had he informed US about his investing in the company
when Hans arrived at the meeting.
D. No sooner had Hans arrived at the meeting than he told US about
his investing in the company.
Kiến thức: Câu đảo ngữ
Câu đê' bài cho: Hans nói với chúng tôi về đầu tư của mình trong công ty. Ông đã
làm điều đó ngay khi đến cuộc họp.
A. Chỉ sau khi đầu tư vào công ty Hans mới bảo cho chúng tôi khi ông ấy đến cuộc
họp.
B. Mãi cho đến khi Hans nói với chúng tôi rằng ông sẽ đầu tư vào công ty ông mới
đến cuộc họp.
C. Ngay khi ông thông báo cho chúng tôi về việc đầu tư của mình trong công ty thì
Hans đến cuộc họp.
D. Ngay khi Hans đến tại cuộc họp thì ông ấy nói với chúng tôi về đầu tư của mình
trong công ty
Question 27. The driver in front stopped so suddenly.
Therefore, the accident happened.
A. If the driver in front didn’t stop so suddenly, the
accident wouldn’t happen.
B. If the driver in front hadn’t stopped so suddenly, the
accident would have happened.
C. If the driver in front hadn’t stopped so suddenly, the
accident wouldn’t have happened.
D. If the driver in front had stopped so suddenly, the
accident would have happened.
Kiến thức: Câu điều kiện
Câu đề bài cho: Người lái xe phía trước đã dừng đột ngột. Vì thế, vụ tai nạn đã xảy ra.
Đề bài cho một tình huống đã xảy ra trong quá khứ nên ta nhận định sẽ viết về câu
điều kiện loại 3 diễn tả những điều không có thật trong quá khứ với công thức: If S +
had P2, S + would have P2.
A loại do A viết về câu điều kiện loại 2 diễn tả những điều không có thật ở hiện tại.
B loại do sai nghĩa: Nếu người lái xe phía trước không dừng đột ngột thì vụ tai nạn sẽ
đã xảy ra.
C đúng: Nếu người lái xe phía trước không dừng đột ngột thì vụ tai nạn sẽ đã không
xảy ra.
D loại do sai nghĩa: Nếu người lái xe phía trước dừng đột ngột thì vụ tai nạn sẽ đã xảy
Question 28: What I told him a few
days ago is not the solution to most
of his problems.
Kiến thức ngữ pháp: ago → was
Chủ ngữ là mệnh đề danh từ động từ chia số it : were –
was
Question 29: Tom’s jokes are
inappropriate but we have to put up
with it just because he’s the boss.
Kiến thức: từ vựng
Giải thích:
“jokes” là danh từ ở dạng số nhiều nên phải dùng tân
ngữ “them” để thay th ế.
it => them
Tạm dịch: Những câu chuyện cười của Tom không phù
hợp nhưng chúng ta phải chịu đựng chúng chỉ vì ông ấy
là ông chủ.
Question 30: Modern office buildings have false floors
under which computer and phone wires can be lain.
Kiến thức về cặp từ dễ gây nhầm lẫn lay và lie
lay - laid - laid : đặt, xếp thứ gì đó nằm ở vị trí tĩnh
lie – lay - lain: tựa lên, nằm nghỉ trên một vị trí bằng phẳng
=> Đáp án là D(can be lain => can be laid)
Tạm dịch: Những tòa văn phòng hiện đại có những sàn nâng,
bên dưới chúng, dây điện máy vi tính và dâ
Question 31. It’s possible that she didn’t hear what I said.
A. She might not hear what I said.
B. She might have not heard what I said.
C. She may not hear what I said.
D. She may not have heard what I said.
Kiến thức: Modal Verb
Câu đề bài cho: Có khả năng là cô ta đã không nghe thấy những gì tôi nói.
Khi nói lại các tình huống đã xảy ra trong quá khứ với Modal Verb (động từ
khuyết thiếu) mà cụ thể ở đây là việc có lẽ cô ta đã không nghe thấy điều tôi
nói, chúng ta phải dùng cấu trúc: S + Modal Verb + HAVE + P2, duy nhất đáp
án D làm được điều này nên D đúng.
Câu A và C sai do hai câu này đang viết sử dụng công thức S + Modal Verb +
V0 nói về một phỏng đoán ở Hiện tại hoặc Tương lai.
Câu B sai do: Viết sai cấu trúc, phải là she might not have heard...
Question 32. “Stop smoking or you’ll be ill,” the doctor told me.
A. The doctor advised me to give up smoking to avoid illness.
B. The doctor suggested smoking to treat illness,
C. I was warned against smoking a lot of cigarettes.
D. I was ordered not to smoke to recover from illness.
Kiến thức: Câu chuyển Trực tiếp- Gián tiếp Câu đề bài cho:
“Ngừng hút thuốc hoặc anh sẽ bị bệnh”, bác sĩ đã nói với tôi.
A. Bác sĩ khuyên tôi nên bỏ hút thuốc để tránh bệnh tật.
B. Bác sĩ đề nghị hút thuốc để điểu trị bệnh.
C. Tôi đã được cảnh báo không hút nhiều thuốc lá.
D. Tôi được lệnh không hút thuốc để khỏi bệnh.
Question 33. I haven’t heard from Mike for
several months.
A. I didn’t hear from Mike several months ago.
B. Mike didn’t hear from me several months ago.
C. Mike heard from me several months ago.
D. I last heard from Mike several months ago.
Kiến thức: Viết lại cấu chuyển đổi Thì của động từ.
Câu đề bài cho: Tôi đã không nghe tin tức gì từ Mike vài
tháng nay rồi.
A. Tôi đã không nghe gì từ Mike vài tháng trước.
B. Mike đã không nghe gì từ tôi vài tháng trước.
C. Mike đã nghe tin tức từ tôi vài tháng trước.
D. Lần cuối cùng tôi nghe tin tức từ Mike là vài tháng trước
đây.
Question 34.
We also use a great …………….of water daily in our
homes, in factories, and in power stations. Most of
this water is fresh water and it comes to US from
reservoirs, rivers and lakes.
A. number B. much C. many D. amount
Kiến thức: Lượng từ
Giải thích: “water” là danh từ không đếm được, cho nên ta
dùng cụm “a great amount of”. Chọn D.
Các phương án còn lại:
A. number: số lượng
B. much: nhiều (dùng với danh từ không đếm được)
C. many: nhiều (dùng với danh từ đếm được số nhiều)
Tạm dịch: Chúng ta cũng sử dụng một lượng nước lớn hàng
ngày trong nhà, trong các nhà máy và trong các nhà máy
điện. Hầu hết nước này là nước ngọt và nó đến với chúng ta
từ các hồ chứa, sông hồ.
Question 35.
The Earth’s surface is ………. by large areas of water
which we call oceans and seas
A. covered B. reserved C. constructed D. included
Kiến thức: Từ vựng
Giải thích:
A. cover (v): bao phủ
B. reserve (v): dự trữ, để dành
C. construct(v): xây dựng
D. include (v): bao gồm
Ta có cách dùng: be covered: được/bị bao phủ
Tạm dịch: Bề mặt Trái đất được bao phủ bởi những vùng
nước rộng lớn mà chúng ta gọi là đại dương và biển.
Question 36.
If you have tasted the water from the sea, you will know that,
unlike fresh water, seawater tastes salty. This is due to the ……..
of sodium chloride which comes from the land.
A. attraction B. presence C. advantage D. realization
Kiến thức: Từ vựng Giải thích:
A. attraction (n): sự thu hút, sự lôi cuốn
B. presence (n): sự hiện diện
C. advantage (n): lợi thế, ưu điểm
D. realization (n): sự nhận biết
Tạm dịch: Nếu bạn đã nếm nước từ biển, bạn sẽ biết rằng,
không giống như nước ngọt, nước biển có vị mặn. Điều này là
do sự hiện diện của natri clorua đến từ đất.
Question 37.
The reason is …………. sunlight is made up of many colors.
A. what B. whoC. which D. that
Giải thích: Câu đã đầy đủ các thành phần: the reason (S) is
(V).., và phía sau là một mệnh đề nên ta dùng “that” đứng
trước mệnh đề đó. Đây là mệnh đề danh ngữ bắt đầu bằng
THAT.
Không có yếu tố thay thế cho danh từ đứng trước nên không
dùng các phương án còn lại.
Tạm dịch: Lý do là ánh nắng mặt trời được tạo thành từ nhiều
màu sắc.
Question 38.
Some colors disappear quickly in the sea but blue light bounces
back or is reflected, to the surface. This makes the sea look blue.
…………, a stormy sky will make the sea look grey.
A. Moreover B. Hence C. Although D. However
Kiến thức: Liên từ Giải thích:
A. Moreover: Hơn nữa
B. Hence: do đó, cũng vì lý do đó
C. Although: mặc dù
D. However: tuy nhiên
Ta chọn B theo ý nghĩa của bài.
Tạm dịch: Một số màu biến mất nhanh chóng trên biển nhưng
ánh sáng xanh bị dội ngược lại hoặc bị phản xạ lên bề mặt.
Điều này làm cho biển trông xanh. Cũng bởi lý do này, một bầu
trời giông bão sẽ làm cho biển trông xám xịt.
Question 39. Whatis the main idea of the passage?
A. Satoru Iwata is a man with amazing ideas.
B. Gaming is not just for dedicated gamers anymore.
C. Satoru Iwata is an important man for dedicated gamers
D. The gaming industry is making educational games now.
Câu hỏi về nội dung chính của bài:Ý chính của bài đọc này là gì?
A. Satoru Iwata là một người đàn ông có những ý tưởng đáng ngạc nhiên.
B. Việc chơi game không chỉ dành cho đối tượng chơi game chuyên nghiệp nữa.
C. Satoru Iwata là một người đàn ông quan trọng cho những người chơi game chuyên
nghiệp.
D. Nền công nghiệp game đang tạo ra nhiều trò chơi mang tính giáo dục.
Xuyên suốt bài đọc chúng ta thấy nói về những sáng kiến và ý tưởng mới của ông
Satoru Iwata làm thay đổi ngành công nghiệp game.
Đoạn 1: Khi ngành game đang chững lại và những sự cải tiến bị dập khuôn thì “Iwata
didn’t just want to attract dedicatedgamers. He wanted to bring in new kinds of playersto
video gaming.” - Iwata không muốn chỉ thu hút những người chơi giỏi. Ông ấy còn muốn
thu hút nhiều kiểu loại người chơi mới vào chơi game.
Đoạn 2: Iwata muốn trò chơi phù hợp với cuộc sống của mọi người hơn nữa, vì thế
ông ta đã tạo ra game mà: “People interacted more directly with the game by using a
touch screen instead of just a set of buttons” - Mọi người tương tác trực tiếp hơn với
game bằng màn hình cảm ứng thay vì chỉ với mỗi những nút bấm.
Đoạn 3: Những thay đổi đi cùng với những game mới do Iwatatạo ra: “Some of these
new games, like Brain Age, improvedthinking abilities.Others, like Wii Fit, improvedfitness.
This has brought in a new age of gaming. Everyone from grandparentsto their grandchildren
Question 40. What first motivated Iwata to change the
strategy of Nintendo?
A. He wanted to make useful consoles.
B. He wanted to lower sales.
C. He wanted to attract more players.
D. He wanted a new concept for gaming.
Điều gì đầu tiên đã thúc đẩy Iwata thay đổi chiến thuật của
Nintendo?
A. Ông ta muốn tạo ra những máy chơi game hữu ích hơn.
B. Ông ta muốn giảm doanh số bán hàng.
C. Ông ta muốn thu hút nhiều người chơi game nữa.
D. Ông ta muốn có một khái niệm chơi game mới.
Thông tin ở đoạn 1: However, Iwata didn’t just want to attract
dedicated gamers. He wanted to bring in new kinds of players to
video gaming.
Tuy nhiên, Iwata không chỉ muốn thu hút những người chơi
game chuyên nghiệp. Ông ta còn muốn thu hút nhiêu người
chơi game khác nữa.
Question 41. The word “that” in paragraph 2 refer to .
A. Bringing in new kinds of players to video gaming.
B. Attracting dedicated gamers
C. Making the same kinds of games
D. Improving games’ power and complexity.
Từ “that” trong đoạn 2 liên hệ với____.
A. Thu hút nhiều người chơi game mới.
B. Thu hút những người vốn đã thích chơi game.
C. Sáng tạo ra những loại game tương tự.
D. Cải thiện sức mạnh và độ phức tạp của game.
Thông tin đoạn 1: He wanted to bring in new kinds of
players to video gaming. How was he going to do that?
Ông ta còn muốn thu hút nhiều người chơi game khác
nữa. Ông ta sẽ định làm điều đó như thế nào?
Chọn A.
Question 42. Which is TRUE about changes Iwata make
about the world of gaming?
A. More games for children and girls.
B. More dedicated games.
C. Created a new style of gaming.
D. Less expensive game systems.
Điều gì là đúng về những thay đổi mà Iwata tạo ra về thế giới game?
A. Có thêm nhiều trò chơi cho trẻ em và phụ nữ.
B. Thu hút thêm nhiều người lúc nào cũng thích chơi game.
C. Tạo ra một kiểu chơi game mới.
D. Trò chơi ít đắt đỏ hơn.
Thông tin:
- Iwata wanted to make video games easier to pick up and more relevant to people’s
lives.
- The kinds of games produced changed as well.
- Ông ta mong muốn khiến cho các trò chơi điện tử dễ hiểu và dễ chơi hơn, liên
quan, phù hợp nhiều hơn đến cuộc sống của con người.
- Loại hình game cũng thay đổi.
Chọn C.
Question 43. The word “groundbreaking” in the last
paragraph is closest in meaning to
A. innovative B. bad C. beautiful D. natural
Từ “groundbreaking” ở đoạn cuối gần nghĩa với_____.
A. innovative (adj): sáng tạo, cải tiến, đổi mới
B. bad (adj): tệ hại
C. beautiful (adj): đẹp
D. natural (adj): tự nhiên
Thông tin: With all that we have seen so far, we can only wonder
what is next for Iwata. Surely it will be something groundbreaking.
Với tất cả những thứ mà chúng ta nhìn thấy, chúng ta chỉ có thể
hỏi là liệu Iwata sẽ làm gì tiếp theo. Chắc chắn đó sẽ là những
thứ mang tính đột phá.
Chọn A
Question44. What topicdoes the passagemainly discuss?
A. The way how to write the resume for job application.
B. The mistakes people make when applying for a job.
C. The commonway to make impression in a job interview.
D. The necessary skills for job application.
Bài văn chủ yếu thảo luận về chủ đề gì?
A. Cách viết bản sơ yếu lý lịch để xin việc.
B. Những lỗi mọi người thường gặp phải khi nộp đơn xin việc.
C. Cách thông thường để tạo ấn tượng tốt trong một cuộc phỏng
vấn xin việc.
D. Những kĩ năng cần thiết để xin việc.
Đoạn đầu tiên của bài đọc này đóng vai trò như phần mở bài giới
thiệu nội dung chính của cả bài, và các đoạn tiếp theo đi làm rõ ý
cho đoạn 1.
Căn cứ thông tin: “There are many mistakes that people make when
writing their resume (CV) or completing a job application. Here are some
of the most common and most serious.”
Có nhiều lỗi mà mọi người thường gặp khi viết bản sơ yếu lí lịch
hay hoàn thành hồ sơ xin việc. Dưới đây là một vài lỗi thường gặp
và nghiêm trọng nhất.
Question 45. The word “executing” in paragraph 2 is closest
in meaning to .
A. enumerating B. determining
C. completing D. implementing
Từ “executing” trong đoạn 2 gần nghĩa nhất với từ______.
A. liệt kê
B. xác định, tìm ra
C. hoàn thành
D. thực hiện, thi hành
Căn cứ thông tin: “They do not necessarily know the
specific skills you used in executing them, nor do they know
what results you achieved” - Họ không cần thiết phải biết
những kĩ năng cụ thể bạn đã sử dụng để thực hiện các
nhiệm vụ đó, họ cũng không cần biết kết quả bạn đã đạt
được là gì.
Từ đồng nghĩa: executing (thực hiện) = implementing. Chọn D.
Question 46. The word “concrete” in paragraph 3 could be
best replaced by .
A. indeterminate B. specific C. substantial D. important
Từ “concrete” trong đoạn 3 được thay thế tốt nhất bởi
từ______.
A. mơ hồ, không rõ
B. cụ thể, rõ ràng
C. chủ yếu, thiết yếu
D. quan trọng
Căn cứ thông tin: “The more concrete information you
can include, the better.” - Thông tin bạn cung cấp càng
cụ thể càng tốt
Từ đồng nghĩa: concrete (cụ thể) = specific.
Chọn B.
Question 47. What does the word “it” in paragraph 3 refer to ?
A. organization money B. information
C. productivity D. percentage
Từ “it” trong đoạn 3 đề cập đến từ nào?
A. tiền của tổ chức
B. thông tin
C. năng suất
D. tỉ lệ phần trăm
Căn cứ thông tin đoạn 3:
“If any innovations you introduced saved the organizationmoney, how much did
they save? If you found a way of increasing productivity, by what percentage did
you increase it?”
Nếu bạn đã đưa ra được ỷ tưởng đổi mới nào giúp tiết kiệm được tiền cho tổ
chức, thì họ đã tiết kiệm được bao nhiêu tiền? Nếu bạn đề xuất được một
cách tăng năng suất, vậy bạn đã làm tăng nó lên bao nhiêu phần trăm?
Vậy “it” ở đây thay thế cho “productivity” → Chọn C.
Question48. Accordingto the passage, what informationshould
candidatesincludein their resume?
A. specific skills for previous jobs B. the past achievements
C. previous positions D. future objective
Theo đoạn văn, thông tin nào những người xin việc nên bao hàm trong bản sơ
yếu lí lịch của họ?
A. các kĩ năng cụ thể cho các công việc trước đây
B. các thành tựu đạt được trong quá khứ
C. các chức vụ, vị trí công việc trước đây
D. mục tiêu trong tương lai
Từ khóa: information/include in their resume
Cản cứ các thông tin trong đoạn văn: “The biggest problem is perhaps listing the
duties for which you were responsible in a past position:all this tells your potential
employers is what you were supposed to do. They do not necessarily know the specific skills
you used in executing them, nor do they know what results you achieved” - Vấn đề lớn
nhất có lẽ là liệt kê các công việc mà bạn đã làm trong chức vụ trước đây: tất cả
những điều này nói cho nhà tuyển dụng tiềm năng của bạn biết những gì bạn có thể
làm được. Họ không cần thiết phải biết những kĩ năng cụ thể bạn đã sử dụng để thực
hiện các nhiệm vụ đó.→ Loại A
...họ cũng không cần biết kết quả bạn đã đạt được là gì. -→ Loại B
“Writing what you are trying to achieve in life - your objective - is a waste of space.” - Viết về
những điều bạn đang cố gắng đạt được trong cuộc sống - mục tiêu của bạn - là một
sự lãng phí giấy. → Loại D
Chọn C: chúng ta liệt kê các chức vụ trước đây đã làm nhưng không nên nhắc các kĩ
Question 49 According to the passage, which of the following is NOT true?
A. The abilityto negotiate effectively is as significant as technicalskills.
B. Candidatesmust study the job they are applyingcarefullybefore writing the CV
C. Applicantsshould not apply for a distinct job from what they are doing.
D. The information interviewees present should be related to the job they are applying.
Theo đoạn văn, câu nào sau đây là không đúng?
A. Khả năng đàm phán hiệu quả cũng quan trọng như các kĩ năng thực hành.
B. Người xin việc cần phải nghiên cứu công việc họ đang nộp đơn xin một cách kĩ
càng trước khi viết bản sơ yếu lí lịch.
C. Người xin việc không nên nộp đơn xin một công việc khác với công việc mà họ
đang làm.
D. Những thông tin mà người đi phỏng vấn trình bày nên có liên quan đến công việc
mà họ đang xin.
Từ khóa: NOT TRUE
Cản cứ vào các thông tin trong đoạn văn:
“However, yourabilityto negotiate effectively, for example, can be just as important as your
technicalskills.” Tuy nhiên, khả năng đàm phán hiệu quả của hạn cũng quan trọng như
các kĩ năng thực hành. Loại A.
“All information you give should be relevant, so carefully consider the job for which you are
applying.If you are applying for a job that is somewhat different than your current job, it is up
to you to draw a connectionfor the resume reviewer, so that they will understandhow your
skillswill fit in their organization.” - Tất cả các thông tin bạn cung cấp nên có mối liên
Question50. It can be inferred from the last paragraphthat
A. you should write accurately about your ability for the vacant position.
B. you should be modest about what you can do.
C. a resume reader is good enough to understand what you imply about
your ability in the CV.
D. you are allowed to exaggerate the truth of your competence if possible.
.
Có thể suy ra từ đoạn văn cuối rằng______.
A. bạn nên viết một cách chính xác về khả năng của bạn cho vị trí còn trống.
B. bạn nên khiêm tốn về những gì bạn có thể làm.
C. người đọc bản sơ yếu lý lịch đủ giỏi để hiểu những gì bạn hàm ý về khả năng của
bạn trong bản sơ yếu lịch.
D. bạn được phép phóng đại sự thật về khả năng của bạn nếu có thể.
Căn cứ vào thông tin đoạn cuối:
“If you are modest about the skills you can offer, or the results you have achieved, a
resume reader may take what you write literally, and be left with a low opinion of your
ability: you need to say exactly how good you are. On the other hand, of course, never
stretch the truth or lie.”
Nếu bạn khiêm tốn vê những kĩ năng bạn có thể làm hay kết quả bạn đã đạt được,
người đọc bản sơ yếu lí lịch sẽ hiểu theo đúng nghĩa đen bạn viết, và sẽ có ấn tượng
không tốt về khả năng của bạn: bạn cần phải nói chính xác bạn giỏi đến mức nào.
Mặc khác, tất nhiên, không bao giờ được phóng đại sự thật hay nói dối.
Question 1: His______of the generator is very famous.
A. invent B. inventive C. invention D. inventor
Căn cứ bằng tính từ sở hữu "his". Sau tính từ sở hữu + N
Trong đó:
invent (v): phát minh, sáng chế
inventive (a): có tài sáng chế, đầy sáng tạo
invention (n): sự phát minh, sự sáng chế
inventor (n) : người phát minh, người sáng tạo
Dịch nghĩa: Phát minh về máy phát điện của ông ấy rất
nổi tiếng.
Question 2: They rarely let her stay out late, _______?
A. do they B. don’t they C. does she D. doesn’t she
- câu hỏi đuôi
Câu hỏi đuôi luôn hỏi cho động từ và chủ ngữ ở mệnh đề
chính. Trong trường hợp này, chủ ngữ của mệnh đề
chính là “they” nên hai đáp án C và D (hỏi cho She) bị
loại.
Câu khẳng định có câu hỏi đuôi dạng phủ định còn câu
phủ định có câu hỏi đuôi dạng khẳng định. Câu đề bài
cho có “rarely” là trạng từ phủ định nên nó là câu phủ
định. Vì vậy, chọn đáp án A - do they (có phần hỏi đuôi
khẳng định).
Tạm dịch: Họ không cho cô ấy ra ngoài quá muộn đâu, có
đúng không?
Question 3: The commission estimates that at least
seven companies took___________ of the program.
A. advantage B. useC. benefit D. dominance
câu hỏi thành ngữ
Thành ngữ “take advantage of sb/sth”: tận dụng/ lợi dụng ai
hoặc cái gì đó. Xét 4 đáp án, chỉ chọn được A - advantage.
A. advantage (n): lợi thế
B. use (n)/ (v): công dụng (n) - thường đi trong “Make use of sth -
tận dụng cái gì đó/ sử dụng (v)
C. benefit (n): lợi ích
D. dominance (n): sự thống trị
Tạm dịch: Ủy ban này ước tính rằng có ít nhất 7 công ty đã lợi
dụng chương trình này
Question 4. Both husband and wife should be
responsible ________ doing the household chores.
A. with B. to C. for D. of
Kiến thức: Giới từ
Giải thích: be responsible for st/doing st: có, chịu trách
nhiệm về cái gì/làm gì
Tạm dịch: Cả hai vợ chồng đều nên có trách nhiệm làm
việc nhà.
Question 5: Mary was clearly nervous; she was sitting right
on the______ of his chair.
A. outside B. edge C. tip D. border
Outside: bên ngoài
Edge: mép
Tip: đầu
Border: biên giới
Câu này dịch như sau: Mary rõ ràng đang lo lắng; cô ây
ấy đang ngồi ngay ở mép ghế.
Question 6: I will stand here and wait for you you
come back.
A. because B. though C. so D. until
A. Because: bởi vì C. So: vì thế
B. Though: mặc dù D. Until: mãi đến khi
Dịch nghĩa: Anh sẽ vẫn đứng đây chờ em cho đến khi em quay lại.
Question 7. She bought a _________ lunchbox that she could
carry lunch to work.
A. new red plastic B. red plastic new
C. new plastic red D. plastic new red
Kiến thức: Trật tự tính từ
Giải thích:
Khi có nhiều tính từ cùng đứng trước 1 danh từ, sắp xếp chúng theo thứ tự:
OSASCOMP+ N. Trong đó:
O – opinion: quan điểm
S – size: kích thước
A – age: độ tuổi
S – shape: hình dạng
C – colour: màu sắc
O – origin: nguồn gốc
M – material: chất liệu
P – purpose: mục đích
N – noun: danh từ
Nếu có số thứ tự => đứng trước tính từ & danh từ
Tạm dịch: Cô ấy đã muamột hộp cơm bằng nhựa màu đỏ mới để có
thể mang đi ăn trưa để đi làm.
Question 8: The government hopes to______its plans
for introducing cable TV.
A. turn out B. carry out C. carry on D. keep on
+ turn out: hoá ra
+ carry on = keep on = go on = continue: tiếp tục
+ carry out: tiến hành
Dịch nghĩa: Chính phủ hi vọng thực hiện được kế hoạch
áp dụng cáp quang ti vi.
Question 9: When she came home from school
yesterday, her mother _______in the kitchen.
A. cooked B. was cooking C. is cooking D. cooks
Kiến thức: Thì trong tiếng Anh.
Giải thích:
Ta dùng thì quá khứ đơn và thì quá khứ tiếp diễn để
diễn tả một hành động đang xảy ra trong quá khứ thì
có một hành động khác xen vào.
Tạm dịch: Hôm qua khi cô ấy đi học về, mẹ cô đang
nấu ăn trong bếp.
Question 10: ______, he went straight home.
A. While he would finish his work
B. When he has finished his work
C. After he had finished his work
D. Before he has been finishing his work
Kiến thức: Thì quá khứ hoàn thành.
Giải thích: Cấu trúc: After S + had +Ved/ Vp2, S + Ved/
V2
Tạm dịch: Sau khi anh ấy hoàn thành xong công việc,
anh ấy đi thẳng về nhà.
Question 11: The jury______her compliments on her
excellent knowledge of the subject
A. paid B. gave C. made D. said
Câu này dịch như sau: Bồi thẩm đoàn khen ngợi cô ấy
kiến thức tuyệt vời về chủ đề này.
Pay compliment /ˈkɒmplɪmənt/ (n) on sth =
compliment/ˈkɒmplɪment/ (v) on sth:
khen ngợi ai về việc gì
Question 12: I took the children to the _______park last
weekend. They really enjoyed going on all the rides.
A. wildlife B. amusement C. national D. entertainment
Kiến thức: từ vựng
Wildlife (n): động vật hoang dã
Amusement(n): sự giải trí
National (a): thuộc về quốc gia
Entertainment(n): sự giải trí
Cụm danh từ: amusemnet park [ khu vui chơi giải trí]
Câu này dịch như sau: Tôi dẫn lũ trẻ đến khu vui chơi cuối tuần
trước. Chúng thật sự thích cả chuyến đi.
Question 13. This villa ———————- in 1975 by my
grandfather.
A. built B. was builtC. was build D. has built
Kiến thức: Câu bị động:
Giải chi tiết: Câu bị động thì Quá khứ đơn.
Hành động được nhấn mạnh là ngôi nhà được
xây dựng, trạng ngữ chỉ thời gian là in 1970.
S + động từ tobe + V-ed/V3
Tạm dịch : Căn biệt thự được xây vào năm 1975 bởi
ông tôi.
Question 14. ________ this movie last week, I don’t want to see it
again.
A. Having seen B. Being seen C. Having been seen D. Seeing
Kiến thức: Mệnh đề phân tử / Rút gọn mệnh đề đồng ngữ
Giải thích:
Khi 2 mệnh đề có cùng chủ ngữ (I) thì có thể rút gọn 1 trong 2
mệnh đề về dạng:
- V-ing / Having P2: nếu mệnh đề được rút gọn mang nghĩa chủ
động
- P2 (quá khứ phân từ): nếu mệnh đề được rút gọn mang nghĩa bị
động
Chủ ngữ “I” có thể làm chủ (tự thực hiện hành động”see” => nghĩa
chủ động. hành động đã xảy ra trong quá khứ. Having+V3/ed
Tạm dịch: Đã xem phim này tuần trước, tôi không muốn xem lại.
Question 15: The richer you are, _______.
A. you may become more worried
B. you more worried may become
C. the more worried you may become
D. the more worry you may become become
Xét các đáp án:
A. you may become more worried => Sai cấu trúc so sánh
kép
B. you more worried may become => Sai cấu trúc so sánh
kép
C. the more worried you may become → Cấu trúc so sánh
kép khi nói về 2 người hoặc sự vật:
The + so sánh hơn + S + V, the + so sánh hơn + S + V
D. the more worry you may become → sau more là
adj/adv, không phải động từ
Question 16: “How fashionable a pair of trainers you have!” -
“______________”
A. Do you want to know where I bought them?
B. Thanks for your compliment.
C. I know it’s fashionable.
D. Yes, of course.
Tình huống: “Bạn có đôi giày thể thao thật thời trang!”
A. Bạn có muốn biết tôi mua chúng ở đâu không?
B. Cảm ơn vì lời khen.
C. Tôi biết nó hợp thời trang.
D. Đúng rồi, tất nhiên là vậy.
=> Khi được khen, chúng ta nói cảm ơn
Question 17: “What can I do for you?” - “_______.”
A. No need to help. B. Thank you.
C. Thanks, I’m just looking. D. Sorry for not buying
anything
“Tôi có thể giúp gì cho bạn?” - “__________”
A. Không cần giúp.
B. Cảm ơn.
C. Cảm ơn, tôi chỉ đang xem thôi.
D. Xin lỗi vì đã không mua thứ gì.
=> Khi được hỏi có cần giúp gì không => chúng ta nói cảm
ơn
Question 18:
A. admit B. suggest C. remind D. manage
Kiến thức: Trọng âm từ có 2 âm tiết
Giải thích:
A. admit /ədˈmɪt/ B. suggest /səˈdʒest/
C. remind /rɪˈmaɪnd/ D. manage /ˈmænɪdʒ/
Question 19:
A. approval B. applicant C. energy D. influence
Kiến thức: Trọng âm từ có 3 âm tiết
Giải thích:
A. approval /əˈpruːvl/ B. applicant /ˈæplɪkənt/
C. energy /ˈenədʒi/ D. influence /ˈɪnfluəns/
Question 20:
A. watched B. cleaned C. stopped D. picked
Kiến thức: Phát âm “ed”
Giải thích:
Cách phát âm đuôi “ed”:
+ Đuôi “ed” được phát âm là /id/ khi động từ có
phát âm kết thúc là /t/ hay /d/
+ Đuôi “ed” được phát âm là /t/ khi động từ có phát
âm kết thúc là /s/,/f/,/p/,/ʃ/,/tʃ/,/k/
+ Đuôi “ed” được phát âm là /d/ với các trường hợp
còn lại
A. watched /wɒtʃt/ B. cleaned /kliːnd/
C. stopped /stɒpt/ D. picked /pɪkt/
Question 21:
A. hole B. home C. come D. hold
Kiến thức: Phát âm “o”
Giải thích:
A. hole /həʊl/ B. home /həʊm/
C. come /kʌm/ D. hold /həʊld/
Question 22: I will not stand for your bad attitude any longer.
A. like B. sit C. tolerate D. care
Kiến thức: Từ đồng nghĩa
Giải thích:
stand for: chịu đựng
A. like (v): thích B. sit (v): ngồi
C. tolerate (v): chịu đựng D. care (v): quan tâm
=> stand for = tolerate
Tạm dịch: Tôi sẽ không chịu đựng thái độ xấu của bạn nữa.
Question 23. The repeated commercials on TV distract many
viewers from watching their favorite films
.A. advertisements B. contests C. business D. economics
Tạm dịch: Những quảng cáo lặp đi lặp lại trên ti vi sẽ làm cho người
ta bị xao nhãng khỏi việc xem các bộ phim yêu thích của họ.
commercials = advertisements: những mẩu quảng cáo
Question 24: In some countries, so few students are accepted by
the universities that admission is almost a guarantee of a good
job upon graduation.
A. a promise B. an uncertainty C. an assurance D. a pledge
Từ trái nghĩa - kiến thức về thành ngữ
Tạm dịch: Ở một số quốc gia, rất ít sinh viên được các trường đại học
chấp nhận mà việc nhập học gần như là một sự đảm bảo cho một
công việc tốt khi tốt nghiệp.
=> guarantee /ˌɡærənˈtiː/ (n): lời cam kết, đảm bảo
Xét các đáp án:
A. promise (n): lời hứa
B. uncertainty /əˈʃʊərəns/ (n): tính không chắc chắn
C. assurance /ʌnˈsɜːrtnti/ (n): lời cam kết, hứa hẹn
D. pledge /pledʒ/ (n): lời cam kết, lời nguyện
=> Do đó: a guarantee >< an uncertainty
Question 25: The burglar crept into the house without making
any noise. That's why no one heard anything.
A. inaudibly B. boisterously C. shrilly D. hurly-burly
Từ trái nghĩa - kiến thức về thành ngữ
Tạm dịch: Tên trộm lẻn vào nhà mà không hề tạo ra chút tiếng ồn
nào. Đó là lý do không ai nghe thấy gì.
=> without making any noise: không hề tạo ra chút tiếng ồn nào
Xét các đáp án:
A. inaudibly /ɪnˈɔːdəbli/ (adv): một cách vô thanh
B. boisterously /ˈbɔɪstərəsli/ (adv): một cách ầm ĩ, náo nhiệt (tràn
đầy năng lượng)
C. shrilly /ˈʃrɪlli/ (adv): một cách the thé, inh tai (gây khó chịu)
D. hurly-burly /ˈhɜːrli bɜːrli/ (adv): một hoạt động, tình huống ồn
ào, náo nhiệt
=> Do đó: without making any noise >< boisterously
Question 26: Seth informed us of his retirement from the
company. He did it when arriving at the meeting.
A. Only after his retiring from the company did Seth tell us about
his arrival at the meeting.
B. Not until Seth said to us that he would leave the company did
he turn up at the meeting.
C. Hardly had Seth notified us of his retiring from the company
when he arrived at the meeting
D. No sooner had Seth arrived at the meeting than we were told
about his leaving the company
Kiến thức: Đảo ngữ
Giải thích:
No sooner + had + S + PP than + S + Ved/ V2: Ngay khi...
thì...
Question 27: They were late for the meeting. The heavy was heavy.
A. If it snowed heavily, they would be late for the meeting.
B. Had it not snowed heavily, they would have been late for the
meeting.
C. But for the heavy snow, they wouldn't have been late for the
meeting.
D. If it didn't snow heavily, they wouldn't be late for the meeting.
Tạm dịch: Họ đến muộn buổi gặp mặt. Tuyết rơi nhiều.
They were late for the meeting. The snow was heavy. (nguyên nhân và kết
quả)
( Quá khứ đơn)
Tình huống ở quá khứ nên viết lại bằng câu điều kiện loại 3.
A. Sai vì là câu điều kiện loại 2
B. Sai vì là câu điều kiện loại 3 nhưng không phù hợp về nghĩa với câu đã
cho (Nếu không có tuyết rơi nhiều, họ đã đến muộn buổi gặp mặt)
C. Đúng vì phù hợp là câu điều kiện loại 3 (rút gọn) và phù hợp về nghĩa.
D. Sai vì là câu điều kiện loại 2
Question 28: Mrs. Hoa and her
friends from Vietnam plan to attend
the festival now
* Chia thì theo đúng trạng ngữ chỉ thời gian: now
Mrs. Hoa and her friends from Vietnam (plan) to
attend the festival now. Ta phải chia động từ plan
thành thì hiện tại tiếp diễn
Question 29: Some manufacturers are not only
raising their prices but also decreasing the
production of its products.
Kiểm tra kiến thức: Sự hoà hợp giữa chủ ngữ và tính từ sở
hữu cách
Some manufacturers số nhiều → their
Dịch: Một số nhà sản xuất không những tăng giá thành
mà còn giảm năng suất của các sản phẩm.
Question 30: The whole matter is farther
complicated by the fact that Amanda and Jo refuse
to speak to each other.
Cả “farther” và “further” đều là dạng so sánh hơn của từ “far”. Tuy
nhiên, chúng có sự khác nhau như
sau:
+ Farther: xa hơn (về khoảng cách địa lí)
+ Further: xa hơn, sâu hơn (về mức độ, tính chất)
Tạm dịch: Toàn bộ vấn đề phức tạp hơn bởi thực tế là Amanda và
Jo từ chối nói chuyện với nhau.
=> Đáp án là B (farther -further)
Cấu trúc khác:
Refuse to do st: từ chối làm gì
Question 31: A supermarket is more convenient than a
shopping centre.
A. A shopping centre is not as convenient as a supermarket.
B. A shopping centre is more convenient than a supermarket.
C. A supermarket is not as convenient as a shopping centre.
D. A supermarket is as inconvenient as a shopping centre.
Kiến thức: diễn đạt câu với Câu so sánh
Giải thích:
Một siêu thị thuận tiện hơn một trung tâm mua sắm.
A. Một trung tâm mua sắm không thuận tiện như siêu thị.
B. Một trung tâm mua sắm thuận tiện hơn siêu thị.
C. Một siêu thị không thuận tiện như một trung tâm mua sắm.
D. Một siêu thị bất tiện như một trung tâm mua sắm.
Question 32: "It was your fault. You broke my windows,
"said the woman to him.
A. The woman insisted him on breaking her windows.
B. The woman advised him to break her windows.
C. The woman told him to break her windows.
D. The woman blamed him for having broken her windows.
Tạm dịch:
“ Đó là lỗi của cháu. Cháu đã làm vỡ cửa kính nhà bác” , Người phụ
nữ nói với cậu bé.
A. Người phụ nữ khăng khăng bắt cậu bé làm vỡ của kính nhà bà ta.
B. Người phụ nữ khuyên cậu bé làm vỡ của kính nhà bà ta.
C. Người phụ nữ bảo cậu bé làm vỡ của kính nhà bà ta.
D. Người phụ nữ đổ lỗi cho cậu bé vì đã làm vỡ của kính nhà bà ta.
Question 33. It was a mistake for Tony to buy that house.
A. Tony couldn’t have bought that house.
B. Tony can’t have bought that house.
C. Tony needn’t have bought that house.
D. Tony shouldn’t have bought that house.
Kiến thức: Động từ khuyết thiếu
Giải thích:
couldn’t have P2: không thể làm gì trong quá khứ
can’t have P2: không thể nào
needn’t have P2: đáng lẽ ra không cần
shouldn’t have P2: đã không nên làm gì
Tạm dịch: Đó là lỗi của Tony khi mua căn nhà đó.
D. Tony đã nên không mua căn nhà đó.
Question 34:
“His sister learned the household skills that would prepare her
to become ..........wife and mother.”
A. many B. the C. an D. a
Giái thích: vì his sister là số ít nên loại bỏ A; Cụm từ
“become” trở thành….không xác định, số ít nên dùng mạo
từ “a”
(Chị gái cậu ấy đã học các kĩ năng nội trợ sẽ chuẩn bị cho
tương lai cô ấy trở thành người vợ và người mẹ.)
Question 35:
“Nowadays young people grow up in a much freer society ........
they enjoy almost unlimited career opportunities”
A. where B. when C. why D. whom
where: trạng từ quan hệ, thay thế cho từ/ cụm từ nơi chốn
when: trạng từ quan hệ, thay thế cho từ/ cụm từ chỉ thởi gian
why: trạng từ quan hệ, thay thế cho từ/ cụm từ chỉ lý do
whom: đại từ quan hệ, thay thế cho tân ngữ chỉ người
Trong câu này từ cần điền vào là đại từ quan hệ thay thế cho
cụm từ nơi chốn “in a society”.
(Ngày nay, những người trẻ lớn lên trong một xã hội tự do hơn
nhiều, nơi mà họ có được hầu như vô số cơ hội nghề nghiệp.)
Question 36:
“In recent years, there ............an enormous increase in
the kinds of vocations from which it is possible to
choose”
A. had been B. has been C. will be D. was
In recent years: trong những năm gần đây => động từ
chia ở thì hiện tại hoàn thành
(Trong những năm gần đây, đã có sự tăng mạnh về
các loại nghề nghiệp để cho mọi người lựa chọn.)
Question 37: “In addition, many of the barriers to career
opportunity that existed only a few decades ago, such as .............
based on sex or religion or ethnic origins...” religion or ethnic
origins...”
A. judgement B. perception C. devotion D. discrimination
judgment (n): sự đánh giá, óc phán đoán
perception (n): sự tiếp nhận, sự nhận thức
devotion (n): sự tận tụy, sự hiến dâng
discrimination (n): sự phân biệt đối xử
(Thêm vào đó, nhiều rào cản đối với cơ hội nghề nghiệp đã
tồn tại một vài thập kỉ trước đây như sự phân biệt đối xử
dựa trên giới tính hoặc tôn giáo hoặc nguồn gốc dân tộc...)
Question 38: “In addition, many of the barriers to career
opportunity that existed only a few decades ago, such as
discrimination based on sex or religion or ethnic origins, are
............. disappearing.”
A. rapidly B. incessantly C. categorically D. vigilantly
rapidly (adv): nhanh chóng
incessantly (adv): không ngừng, không dứt
categorically (adv): rõ ràng, minh bạch
vigilantly (adv): thận trọng, cảnh giác
(Thêm vào đó, nhiều rào cản đối với cơ hội nghề nghiệp
đã tồn tại chỉ một vài thập kỉ trước đây như sự phân biệt
đối xử dựa trên giới tính hoặc tôn giảo hoặc nguồn gốc
dân tộc đang nhanh chóng biến mất.)
Question39. What is the main focus of this passage?
A. Jobs on Wall Street
B. Types of graduate degrees
C. Changes in enrollment for MBA schools
D. How schools are changing to reflect the economy
Trọng tâm chính của bài đọc này là gì?
Các công việc trên phố Wall
Các loại bằng tốt nghiệp
Những thay đổi trong việc tuyển sinh của các trường MBA
Cách thức các trường đang thay đổi để phản ánh nền kinh tế
Dẫn chứng:
Đoạn 1: After twenty years of growing student enrollments and economic
prosperity, business schools in the United States have started to face harder
times... (Sau 20 năm gia tăng số sinh viên đăng kí học và sự phát triển thịnh
vượng của nền kinh tế thì các trường kinh doanh ở Mỹ đã bắt đầu phải đối mặt
với những khó khăn...)
Đoạn 2: There are two factors causing this decrease in students seeking an MBA
degree. (Có 2 yếu tố dẫn đến tình trạng giảm số lượng sinh viên theo đuổi bằng
MBA.)
Question 40. The word “prosperity” in the first paragraph
could be best replaced by which of the following?
A. success B. surplus C. nurturing D. education
success (n): sự thành công
surplus (n): số dư
nurture (n): sự nuôi dưỡng
education (n): giáo dục
“After twenty years of growing student enrollments and
economic prosperity, business schools in the United States
have started to face harder times... (Sau 20 năm gia tăng
số sinh viên đăng kí học và sự phát triển thịnh vượng của
nền kinh tế thì các trường kinh doanh ở Mỹ đã bắt đầu
phải đối mặt với những khó khăn...)”
Do đó: prosperity: ~ success: sự thịnh vượng, sự thành
công
Question 41. Which of the following business schools has
shown an increase in enrollment?
A. Princeton B. Harvard C. Stanford D. Yale
Trường kinh doanh nào sau đây cho thấy sự gia tăng trong việc
tuyển sinh?
Princeton
Harvard
Stanford
Yale
Dẫn chứng: Only Harvard’s MBA School has shown a substantial
increase in enrollment in recent years. Both Princeton and Stanford
have seen decreases in their enrollments. (Chỉ có trường kinh
doanh MBA ở Harvard đã cho thấy việc tuyển sinh tăng đáng kể
trong những năm gần đây. Cả Princeton và Standford đều cho thấy
việc tuyển sinh giảm xuống.)
Question 42. Which of the following descriptions most likely
applies to Wall Street?
A. a center for international affairs B. a major financial center
C. a shopping district D. a neighborhood in New York
Miêu tả nào sau đây có thể thích hợp nhất với Phố Wall?
Một trung tâm ngoại giao
Một trung tâm tài chính lớn
Một khu vực mua sắm
Một vùng lân cận ở New York
Dẫn chứng: The first one is that many graduates of four-year
colleges are finding that an MBA degree does not guarantee a plush
job on Wall Street, or in other financial districts of major American
cities... (Yếu tố đầu tiên là nhiều sinh viên tốt nghiệp đại học 4 năm
nhận thấy rằng bằng MBA không đảm bảo công việc tốt ở phố Wall
hoặc các khu vực tài chính khác ở các thành phố lớn của nước Mỹ.)
Question 43: As used in the second paragraph, the
word “struggling” is closest in meaning to_____.
A. evolving B. plunging C. starting D. striving
evolving: tiến hóa
plunging: lao xuống, đẩy vào
starting: bắt đầu
striving: cố gắng, đấu tranh
Business needs are changing, and MBA schools are
struggling to meet the new demands. (Nhu cầu kinh
doanh đang thay đổi và các trường MBA đang cố gắng
hết sức để đáp ứng các yêu cầu mới.)”
Do đó: struggling ~ striving: cố gắng phấn đẩu
DE PHAT TRIEN THEO CAU TRUC DE MINH HOA 2022 MON TIENG ANH DE 1 5.pdf
DE PHAT TRIEN THEO CAU TRUC DE MINH HOA 2022 MON TIENG ANH DE 1 5.pdf
DE PHAT TRIEN THEO CAU TRUC DE MINH HOA 2022 MON TIENG ANH DE 1 5.pdf
DE PHAT TRIEN THEO CAU TRUC DE MINH HOA 2022 MON TIENG ANH DE 1 5.pdf
DE PHAT TRIEN THEO CAU TRUC DE MINH HOA 2022 MON TIENG ANH DE 1 5.pdf
DE PHAT TRIEN THEO CAU TRUC DE MINH HOA 2022 MON TIENG ANH DE 1 5.pdf
DE PHAT TRIEN THEO CAU TRUC DE MINH HOA 2022 MON TIENG ANH DE 1 5.pdf
DE PHAT TRIEN THEO CAU TRUC DE MINH HOA 2022 MON TIENG ANH DE 1 5.pdf
DE PHAT TRIEN THEO CAU TRUC DE MINH HOA 2022 MON TIENG ANH DE 1 5.pdf
DE PHAT TRIEN THEO CAU TRUC DE MINH HOA 2022 MON TIENG ANH DE 1 5.pdf
DE PHAT TRIEN THEO CAU TRUC DE MINH HOA 2022 MON TIENG ANH DE 1 5.pdf
DE PHAT TRIEN THEO CAU TRUC DE MINH HOA 2022 MON TIENG ANH DE 1 5.pdf
DE PHAT TRIEN THEO CAU TRUC DE MINH HOA 2022 MON TIENG ANH DE 1 5.pdf
DE PHAT TRIEN THEO CAU TRUC DE MINH HOA 2022 MON TIENG ANH DE 1 5.pdf
DE PHAT TRIEN THEO CAU TRUC DE MINH HOA 2022 MON TIENG ANH DE 1 5.pdf
DE PHAT TRIEN THEO CAU TRUC DE MINH HOA 2022 MON TIENG ANH DE 1 5.pdf
DE PHAT TRIEN THEO CAU TRUC DE MINH HOA 2022 MON TIENG ANH DE 1 5.pdf
DE PHAT TRIEN THEO CAU TRUC DE MINH HOA 2022 MON TIENG ANH DE 1 5.pdf
DE PHAT TRIEN THEO CAU TRUC DE MINH HOA 2022 MON TIENG ANH DE 1 5.pdf
DE PHAT TRIEN THEO CAU TRUC DE MINH HOA 2022 MON TIENG ANH DE 1 5.pdf
DE PHAT TRIEN THEO CAU TRUC DE MINH HOA 2022 MON TIENG ANH DE 1 5.pdf
DE PHAT TRIEN THEO CAU TRUC DE MINH HOA 2022 MON TIENG ANH DE 1 5.pdf
DE PHAT TRIEN THEO CAU TRUC DE MINH HOA 2022 MON TIENG ANH DE 1 5.pdf
DE PHAT TRIEN THEO CAU TRUC DE MINH HOA 2022 MON TIENG ANH DE 1 5.pdf
DE PHAT TRIEN THEO CAU TRUC DE MINH HOA 2022 MON TIENG ANH DE 1 5.pdf
DE PHAT TRIEN THEO CAU TRUC DE MINH HOA 2022 MON TIENG ANH DE 1 5.pdf
DE PHAT TRIEN THEO CAU TRUC DE MINH HOA 2022 MON TIENG ANH DE 1 5.pdf
DE PHAT TRIEN THEO CAU TRUC DE MINH HOA 2022 MON TIENG ANH DE 1 5.pdf
DE PHAT TRIEN THEO CAU TRUC DE MINH HOA 2022 MON TIENG ANH DE 1 5.pdf
DE PHAT TRIEN THEO CAU TRUC DE MINH HOA 2022 MON TIENG ANH DE 1 5.pdf
DE PHAT TRIEN THEO CAU TRUC DE MINH HOA 2022 MON TIENG ANH DE 1 5.pdf
DE PHAT TRIEN THEO CAU TRUC DE MINH HOA 2022 MON TIENG ANH DE 1 5.pdf
DE PHAT TRIEN THEO CAU TRUC DE MINH HOA 2022 MON TIENG ANH DE 1 5.pdf
DE PHAT TRIEN THEO CAU TRUC DE MINH HOA 2022 MON TIENG ANH DE 1 5.pdf
DE PHAT TRIEN THEO CAU TRUC DE MINH HOA 2022 MON TIENG ANH DE 1 5.pdf
DE PHAT TRIEN THEO CAU TRUC DE MINH HOA 2022 MON TIENG ANH DE 1 5.pdf
DE PHAT TRIEN THEO CAU TRUC DE MINH HOA 2022 MON TIENG ANH DE 1 5.pdf
DE PHAT TRIEN THEO CAU TRUC DE MINH HOA 2022 MON TIENG ANH DE 1 5.pdf
DE PHAT TRIEN THEO CAU TRUC DE MINH HOA 2022 MON TIENG ANH DE 1 5.pdf
DE PHAT TRIEN THEO CAU TRUC DE MINH HOA 2022 MON TIENG ANH DE 1 5.pdf
DE PHAT TRIEN THEO CAU TRUC DE MINH HOA 2022 MON TIENG ANH DE 1 5.pdf
DE PHAT TRIEN THEO CAU TRUC DE MINH HOA 2022 MON TIENG ANH DE 1 5.pdf
DE PHAT TRIEN THEO CAU TRUC DE MINH HOA 2022 MON TIENG ANH DE 1 5.pdf
DE PHAT TRIEN THEO CAU TRUC DE MINH HOA 2022 MON TIENG ANH DE 1 5.pdf
DE PHAT TRIEN THEO CAU TRUC DE MINH HOA 2022 MON TIENG ANH DE 1 5.pdf
DE PHAT TRIEN THEO CAU TRUC DE MINH HOA 2022 MON TIENG ANH DE 1 5.pdf
DE PHAT TRIEN THEO CAU TRUC DE MINH HOA 2022 MON TIENG ANH DE 1 5.pdf
DE PHAT TRIEN THEO CAU TRUC DE MINH HOA 2022 MON TIENG ANH DE 1 5.pdf
DE PHAT TRIEN THEO CAU TRUC DE MINH HOA 2022 MON TIENG ANH DE 1 5.pdf
DE PHAT TRIEN THEO CAU TRUC DE MINH HOA 2022 MON TIENG ANH DE 1 5.pdf
DE PHAT TRIEN THEO CAU TRUC DE MINH HOA 2022 MON TIENG ANH DE 1 5.pdf
DE PHAT TRIEN THEO CAU TRUC DE MINH HOA 2022 MON TIENG ANH DE 1 5.pdf
DE PHAT TRIEN THEO CAU TRUC DE MINH HOA 2022 MON TIENG ANH DE 1 5.pdf
DE PHAT TRIEN THEO CAU TRUC DE MINH HOA 2022 MON TIENG ANH DE 1 5.pdf
DE PHAT TRIEN THEO CAU TRUC DE MINH HOA 2022 MON TIENG ANH DE 1 5.pdf
DE PHAT TRIEN THEO CAU TRUC DE MINH HOA 2022 MON TIENG ANH DE 1 5.pdf
DE PHAT TRIEN THEO CAU TRUC DE MINH HOA 2022 MON TIENG ANH DE 1 5.pdf
DE PHAT TRIEN THEO CAU TRUC DE MINH HOA 2022 MON TIENG ANH DE 1 5.pdf
DE PHAT TRIEN THEO CAU TRUC DE MINH HOA 2022 MON TIENG ANH DE 1 5.pdf
DE PHAT TRIEN THEO CAU TRUC DE MINH HOA 2022 MON TIENG ANH DE 1 5.pdf
DE PHAT TRIEN THEO CAU TRUC DE MINH HOA 2022 MON TIENG ANH DE 1 5.pdf
DE PHAT TRIEN THEO CAU TRUC DE MINH HOA 2022 MON TIENG ANH DE 1 5.pdf
DE PHAT TRIEN THEO CAU TRUC DE MINH HOA 2022 MON TIENG ANH DE 1 5.pdf
DE PHAT TRIEN THEO CAU TRUC DE MINH HOA 2022 MON TIENG ANH DE 1 5.pdf
DE PHAT TRIEN THEO CAU TRUC DE MINH HOA 2022 MON TIENG ANH DE 1 5.pdf
DE PHAT TRIEN THEO CAU TRUC DE MINH HOA 2022 MON TIENG ANH DE 1 5.pdf
DE PHAT TRIEN THEO CAU TRUC DE MINH HOA 2022 MON TIENG ANH DE 1 5.pdf
DE PHAT TRIEN THEO CAU TRUC DE MINH HOA 2022 MON TIENG ANH DE 1 5.pdf
DE PHAT TRIEN THEO CAU TRUC DE MINH HOA 2022 MON TIENG ANH DE 1 5.pdf
DE PHAT TRIEN THEO CAU TRUC DE MINH HOA 2022 MON TIENG ANH DE 1 5.pdf
DE PHAT TRIEN THEO CAU TRUC DE MINH HOA 2022 MON TIENG ANH DE 1 5.pdf
DE PHAT TRIEN THEO CAU TRUC DE MINH HOA 2022 MON TIENG ANH DE 1 5.pdf
DE PHAT TRIEN THEO CAU TRUC DE MINH HOA 2022 MON TIENG ANH DE 1 5.pdf
DE PHAT TRIEN THEO CAU TRUC DE MINH HOA 2022 MON TIENG ANH DE 1 5.pdf
DE PHAT TRIEN THEO CAU TRUC DE MINH HOA 2022 MON TIENG ANH DE 1 5.pdf
DE PHAT TRIEN THEO CAU TRUC DE MINH HOA 2022 MON TIENG ANH DE 1 5.pdf
DE PHAT TRIEN THEO CAU TRUC DE MINH HOA 2022 MON TIENG ANH DE 1 5.pdf
DE PHAT TRIEN THEO CAU TRUC DE MINH HOA 2022 MON TIENG ANH DE 1 5.pdf
DE PHAT TRIEN THEO CAU TRUC DE MINH HOA 2022 MON TIENG ANH DE 1 5.pdf
DE PHAT TRIEN THEO CAU TRUC DE MINH HOA 2022 MON TIENG ANH DE 1 5.pdf
DE PHAT TRIEN THEO CAU TRUC DE MINH HOA 2022 MON TIENG ANH DE 1 5.pdf
DE PHAT TRIEN THEO CAU TRUC DE MINH HOA 2022 MON TIENG ANH DE 1 5.pdf
DE PHAT TRIEN THEO CAU TRUC DE MINH HOA 2022 MON TIENG ANH DE 1 5.pdf
DE PHAT TRIEN THEO CAU TRUC DE MINH HOA 2022 MON TIENG ANH DE 1 5.pdf
DE PHAT TRIEN THEO CAU TRUC DE MINH HOA 2022 MON TIENG ANH DE 1 5.pdf
DE PHAT TRIEN THEO CAU TRUC DE MINH HOA 2022 MON TIENG ANH DE 1 5.pdf
DE PHAT TRIEN THEO CAU TRUC DE MINH HOA 2022 MON TIENG ANH DE 1 5.pdf
DE PHAT TRIEN THEO CAU TRUC DE MINH HOA 2022 MON TIENG ANH DE 1 5.pdf
DE PHAT TRIEN THEO CAU TRUC DE MINH HOA 2022 MON TIENG ANH DE 1 5.pdf
DE PHAT TRIEN THEO CAU TRUC DE MINH HOA 2022 MON TIENG ANH DE 1 5.pdf
DE PHAT TRIEN THEO CAU TRUC DE MINH HOA 2022 MON TIENG ANH DE 1 5.pdf
DE PHAT TRIEN THEO CAU TRUC DE MINH HOA 2022 MON TIENG ANH DE 1 5.pdf
DE PHAT TRIEN THEO CAU TRUC DE MINH HOA 2022 MON TIENG ANH DE 1 5.pdf
DE PHAT TRIEN THEO CAU TRUC DE MINH HOA 2022 MON TIENG ANH DE 1 5.pdf
DE PHAT TRIEN THEO CAU TRUC DE MINH HOA 2022 MON TIENG ANH DE 1 5.pdf
DE PHAT TRIEN THEO CAU TRUC DE MINH HOA 2022 MON TIENG ANH DE 1 5.pdf
DE PHAT TRIEN THEO CAU TRUC DE MINH HOA 2022 MON TIENG ANH DE 1 5.pdf
DE PHAT TRIEN THEO CAU TRUC DE MINH HOA 2022 MON TIENG ANH DE 1 5.pdf
DE PHAT TRIEN THEO CAU TRUC DE MINH HOA 2022 MON TIENG ANH DE 1 5.pdf
DE PHAT TRIEN THEO CAU TRUC DE MINH HOA 2022 MON TIENG ANH DE 1 5.pdf
DE PHAT TRIEN THEO CAU TRUC DE MINH HOA 2022 MON TIENG ANH DE 1 5.pdf
DE PHAT TRIEN THEO CAU TRUC DE MINH HOA 2022 MON TIENG ANH DE 1 5.pdf
DE PHAT TRIEN THEO CAU TRUC DE MINH HOA 2022 MON TIENG ANH DE 1 5.pdf
DE PHAT TRIEN THEO CAU TRUC DE MINH HOA 2022 MON TIENG ANH DE 1 5.pdf
DE PHAT TRIEN THEO CAU TRUC DE MINH HOA 2022 MON TIENG ANH DE 1 5.pdf
DE PHAT TRIEN THEO CAU TRUC DE MINH HOA 2022 MON TIENG ANH DE 1 5.pdf
DE PHAT TRIEN THEO CAU TRUC DE MINH HOA 2022 MON TIENG ANH DE 1 5.pdf
DE PHAT TRIEN THEO CAU TRUC DE MINH HOA 2022 MON TIENG ANH DE 1 5.pdf
DE PHAT TRIEN THEO CAU TRUC DE MINH HOA 2022 MON TIENG ANH DE 1 5.pdf

Mais conteúdo relacionado

Mais procurados

Đề Tiếng Anh 12 cơ bản unit 12 có đáp án - VipLam.Net
Đề Tiếng Anh 12 cơ bản unit 12 có đáp án - VipLam.NetĐề Tiếng Anh 12 cơ bản unit 12 có đáp án - VipLam.Net
Đề Tiếng Anh 12 cơ bản unit 12 có đáp án - VipLam.NetThùy Linh
 
đáP án và giải thích đề 9
đáP án và giải thích đề 9đáP án và giải thích đề 9
đáP án và giải thích đề 9Huynh ICT
 
Đề Tiếng Anh 12 cơ bản unit 4 có đáp án - VipLam.Net
Đề Tiếng Anh 12 cơ bản unit 4 có đáp án - VipLam.NetĐề Tiếng Anh 12 cơ bản unit 4 có đáp án - VipLam.Net
Đề Tiếng Anh 12 cơ bản unit 4 có đáp án - VipLam.NetThùy Linh
 
Dịch nghĩa và giải thích chi tiết starter toeic unit 3
Dịch nghĩa và giải thích chi tiết starter toeic unit 3Dịch nghĩa và giải thích chi tiết starter toeic unit 3
Dịch nghĩa và giải thích chi tiết starter toeic unit 3my nguyễn
 
đáP án và giải thích đề 4
đáP án và giải thích đề 4đáP án và giải thích đề 4
đáP án và giải thích đề 4Huynh ICT
 
Starter Toeic
Starter ToeicStarter Toeic
Starter ToeicVõ Phúc
 
Dịch nghĩa và giải thích chi tiết starter toeic unit 2
Dịch nghĩa và giải thích chi tiết starter toeic unit 2Dịch nghĩa và giải thích chi tiết starter toeic unit 2
Dịch nghĩa và giải thích chi tiết starter toeic unit 2my nguyễn
 
Trắc nghiệm từng unit tiếng Anh 12 (16 unit) + key
Trắc nghiệm từng unit tiếng Anh 12 (16 unit) + keyTrắc nghiệm từng unit tiếng Anh 12 (16 unit) + key
Trắc nghiệm từng unit tiếng Anh 12 (16 unit) + keyThùy Linh
 
40 ĐỀ THI THỬ TỐT NGHIỆP THPT - MÔN TIẾNG ANH - NĂM 2023 - SOẠN CHUẨN CẤU TR...
40 ĐỀ THI THỬ TỐT NGHIỆP THPT - MÔN TIẾNG ANH - NĂM 2023 - SOẠN CHUẨN CẤU TR...40 ĐỀ THI THỬ TỐT NGHIỆP THPT - MÔN TIẾNG ANH - NĂM 2023 - SOẠN CHUẨN CẤU TR...
40 ĐỀ THI THỬ TỐT NGHIỆP THPT - MÔN TIẾNG ANH - NĂM 2023 - SOẠN CHUẨN CẤU TR...Nguyen Thanh Tu Collection
 
Đề Tiếng Anh 12 cơ bản unit 15 có đáp án - VipLam.Net
Đề Tiếng Anh 12 cơ bản unit 15 có đáp án - VipLam.NetĐề Tiếng Anh 12 cơ bản unit 15 có đáp án - VipLam.Net
Đề Tiếng Anh 12 cơ bản unit 15 có đáp án - VipLam.NetThùy Linh
 
BÀI TẬP BỔ TRỢ TIẾNG ANH LỚP 10 GLOBAL SUCCESS NĂM 2023 CÓ FILE NGHE - 2 HỌC ...
BÀI TẬP BỔ TRỢ TIẾNG ANH LỚP 10 GLOBAL SUCCESS NĂM 2023 CÓ FILE NGHE - 2 HỌC ...BÀI TẬP BỔ TRỢ TIẾNG ANH LỚP 10 GLOBAL SUCCESS NĂM 2023 CÓ FILE NGHE - 2 HỌC ...
BÀI TẬP BỔ TRỢ TIẾNG ANH LỚP 10 GLOBAL SUCCESS NĂM 2023 CÓ FILE NGHE - 2 HỌC ...Nguyen Thanh Tu Collection
 
Xstk de thi mau 01 (may12)
Xstk de thi mau 01 (may12)Xstk de thi mau 01 (may12)
Xstk de thi mau 01 (may12)Võ Thùy Linh
 
Bài tập trắc nghiệm xác suất thống kê
Bài tập trắc nghiệm xác suất thống kêBài tập trắc nghiệm xác suất thống kê
Bài tập trắc nghiệm xác suất thống kêHọc Huỳnh Bá
 
De tieng-anh-part5-toeic-co-dap-an-loi-giai-chi-tiet
De tieng-anh-part5-toeic-co-dap-an-loi-giai-chi-tietDe tieng-anh-part5-toeic-co-dap-an-loi-giai-chi-tiet
De tieng-anh-part5-toeic-co-dap-an-loi-giai-chi-tietCharlie Cúc Cu
 
50 ĐỀ PHÁT TRIỂN THEO CẤU TRÚC ĐỀ MINH HỌA BGD NGÀY 22-3-2024 KỲ THI TỐT NGHI...
50 ĐỀ PHÁT TRIỂN THEO CẤU TRÚC ĐỀ MINH HỌA BGD NGÀY 22-3-2024 KỲ THI TỐT NGHI...50 ĐỀ PHÁT TRIỂN THEO CẤU TRÚC ĐỀ MINH HỌA BGD NGÀY 22-3-2024 KỲ THI TỐT NGHI...
50 ĐỀ PHÁT TRIỂN THEO CẤU TRÚC ĐỀ MINH HỌA BGD NGÀY 22-3-2024 KỲ THI TỐT NGHI...Nguyen Thanh Tu Collection
 
Đề Tiếng Anh 12 cơ bản unit 10 có đáp án - VipLam.Net
Đề Tiếng Anh 12 cơ bản unit 10 có đáp án - VipLam.NetĐề Tiếng Anh 12 cơ bản unit 10 có đáp án - VipLam.Net
Đề Tiếng Anh 12 cơ bản unit 10 có đáp án - VipLam.NetThùy Linh
 
ĐỀ CƯƠNG + TEST ÔN TẬP GIỮA KÌ 2 TIẾNG ANH 11 - GLOBAL SUCCESS (THEO CHUẨN MI...
ĐỀ CƯƠNG + TEST ÔN TẬP GIỮA KÌ 2 TIẾNG ANH 11 - GLOBAL SUCCESS (THEO CHUẨN MI...ĐỀ CƯƠNG + TEST ÔN TẬP GIỮA KÌ 2 TIẾNG ANH 11 - GLOBAL SUCCESS (THEO CHUẨN MI...
ĐỀ CƯƠNG + TEST ÔN TẬP GIỮA KÌ 2 TIẾNG ANH 11 - GLOBAL SUCCESS (THEO CHUẨN MI...Nguyen Thanh Tu Collection
 
đồ Thị hàm số chứa giá trị tuyệt đối
đồ Thị hàm số chứa giá trị tuyệt đốiđồ Thị hàm số chứa giá trị tuyệt đối
đồ Thị hàm số chứa giá trị tuyệt đốiHướng Trần Minh
 
Giải chi tiết đề tiếng anh THPTQG 2018 mã 403
Giải chi tiết đề tiếng anh THPTQG 2018 mã 403Giải chi tiết đề tiếng anh THPTQG 2018 mã 403
Giải chi tiết đề tiếng anh THPTQG 2018 mã 403Viet Le
 
Bai giai chi tiet mon anh (chuyen) tuyen sinh lop 10 tphcm 2013
Bai giai chi tiet mon anh (chuyen) tuyen sinh lop 10 tphcm   2013Bai giai chi tiet mon anh (chuyen) tuyen sinh lop 10 tphcm   2013
Bai giai chi tiet mon anh (chuyen) tuyen sinh lop 10 tphcm 2013Tommy Bảo
 

Mais procurados (20)

Đề Tiếng Anh 12 cơ bản unit 12 có đáp án - VipLam.Net
Đề Tiếng Anh 12 cơ bản unit 12 có đáp án - VipLam.NetĐề Tiếng Anh 12 cơ bản unit 12 có đáp án - VipLam.Net
Đề Tiếng Anh 12 cơ bản unit 12 có đáp án - VipLam.Net
 
đáP án và giải thích đề 9
đáP án và giải thích đề 9đáP án và giải thích đề 9
đáP án và giải thích đề 9
 
Đề Tiếng Anh 12 cơ bản unit 4 có đáp án - VipLam.Net
Đề Tiếng Anh 12 cơ bản unit 4 có đáp án - VipLam.NetĐề Tiếng Anh 12 cơ bản unit 4 có đáp án - VipLam.Net
Đề Tiếng Anh 12 cơ bản unit 4 có đáp án - VipLam.Net
 
Dịch nghĩa và giải thích chi tiết starter toeic unit 3
Dịch nghĩa và giải thích chi tiết starter toeic unit 3Dịch nghĩa và giải thích chi tiết starter toeic unit 3
Dịch nghĩa và giải thích chi tiết starter toeic unit 3
 
đáP án và giải thích đề 4
đáP án và giải thích đề 4đáP án và giải thích đề 4
đáP án và giải thích đề 4
 
Starter Toeic
Starter ToeicStarter Toeic
Starter Toeic
 
Dịch nghĩa và giải thích chi tiết starter toeic unit 2
Dịch nghĩa và giải thích chi tiết starter toeic unit 2Dịch nghĩa và giải thích chi tiết starter toeic unit 2
Dịch nghĩa và giải thích chi tiết starter toeic unit 2
 
Trắc nghiệm từng unit tiếng Anh 12 (16 unit) + key
Trắc nghiệm từng unit tiếng Anh 12 (16 unit) + keyTrắc nghiệm từng unit tiếng Anh 12 (16 unit) + key
Trắc nghiệm từng unit tiếng Anh 12 (16 unit) + key
 
40 ĐỀ THI THỬ TỐT NGHIỆP THPT - MÔN TIẾNG ANH - NĂM 2023 - SOẠN CHUẨN CẤU TR...
40 ĐỀ THI THỬ TỐT NGHIỆP THPT - MÔN TIẾNG ANH - NĂM 2023 - SOẠN CHUẨN CẤU TR...40 ĐỀ THI THỬ TỐT NGHIỆP THPT - MÔN TIẾNG ANH - NĂM 2023 - SOẠN CHUẨN CẤU TR...
40 ĐỀ THI THỬ TỐT NGHIỆP THPT - MÔN TIẾNG ANH - NĂM 2023 - SOẠN CHUẨN CẤU TR...
 
Đề Tiếng Anh 12 cơ bản unit 15 có đáp án - VipLam.Net
Đề Tiếng Anh 12 cơ bản unit 15 có đáp án - VipLam.NetĐề Tiếng Anh 12 cơ bản unit 15 có đáp án - VipLam.Net
Đề Tiếng Anh 12 cơ bản unit 15 có đáp án - VipLam.Net
 
BÀI TẬP BỔ TRỢ TIẾNG ANH LỚP 10 GLOBAL SUCCESS NĂM 2023 CÓ FILE NGHE - 2 HỌC ...
BÀI TẬP BỔ TRỢ TIẾNG ANH LỚP 10 GLOBAL SUCCESS NĂM 2023 CÓ FILE NGHE - 2 HỌC ...BÀI TẬP BỔ TRỢ TIẾNG ANH LỚP 10 GLOBAL SUCCESS NĂM 2023 CÓ FILE NGHE - 2 HỌC ...
BÀI TẬP BỔ TRỢ TIẾNG ANH LỚP 10 GLOBAL SUCCESS NĂM 2023 CÓ FILE NGHE - 2 HỌC ...
 
Xstk de thi mau 01 (may12)
Xstk de thi mau 01 (may12)Xstk de thi mau 01 (may12)
Xstk de thi mau 01 (may12)
 
Bài tập trắc nghiệm xác suất thống kê
Bài tập trắc nghiệm xác suất thống kêBài tập trắc nghiệm xác suất thống kê
Bài tập trắc nghiệm xác suất thống kê
 
De tieng-anh-part5-toeic-co-dap-an-loi-giai-chi-tiet
De tieng-anh-part5-toeic-co-dap-an-loi-giai-chi-tietDe tieng-anh-part5-toeic-co-dap-an-loi-giai-chi-tiet
De tieng-anh-part5-toeic-co-dap-an-loi-giai-chi-tiet
 
50 ĐỀ PHÁT TRIỂN THEO CẤU TRÚC ĐỀ MINH HỌA BGD NGÀY 22-3-2024 KỲ THI TỐT NGHI...
50 ĐỀ PHÁT TRIỂN THEO CẤU TRÚC ĐỀ MINH HỌA BGD NGÀY 22-3-2024 KỲ THI TỐT NGHI...50 ĐỀ PHÁT TRIỂN THEO CẤU TRÚC ĐỀ MINH HỌA BGD NGÀY 22-3-2024 KỲ THI TỐT NGHI...
50 ĐỀ PHÁT TRIỂN THEO CẤU TRÚC ĐỀ MINH HỌA BGD NGÀY 22-3-2024 KỲ THI TỐT NGHI...
 
Đề Tiếng Anh 12 cơ bản unit 10 có đáp án - VipLam.Net
Đề Tiếng Anh 12 cơ bản unit 10 có đáp án - VipLam.NetĐề Tiếng Anh 12 cơ bản unit 10 có đáp án - VipLam.Net
Đề Tiếng Anh 12 cơ bản unit 10 có đáp án - VipLam.Net
 
ĐỀ CƯƠNG + TEST ÔN TẬP GIỮA KÌ 2 TIẾNG ANH 11 - GLOBAL SUCCESS (THEO CHUẨN MI...
ĐỀ CƯƠNG + TEST ÔN TẬP GIỮA KÌ 2 TIẾNG ANH 11 - GLOBAL SUCCESS (THEO CHUẨN MI...ĐỀ CƯƠNG + TEST ÔN TẬP GIỮA KÌ 2 TIẾNG ANH 11 - GLOBAL SUCCESS (THEO CHUẨN MI...
ĐỀ CƯƠNG + TEST ÔN TẬP GIỮA KÌ 2 TIẾNG ANH 11 - GLOBAL SUCCESS (THEO CHUẨN MI...
 
đồ Thị hàm số chứa giá trị tuyệt đối
đồ Thị hàm số chứa giá trị tuyệt đốiđồ Thị hàm số chứa giá trị tuyệt đối
đồ Thị hàm số chứa giá trị tuyệt đối
 
Giải chi tiết đề tiếng anh THPTQG 2018 mã 403
Giải chi tiết đề tiếng anh THPTQG 2018 mã 403Giải chi tiết đề tiếng anh THPTQG 2018 mã 403
Giải chi tiết đề tiếng anh THPTQG 2018 mã 403
 
Bai giai chi tiet mon anh (chuyen) tuyen sinh lop 10 tphcm 2013
Bai giai chi tiet mon anh (chuyen) tuyen sinh lop 10 tphcm   2013Bai giai chi tiet mon anh (chuyen) tuyen sinh lop 10 tphcm   2013
Bai giai chi tiet mon anh (chuyen) tuyen sinh lop 10 tphcm 2013
 

Semelhante a DE PHAT TRIEN THEO CAU TRUC DE MINH HOA 2022 MON TIENG ANH DE 1 5.pdf

đáP án và giải thích đề 19
đáP án và giải thích đề 19đáP án và giải thích đề 19
đáP án và giải thích đề 19Huynh ICT
 
đáP án và giải thích đề 8
đáP án và giải thích đề 8đáP án và giải thích đề 8
đáP án và giải thích đề 8Huynh ICT
 
CD21 Exercise 2.1 KEY.docx tieng anh cho
CD21 Exercise 2.1 KEY.docx tieng anh choCD21 Exercise 2.1 KEY.docx tieng anh cho
CD21 Exercise 2.1 KEY.docx tieng anh chonamc250
 
Tuyển tập bộ đề thi thử mới nhất 2011 key
Tuyển tập bộ đề thi thử mới nhất 2011   keyTuyển tập bộ đề thi thử mới nhất 2011   key
Tuyển tập bộ đề thi thử mới nhất 2011 keyHùng Trần Mạnh
 
đáP án và giải thích đề 20
đáP án và giải thích đề 20đáP án và giải thích đề 20
đáP án và giải thích đề 20Huynh ICT
 
Giai chi tiet de thi dh khoi d t anh 2011
Giai chi tiet de thi dh khoi d t anh 2011Giai chi tiet de thi dh khoi d t anh 2011
Giai chi tiet de thi dh khoi d t anh 2011Hua Tran Phuong Thao
 
đáP án và giải thích đề 24
đáP án và giải thích đề 24đáP án và giải thích đề 24
đáP án và giải thích đề 24Huynh ICT
 
đáP án và giải thích đề 23
đáP án và giải thích đề 23đáP án và giải thích đề 23
đáP án và giải thích đề 23Huynh ICT
 
đáP án và giải thích đề 14
đáP án và giải thích đề 14đáP án và giải thích đề 14
đáP án và giải thích đề 14Huynh ICT
 
Đề thi thử Ôn thi Tiếng Anh vào Cao Đẳng , Đại học năm 2013 - Đề 110
Đề thi thử  Ôn thi Tiếng Anh vào Cao Đẳng , Đại học năm 2013 - Đề 110Đề thi thử  Ôn thi Tiếng Anh vào Cao Đẳng , Đại học năm 2013 - Đề 110
Đề thi thử Ôn thi Tiếng Anh vào Cao Đẳng , Đại học năm 2013 - Đề 110phamnhakb
 
đáP án và giải thích đề 10
đáP án và giải thích đề 10đáP án và giải thích đề 10
đáP án và giải thích đề 10Huynh ICT
 
đáP án và giải thích đề 16
đáP án và giải thích đề 16đáP án và giải thích đề 16
đáP án và giải thích đề 16Huynh ICT
 
đáP án và giải thích đề 25
đáP án và giải thích đề 25đáP án và giải thích đề 25
đáP án và giải thích đề 25Huynh ICT
 
đáP án và giải thích đề 32
đáP án và giải thích đề 32đáP án và giải thích đề 32
đáP án và giải thích đề 32Huynh ICT
 
đáP án và giải thích đề 31
đáP án và giải thích đề 31đáP án và giải thích đề 31
đáP án và giải thích đề 31Huynh ICT
 
đáP án và giải thích đề 17
đáP án và giải thích đề 17đáP án và giải thích đề 17
đáP án và giải thích đề 17Huynh ICT
 
đáP án và giải thích đề 15
đáP án và giải thích đề 15đáP án và giải thích đề 15
đáP án và giải thích đề 15Huynh ICT
 
đáP án và giải thích đề 6
đáP án và giải thích đề 6đáP án và giải thích đề 6
đáP án và giải thích đề 6Huynh ICT
 
đáP án và giải thích đề 11
đáP án và giải thích đề 11đáP án và giải thích đề 11
đáP án và giải thích đề 11Huynh ICT
 
đáP án và giải thích đề 5
đáP án và giải thích đề 5đáP án và giải thích đề 5
đáP án và giải thích đề 5Huynh ICT
 

Semelhante a DE PHAT TRIEN THEO CAU TRUC DE MINH HOA 2022 MON TIENG ANH DE 1 5.pdf (20)

đáP án và giải thích đề 19
đáP án và giải thích đề 19đáP án và giải thích đề 19
đáP án và giải thích đề 19
 
đáP án và giải thích đề 8
đáP án và giải thích đề 8đáP án và giải thích đề 8
đáP án và giải thích đề 8
 
CD21 Exercise 2.1 KEY.docx tieng anh cho
CD21 Exercise 2.1 KEY.docx tieng anh choCD21 Exercise 2.1 KEY.docx tieng anh cho
CD21 Exercise 2.1 KEY.docx tieng anh cho
 
Tuyển tập bộ đề thi thử mới nhất 2011 key
Tuyển tập bộ đề thi thử mới nhất 2011   keyTuyển tập bộ đề thi thử mới nhất 2011   key
Tuyển tập bộ đề thi thử mới nhất 2011 key
 
đáP án và giải thích đề 20
đáP án và giải thích đề 20đáP án và giải thích đề 20
đáP án và giải thích đề 20
 
Giai chi tiet de thi dh khoi d t anh 2011
Giai chi tiet de thi dh khoi d t anh 2011Giai chi tiet de thi dh khoi d t anh 2011
Giai chi tiet de thi dh khoi d t anh 2011
 
đáP án và giải thích đề 24
đáP án và giải thích đề 24đáP án và giải thích đề 24
đáP án và giải thích đề 24
 
đáP án và giải thích đề 23
đáP án và giải thích đề 23đáP án và giải thích đề 23
đáP án và giải thích đề 23
 
đáP án và giải thích đề 14
đáP án và giải thích đề 14đáP án và giải thích đề 14
đáP án và giải thích đề 14
 
Đề thi thử Ôn thi Tiếng Anh vào Cao Đẳng , Đại học năm 2013 - Đề 110
Đề thi thử  Ôn thi Tiếng Anh vào Cao Đẳng , Đại học năm 2013 - Đề 110Đề thi thử  Ôn thi Tiếng Anh vào Cao Đẳng , Đại học năm 2013 - Đề 110
Đề thi thử Ôn thi Tiếng Anh vào Cao Đẳng , Đại học năm 2013 - Đề 110
 
đáP án và giải thích đề 10
đáP án và giải thích đề 10đáP án và giải thích đề 10
đáP án và giải thích đề 10
 
đáP án và giải thích đề 16
đáP án và giải thích đề 16đáP án và giải thích đề 16
đáP án và giải thích đề 16
 
đáP án và giải thích đề 25
đáP án và giải thích đề 25đáP án và giải thích đề 25
đáP án và giải thích đề 25
 
đáP án và giải thích đề 32
đáP án và giải thích đề 32đáP án và giải thích đề 32
đáP án và giải thích đề 32
 
đáP án và giải thích đề 31
đáP án và giải thích đề 31đáP án và giải thích đề 31
đáP án và giải thích đề 31
 
đáP án và giải thích đề 17
đáP án và giải thích đề 17đáP án và giải thích đề 17
đáP án và giải thích đề 17
 
đáP án và giải thích đề 15
đáP án và giải thích đề 15đáP án và giải thích đề 15
đáP án và giải thích đề 15
 
đáP án và giải thích đề 6
đáP án và giải thích đề 6đáP án và giải thích đề 6
đáP án và giải thích đề 6
 
đáP án và giải thích đề 11
đáP án và giải thích đề 11đáP án và giải thích đề 11
đáP án và giải thích đề 11
 
đáP án và giải thích đề 5
đáP án và giải thích đề 5đáP án và giải thích đề 5
đáP án và giải thích đề 5
 

Mais de Nguyen Thanh Tu Collection

TÀI LIỆU BỒI DƯỠNG HỌC SINH GIỎI KỸ NĂNG VIẾT ĐOẠN VĂN NGHỊ LUẬN XÃ HỘI 200 C...
TÀI LIỆU BỒI DƯỠNG HỌC SINH GIỎI KỸ NĂNG VIẾT ĐOẠN VĂN NGHỊ LUẬN XÃ HỘI 200 C...TÀI LIỆU BỒI DƯỠNG HỌC SINH GIỎI KỸ NĂNG VIẾT ĐOẠN VĂN NGHỊ LUẬN XÃ HỘI 200 C...
TÀI LIỆU BỒI DƯỠNG HỌC SINH GIỎI KỸ NĂNG VIẾT ĐOẠN VĂN NGHỊ LUẬN XÃ HỘI 200 C...Nguyen Thanh Tu Collection
 
BỘ LUYỆN NGHE VÀO 10 TIẾNG ANH DẠNG TRẮC NGHIỆM 4 CÂU TRẢ LỜI - CÓ FILE NGHE.pdf
BỘ LUYỆN NGHE VÀO 10 TIẾNG ANH DẠNG TRẮC NGHIỆM 4 CÂU TRẢ LỜI - CÓ FILE NGHE.pdfBỘ LUYỆN NGHE VÀO 10 TIẾNG ANH DẠNG TRẮC NGHIỆM 4 CÂU TRẢ LỜI - CÓ FILE NGHE.pdf
BỘ LUYỆN NGHE VÀO 10 TIẾNG ANH DẠNG TRẮC NGHIỆM 4 CÂU TRẢ LỜI - CÓ FILE NGHE.pdfNguyen Thanh Tu Collection
 
ĐỀ CHÍNH THỨC KỲ THI TUYỂN SINH VÀO LỚP 10 THPT CÁC TỈNH THÀNH NĂM HỌC 2020 –...
ĐỀ CHÍNH THỨC KỲ THI TUYỂN SINH VÀO LỚP 10 THPT CÁC TỈNH THÀNH NĂM HỌC 2020 –...ĐỀ CHÍNH THỨC KỲ THI TUYỂN SINH VÀO LỚP 10 THPT CÁC TỈNH THÀNH NĂM HỌC 2020 –...
ĐỀ CHÍNH THỨC KỲ THI TUYỂN SINH VÀO LỚP 10 THPT CÁC TỈNH THÀNH NĂM HỌC 2020 –...Nguyen Thanh Tu Collection
 
TÀI LIỆU BỒI DƯỠNG HỌC SINH GIỎI LÝ LUẬN VĂN HỌC NĂM HỌC 2023-2024 - MÔN NGỮ ...
TÀI LIỆU BỒI DƯỠNG HỌC SINH GIỎI LÝ LUẬN VĂN HỌC NĂM HỌC 2023-2024 - MÔN NGỮ ...TÀI LIỆU BỒI DƯỠNG HỌC SINH GIỎI LÝ LUẬN VĂN HỌC NĂM HỌC 2023-2024 - MÔN NGỮ ...
TÀI LIỆU BỒI DƯỠNG HỌC SINH GIỎI LÝ LUẬN VĂN HỌC NĂM HỌC 2023-2024 - MÔN NGỮ ...Nguyen Thanh Tu Collection
 
30 ĐỀ PHÁT TRIỂN THEO CẤU TRÚC ĐỀ MINH HỌA BGD NGÀY 22-3-2024 KỲ THI TỐT NGHI...
30 ĐỀ PHÁT TRIỂN THEO CẤU TRÚC ĐỀ MINH HỌA BGD NGÀY 22-3-2024 KỲ THI TỐT NGHI...30 ĐỀ PHÁT TRIỂN THEO CẤU TRÚC ĐỀ MINH HỌA BGD NGÀY 22-3-2024 KỲ THI TỐT NGHI...
30 ĐỀ PHÁT TRIỂN THEO CẤU TRÚC ĐỀ MINH HỌA BGD NGÀY 22-3-2024 KỲ THI TỐT NGHI...Nguyen Thanh Tu Collection
 
GIÁO ÁN DẠY THÊM (KẾ HOẠCH BÀI DẠY BUỔI 2) - TIẾNG ANH 7 GLOBAL SUCCESS (2 CỘ...
GIÁO ÁN DẠY THÊM (KẾ HOẠCH BÀI DẠY BUỔI 2) - TIẾNG ANH 7 GLOBAL SUCCESS (2 CỘ...GIÁO ÁN DẠY THÊM (KẾ HOẠCH BÀI DẠY BUỔI 2) - TIẾNG ANH 7 GLOBAL SUCCESS (2 CỘ...
GIÁO ÁN DẠY THÊM (KẾ HOẠCH BÀI DẠY BUỔI 2) - TIẾNG ANH 7 GLOBAL SUCCESS (2 CỘ...Nguyen Thanh Tu Collection
 
SÁNG KIẾN ÁP DỤNG CLT (COMMUNICATIVE LANGUAGE TEACHING) VÀO QUÁ TRÌNH DẠY - H...
SÁNG KIẾN ÁP DỤNG CLT (COMMUNICATIVE LANGUAGE TEACHING) VÀO QUÁ TRÌNH DẠY - H...SÁNG KIẾN ÁP DỤNG CLT (COMMUNICATIVE LANGUAGE TEACHING) VÀO QUÁ TRÌNH DẠY - H...
SÁNG KIẾN ÁP DỤNG CLT (COMMUNICATIVE LANGUAGE TEACHING) VÀO QUÁ TRÌNH DẠY - H...Nguyen Thanh Tu Collection
 
30 ĐỀ PHÁT TRIỂN THEO CẤU TRÚC ĐỀ MINH HỌA BGD NGÀY 22-3-2024 KỲ THI TỐT NGHI...
30 ĐỀ PHÁT TRIỂN THEO CẤU TRÚC ĐỀ MINH HỌA BGD NGÀY 22-3-2024 KỲ THI TỐT NGHI...30 ĐỀ PHÁT TRIỂN THEO CẤU TRÚC ĐỀ MINH HỌA BGD NGÀY 22-3-2024 KỲ THI TỐT NGHI...
30 ĐỀ PHÁT TRIỂN THEO CẤU TRÚC ĐỀ MINH HỌA BGD NGÀY 22-3-2024 KỲ THI TỐT NGHI...Nguyen Thanh Tu Collection
 
30 ĐỀ PHÁT TRIỂN THEO CẤU TRÚC ĐỀ MINH HỌA BGD NGÀY 22-3-2024 KỲ THI TỐT NGHI...
30 ĐỀ PHÁT TRIỂN THEO CẤU TRÚC ĐỀ MINH HỌA BGD NGÀY 22-3-2024 KỲ THI TỐT NGHI...30 ĐỀ PHÁT TRIỂN THEO CẤU TRÚC ĐỀ MINH HỌA BGD NGÀY 22-3-2024 KỲ THI TỐT NGHI...
30 ĐỀ PHÁT TRIỂN THEO CẤU TRÚC ĐỀ MINH HỌA BGD NGÀY 22-3-2024 KỲ THI TỐT NGHI...Nguyen Thanh Tu Collection
 
30 ĐỀ PHÁT TRIỂN THEO CẤU TRÚC ĐỀ MINH HỌA BGD NGÀY 22-3-2024 KỲ THI TỐT NGHI...
30 ĐỀ PHÁT TRIỂN THEO CẤU TRÚC ĐỀ MINH HỌA BGD NGÀY 22-3-2024 KỲ THI TỐT NGHI...30 ĐỀ PHÁT TRIỂN THEO CẤU TRÚC ĐỀ MINH HỌA BGD NGÀY 22-3-2024 KỲ THI TỐT NGHI...
30 ĐỀ PHÁT TRIỂN THEO CẤU TRÚC ĐỀ MINH HỌA BGD NGÀY 22-3-2024 KỲ THI TỐT NGHI...Nguyen Thanh Tu Collection
 
30 ĐỀ PHÁT TRIỂN THEO CẤU TRÚC ĐỀ MINH HỌA BGD NGÀY 22-3-2024 KỲ THI TỐT NGHI...
30 ĐỀ PHÁT TRIỂN THEO CẤU TRÚC ĐỀ MINH HỌA BGD NGÀY 22-3-2024 KỲ THI TỐT NGHI...30 ĐỀ PHÁT TRIỂN THEO CẤU TRÚC ĐỀ MINH HỌA BGD NGÀY 22-3-2024 KỲ THI TỐT NGHI...
30 ĐỀ PHÁT TRIỂN THEO CẤU TRÚC ĐỀ MINH HỌA BGD NGÀY 22-3-2024 KỲ THI TỐT NGHI...Nguyen Thanh Tu Collection
 
TỔNG HỢP ĐỀ THI CHÍNH THỨC KỲ THI TUYỂN SINH VÀO LỚP 10 THPT MÔN NGỮ VĂN NĂM ...
TỔNG HỢP ĐỀ THI CHÍNH THỨC KỲ THI TUYỂN SINH VÀO LỚP 10 THPT MÔN NGỮ VĂN NĂM ...TỔNG HỢP ĐỀ THI CHÍNH THỨC KỲ THI TUYỂN SINH VÀO LỚP 10 THPT MÔN NGỮ VĂN NĂM ...
TỔNG HỢP ĐỀ THI CHÍNH THỨC KỲ THI TUYỂN SINH VÀO LỚP 10 THPT MÔN NGỮ VĂN NĂM ...Nguyen Thanh Tu Collection
 
TUYỂN TẬP 20 ĐỀ THI KHẢO SÁT HỌC SINH GIỎI MÔN TIẾNG ANH LỚP 6 NĂM 2020 (CÓ Đ...
TUYỂN TẬP 20 ĐỀ THI KHẢO SÁT HỌC SINH GIỎI MÔN TIẾNG ANH LỚP 6 NĂM 2020 (CÓ Đ...TUYỂN TẬP 20 ĐỀ THI KHẢO SÁT HỌC SINH GIỎI MÔN TIẾNG ANH LỚP 6 NĂM 2020 (CÓ Đ...
TUYỂN TẬP 20 ĐỀ THI KHẢO SÁT HỌC SINH GIỎI MÔN TIẾNG ANH LỚP 6 NĂM 2020 (CÓ Đ...Nguyen Thanh Tu Collection
 
TUYỂN TẬP 25 ĐỀ THI HỌC SINH GIỎI MÔN TIẾNG ANH LỚP 6 NĂM 2023 CÓ ĐÁP ÁN (SƯU...
TUYỂN TẬP 25 ĐỀ THI HỌC SINH GIỎI MÔN TIẾNG ANH LỚP 6 NĂM 2023 CÓ ĐÁP ÁN (SƯU...TUYỂN TẬP 25 ĐỀ THI HỌC SINH GIỎI MÔN TIẾNG ANH LỚP 6 NĂM 2023 CÓ ĐÁP ÁN (SƯU...
TUYỂN TẬP 25 ĐỀ THI HỌC SINH GIỎI MÔN TIẾNG ANH LỚP 6 NĂM 2023 CÓ ĐÁP ÁN (SƯU...Nguyen Thanh Tu Collection
 
BỘ ĐỀ PHÁT TRIỂN THEO CẤU TRÚC ĐỀ MINH HỌA BGD NGÀY 22-3-2024 KỲ THI TỐT NGHI...
BỘ ĐỀ PHÁT TRIỂN THEO CẤU TRÚC ĐỀ MINH HỌA BGD NGÀY 22-3-2024 KỲ THI TỐT NGHI...BỘ ĐỀ PHÁT TRIỂN THEO CẤU TRÚC ĐỀ MINH HỌA BGD NGÀY 22-3-2024 KỲ THI TỐT NGHI...
BỘ ĐỀ PHÁT TRIỂN THEO CẤU TRÚC ĐỀ MINH HỌA BGD NGÀY 22-3-2024 KỲ THI TỐT NGHI...Nguyen Thanh Tu Collection
 
Sáng kiến Dạy học theo định hướng STEM một số chủ đề phần “vật sống”, Khoa họ...
Sáng kiến Dạy học theo định hướng STEM một số chủ đề phần “vật sống”, Khoa họ...Sáng kiến Dạy học theo định hướng STEM một số chủ đề phần “vật sống”, Khoa họ...
Sáng kiến Dạy học theo định hướng STEM một số chủ đề phần “vật sống”, Khoa họ...Nguyen Thanh Tu Collection
 
Sáng kiến “Sử dụng ứng dụng Quizizz nhằm nâng cao chất lượng ôn thi tốt nghiệ...
Sáng kiến “Sử dụng ứng dụng Quizizz nhằm nâng cao chất lượng ôn thi tốt nghiệ...Sáng kiến “Sử dụng ứng dụng Quizizz nhằm nâng cao chất lượng ôn thi tốt nghiệ...
Sáng kiến “Sử dụng ứng dụng Quizizz nhằm nâng cao chất lượng ôn thi tốt nghiệ...Nguyen Thanh Tu Collection
 
30 ĐỀ PHÁT TRIỂN THEO CẤU TRÚC ĐỀ MINH HỌA BGD NGÀY 22-3-2024 KỲ THI TỐT NGHI...
30 ĐỀ PHÁT TRIỂN THEO CẤU TRÚC ĐỀ MINH HỌA BGD NGÀY 22-3-2024 KỲ THI TỐT NGHI...30 ĐỀ PHÁT TRIỂN THEO CẤU TRÚC ĐỀ MINH HỌA BGD NGÀY 22-3-2024 KỲ THI TỐT NGHI...
30 ĐỀ PHÁT TRIỂN THEO CẤU TRÚC ĐỀ MINH HỌA BGD NGÀY 22-3-2024 KỲ THI TỐT NGHI...Nguyen Thanh Tu Collection
 
10 ĐỀ KIỂM TRA + 6 ĐỀ ÔN TẬP CUỐI KÌ 2 VẬT LÝ 11 - KẾT NỐI TRI THỨC - THEO C...
10 ĐỀ KIỂM TRA + 6 ĐỀ ÔN TẬP CUỐI KÌ 2 VẬT LÝ 11 - KẾT NỐI TRI THỨC - THEO C...10 ĐỀ KIỂM TRA + 6 ĐỀ ÔN TẬP CUỐI KÌ 2 VẬT LÝ 11 - KẾT NỐI TRI THỨC - THEO C...
10 ĐỀ KIỂM TRA + 6 ĐỀ ÔN TẬP CUỐI KÌ 2 VẬT LÝ 11 - KẾT NỐI TRI THỨC - THEO C...Nguyen Thanh Tu Collection
 
BỘ ĐỀ KIỂM TRA CUỐI KÌ 2 VẬT LÝ 11 - KẾT NỐI TRI THỨC - THEO CẤU TRÚC ĐỀ MIN...
BỘ ĐỀ KIỂM TRA CUỐI KÌ 2 VẬT LÝ 11 - KẾT NỐI TRI THỨC - THEO CẤU TRÚC ĐỀ MIN...BỘ ĐỀ KIỂM TRA CUỐI KÌ 2 VẬT LÝ 11 - KẾT NỐI TRI THỨC - THEO CẤU TRÚC ĐỀ MIN...
BỘ ĐỀ KIỂM TRA CUỐI KÌ 2 VẬT LÝ 11 - KẾT NỐI TRI THỨC - THEO CẤU TRÚC ĐỀ MIN...Nguyen Thanh Tu Collection
 

Mais de Nguyen Thanh Tu Collection (20)

TÀI LIỆU BỒI DƯỠNG HỌC SINH GIỎI KỸ NĂNG VIẾT ĐOẠN VĂN NGHỊ LUẬN XÃ HỘI 200 C...
TÀI LIỆU BỒI DƯỠNG HỌC SINH GIỎI KỸ NĂNG VIẾT ĐOẠN VĂN NGHỊ LUẬN XÃ HỘI 200 C...TÀI LIỆU BỒI DƯỠNG HỌC SINH GIỎI KỸ NĂNG VIẾT ĐOẠN VĂN NGHỊ LUẬN XÃ HỘI 200 C...
TÀI LIỆU BỒI DƯỠNG HỌC SINH GIỎI KỸ NĂNG VIẾT ĐOẠN VĂN NGHỊ LUẬN XÃ HỘI 200 C...
 
BỘ LUYỆN NGHE VÀO 10 TIẾNG ANH DẠNG TRẮC NGHIỆM 4 CÂU TRẢ LỜI - CÓ FILE NGHE.pdf
BỘ LUYỆN NGHE VÀO 10 TIẾNG ANH DẠNG TRẮC NGHIỆM 4 CÂU TRẢ LỜI - CÓ FILE NGHE.pdfBỘ LUYỆN NGHE VÀO 10 TIẾNG ANH DẠNG TRẮC NGHIỆM 4 CÂU TRẢ LỜI - CÓ FILE NGHE.pdf
BỘ LUYỆN NGHE VÀO 10 TIẾNG ANH DẠNG TRẮC NGHIỆM 4 CÂU TRẢ LỜI - CÓ FILE NGHE.pdf
 
ĐỀ CHÍNH THỨC KỲ THI TUYỂN SINH VÀO LỚP 10 THPT CÁC TỈNH THÀNH NĂM HỌC 2020 –...
ĐỀ CHÍNH THỨC KỲ THI TUYỂN SINH VÀO LỚP 10 THPT CÁC TỈNH THÀNH NĂM HỌC 2020 –...ĐỀ CHÍNH THỨC KỲ THI TUYỂN SINH VÀO LỚP 10 THPT CÁC TỈNH THÀNH NĂM HỌC 2020 –...
ĐỀ CHÍNH THỨC KỲ THI TUYỂN SINH VÀO LỚP 10 THPT CÁC TỈNH THÀNH NĂM HỌC 2020 –...
 
TÀI LIỆU BỒI DƯỠNG HỌC SINH GIỎI LÝ LUẬN VĂN HỌC NĂM HỌC 2023-2024 - MÔN NGỮ ...
TÀI LIỆU BỒI DƯỠNG HỌC SINH GIỎI LÝ LUẬN VĂN HỌC NĂM HỌC 2023-2024 - MÔN NGỮ ...TÀI LIỆU BỒI DƯỠNG HỌC SINH GIỎI LÝ LUẬN VĂN HỌC NĂM HỌC 2023-2024 - MÔN NGỮ ...
TÀI LIỆU BỒI DƯỠNG HỌC SINH GIỎI LÝ LUẬN VĂN HỌC NĂM HỌC 2023-2024 - MÔN NGỮ ...
 
30 ĐỀ PHÁT TRIỂN THEO CẤU TRÚC ĐỀ MINH HỌA BGD NGÀY 22-3-2024 KỲ THI TỐT NGHI...
30 ĐỀ PHÁT TRIỂN THEO CẤU TRÚC ĐỀ MINH HỌA BGD NGÀY 22-3-2024 KỲ THI TỐT NGHI...30 ĐỀ PHÁT TRIỂN THEO CẤU TRÚC ĐỀ MINH HỌA BGD NGÀY 22-3-2024 KỲ THI TỐT NGHI...
30 ĐỀ PHÁT TRIỂN THEO CẤU TRÚC ĐỀ MINH HỌA BGD NGÀY 22-3-2024 KỲ THI TỐT NGHI...
 
GIÁO ÁN DẠY THÊM (KẾ HOẠCH BÀI DẠY BUỔI 2) - TIẾNG ANH 7 GLOBAL SUCCESS (2 CỘ...
GIÁO ÁN DẠY THÊM (KẾ HOẠCH BÀI DẠY BUỔI 2) - TIẾNG ANH 7 GLOBAL SUCCESS (2 CỘ...GIÁO ÁN DẠY THÊM (KẾ HOẠCH BÀI DẠY BUỔI 2) - TIẾNG ANH 7 GLOBAL SUCCESS (2 CỘ...
GIÁO ÁN DẠY THÊM (KẾ HOẠCH BÀI DẠY BUỔI 2) - TIẾNG ANH 7 GLOBAL SUCCESS (2 CỘ...
 
SÁNG KIẾN ÁP DỤNG CLT (COMMUNICATIVE LANGUAGE TEACHING) VÀO QUÁ TRÌNH DẠY - H...
SÁNG KIẾN ÁP DỤNG CLT (COMMUNICATIVE LANGUAGE TEACHING) VÀO QUÁ TRÌNH DẠY - H...SÁNG KIẾN ÁP DỤNG CLT (COMMUNICATIVE LANGUAGE TEACHING) VÀO QUÁ TRÌNH DẠY - H...
SÁNG KIẾN ÁP DỤNG CLT (COMMUNICATIVE LANGUAGE TEACHING) VÀO QUÁ TRÌNH DẠY - H...
 
30 ĐỀ PHÁT TRIỂN THEO CẤU TRÚC ĐỀ MINH HỌA BGD NGÀY 22-3-2024 KỲ THI TỐT NGHI...
30 ĐỀ PHÁT TRIỂN THEO CẤU TRÚC ĐỀ MINH HỌA BGD NGÀY 22-3-2024 KỲ THI TỐT NGHI...30 ĐỀ PHÁT TRIỂN THEO CẤU TRÚC ĐỀ MINH HỌA BGD NGÀY 22-3-2024 KỲ THI TỐT NGHI...
30 ĐỀ PHÁT TRIỂN THEO CẤU TRÚC ĐỀ MINH HỌA BGD NGÀY 22-3-2024 KỲ THI TỐT NGHI...
 
30 ĐỀ PHÁT TRIỂN THEO CẤU TRÚC ĐỀ MINH HỌA BGD NGÀY 22-3-2024 KỲ THI TỐT NGHI...
30 ĐỀ PHÁT TRIỂN THEO CẤU TRÚC ĐỀ MINH HỌA BGD NGÀY 22-3-2024 KỲ THI TỐT NGHI...30 ĐỀ PHÁT TRIỂN THEO CẤU TRÚC ĐỀ MINH HỌA BGD NGÀY 22-3-2024 KỲ THI TỐT NGHI...
30 ĐỀ PHÁT TRIỂN THEO CẤU TRÚC ĐỀ MINH HỌA BGD NGÀY 22-3-2024 KỲ THI TỐT NGHI...
 
30 ĐỀ PHÁT TRIỂN THEO CẤU TRÚC ĐỀ MINH HỌA BGD NGÀY 22-3-2024 KỲ THI TỐT NGHI...
30 ĐỀ PHÁT TRIỂN THEO CẤU TRÚC ĐỀ MINH HỌA BGD NGÀY 22-3-2024 KỲ THI TỐT NGHI...30 ĐỀ PHÁT TRIỂN THEO CẤU TRÚC ĐỀ MINH HỌA BGD NGÀY 22-3-2024 KỲ THI TỐT NGHI...
30 ĐỀ PHÁT TRIỂN THEO CẤU TRÚC ĐỀ MINH HỌA BGD NGÀY 22-3-2024 KỲ THI TỐT NGHI...
 
30 ĐỀ PHÁT TRIỂN THEO CẤU TRÚC ĐỀ MINH HỌA BGD NGÀY 22-3-2024 KỲ THI TỐT NGHI...
30 ĐỀ PHÁT TRIỂN THEO CẤU TRÚC ĐỀ MINH HỌA BGD NGÀY 22-3-2024 KỲ THI TỐT NGHI...30 ĐỀ PHÁT TRIỂN THEO CẤU TRÚC ĐỀ MINH HỌA BGD NGÀY 22-3-2024 KỲ THI TỐT NGHI...
30 ĐỀ PHÁT TRIỂN THEO CẤU TRÚC ĐỀ MINH HỌA BGD NGÀY 22-3-2024 KỲ THI TỐT NGHI...
 
TỔNG HỢP ĐỀ THI CHÍNH THỨC KỲ THI TUYỂN SINH VÀO LỚP 10 THPT MÔN NGỮ VĂN NĂM ...
TỔNG HỢP ĐỀ THI CHÍNH THỨC KỲ THI TUYỂN SINH VÀO LỚP 10 THPT MÔN NGỮ VĂN NĂM ...TỔNG HỢP ĐỀ THI CHÍNH THỨC KỲ THI TUYỂN SINH VÀO LỚP 10 THPT MÔN NGỮ VĂN NĂM ...
TỔNG HỢP ĐỀ THI CHÍNH THỨC KỲ THI TUYỂN SINH VÀO LỚP 10 THPT MÔN NGỮ VĂN NĂM ...
 
TUYỂN TẬP 20 ĐỀ THI KHẢO SÁT HỌC SINH GIỎI MÔN TIẾNG ANH LỚP 6 NĂM 2020 (CÓ Đ...
TUYỂN TẬP 20 ĐỀ THI KHẢO SÁT HỌC SINH GIỎI MÔN TIẾNG ANH LỚP 6 NĂM 2020 (CÓ Đ...TUYỂN TẬP 20 ĐỀ THI KHẢO SÁT HỌC SINH GIỎI MÔN TIẾNG ANH LỚP 6 NĂM 2020 (CÓ Đ...
TUYỂN TẬP 20 ĐỀ THI KHẢO SÁT HỌC SINH GIỎI MÔN TIẾNG ANH LỚP 6 NĂM 2020 (CÓ Đ...
 
TUYỂN TẬP 25 ĐỀ THI HỌC SINH GIỎI MÔN TIẾNG ANH LỚP 6 NĂM 2023 CÓ ĐÁP ÁN (SƯU...
TUYỂN TẬP 25 ĐỀ THI HỌC SINH GIỎI MÔN TIẾNG ANH LỚP 6 NĂM 2023 CÓ ĐÁP ÁN (SƯU...TUYỂN TẬP 25 ĐỀ THI HỌC SINH GIỎI MÔN TIẾNG ANH LỚP 6 NĂM 2023 CÓ ĐÁP ÁN (SƯU...
TUYỂN TẬP 25 ĐỀ THI HỌC SINH GIỎI MÔN TIẾNG ANH LỚP 6 NĂM 2023 CÓ ĐÁP ÁN (SƯU...
 
BỘ ĐỀ PHÁT TRIỂN THEO CẤU TRÚC ĐỀ MINH HỌA BGD NGÀY 22-3-2024 KỲ THI TỐT NGHI...
BỘ ĐỀ PHÁT TRIỂN THEO CẤU TRÚC ĐỀ MINH HỌA BGD NGÀY 22-3-2024 KỲ THI TỐT NGHI...BỘ ĐỀ PHÁT TRIỂN THEO CẤU TRÚC ĐỀ MINH HỌA BGD NGÀY 22-3-2024 KỲ THI TỐT NGHI...
BỘ ĐỀ PHÁT TRIỂN THEO CẤU TRÚC ĐỀ MINH HỌA BGD NGÀY 22-3-2024 KỲ THI TỐT NGHI...
 
Sáng kiến Dạy học theo định hướng STEM một số chủ đề phần “vật sống”, Khoa họ...
Sáng kiến Dạy học theo định hướng STEM một số chủ đề phần “vật sống”, Khoa họ...Sáng kiến Dạy học theo định hướng STEM một số chủ đề phần “vật sống”, Khoa họ...
Sáng kiến Dạy học theo định hướng STEM một số chủ đề phần “vật sống”, Khoa họ...
 
Sáng kiến “Sử dụng ứng dụng Quizizz nhằm nâng cao chất lượng ôn thi tốt nghiệ...
Sáng kiến “Sử dụng ứng dụng Quizizz nhằm nâng cao chất lượng ôn thi tốt nghiệ...Sáng kiến “Sử dụng ứng dụng Quizizz nhằm nâng cao chất lượng ôn thi tốt nghiệ...
Sáng kiến “Sử dụng ứng dụng Quizizz nhằm nâng cao chất lượng ôn thi tốt nghiệ...
 
30 ĐỀ PHÁT TRIỂN THEO CẤU TRÚC ĐỀ MINH HỌA BGD NGÀY 22-3-2024 KỲ THI TỐT NGHI...
30 ĐỀ PHÁT TRIỂN THEO CẤU TRÚC ĐỀ MINH HỌA BGD NGÀY 22-3-2024 KỲ THI TỐT NGHI...30 ĐỀ PHÁT TRIỂN THEO CẤU TRÚC ĐỀ MINH HỌA BGD NGÀY 22-3-2024 KỲ THI TỐT NGHI...
30 ĐỀ PHÁT TRIỂN THEO CẤU TRÚC ĐỀ MINH HỌA BGD NGÀY 22-3-2024 KỲ THI TỐT NGHI...
 
10 ĐỀ KIỂM TRA + 6 ĐỀ ÔN TẬP CUỐI KÌ 2 VẬT LÝ 11 - KẾT NỐI TRI THỨC - THEO C...
10 ĐỀ KIỂM TRA + 6 ĐỀ ÔN TẬP CUỐI KÌ 2 VẬT LÝ 11 - KẾT NỐI TRI THỨC - THEO C...10 ĐỀ KIỂM TRA + 6 ĐỀ ÔN TẬP CUỐI KÌ 2 VẬT LÝ 11 - KẾT NỐI TRI THỨC - THEO C...
10 ĐỀ KIỂM TRA + 6 ĐỀ ÔN TẬP CUỐI KÌ 2 VẬT LÝ 11 - KẾT NỐI TRI THỨC - THEO C...
 
BỘ ĐỀ KIỂM TRA CUỐI KÌ 2 VẬT LÝ 11 - KẾT NỐI TRI THỨC - THEO CẤU TRÚC ĐỀ MIN...
BỘ ĐỀ KIỂM TRA CUỐI KÌ 2 VẬT LÝ 11 - KẾT NỐI TRI THỨC - THEO CẤU TRÚC ĐỀ MIN...BỘ ĐỀ KIỂM TRA CUỐI KÌ 2 VẬT LÝ 11 - KẾT NỐI TRI THỨC - THEO CẤU TRÚC ĐỀ MIN...
BỘ ĐỀ KIỂM TRA CUỐI KÌ 2 VẬT LÝ 11 - KẾT NỐI TRI THỨC - THEO CẤU TRÚC ĐỀ MIN...
 

Último

Đề cương môn giải phẫu......................
Đề cương môn giải phẫu......................Đề cương môn giải phẫu......................
Đề cương môn giải phẫu......................TrnHoa46
 
PHÁT TRIỂN DU LỊCH BỀN VỮNG Ở TUYÊN QUANG
PHÁT TRIỂN DU LỊCH BỀN VỮNG Ở TUYÊN QUANGPHÁT TRIỂN DU LỊCH BỀN VỮNG Ở TUYÊN QUANG
PHÁT TRIỂN DU LỊCH BỀN VỮNG Ở TUYÊN QUANGhoinnhgtctat
 
chuong-7-van-de-gia-dinh-trong-thoi-ky-qua-do-len-cnxh.pdf
chuong-7-van-de-gia-dinh-trong-thoi-ky-qua-do-len-cnxh.pdfchuong-7-van-de-gia-dinh-trong-thoi-ky-qua-do-len-cnxh.pdf
chuong-7-van-de-gia-dinh-trong-thoi-ky-qua-do-len-cnxh.pdfVyTng986513
 
Campbell _2011_ - Sinh học - Tế bào - Ref.pdf
Campbell _2011_ - Sinh học - Tế bào - Ref.pdfCampbell _2011_ - Sinh học - Tế bào - Ref.pdf
Campbell _2011_ - Sinh học - Tế bào - Ref.pdfTrnHoa46
 
Chuong trinh dao tao Su pham Khoa hoc tu nhien, ma nganh - 7140247.pdf
Chuong trinh dao tao Su pham Khoa hoc tu nhien, ma nganh - 7140247.pdfChuong trinh dao tao Su pham Khoa hoc tu nhien, ma nganh - 7140247.pdf
Chuong trinh dao tao Su pham Khoa hoc tu nhien, ma nganh - 7140247.pdfhoangtuansinh1
 
GIÁO TRÌNH KHỐI NGUỒN CÁC LOẠI - ĐIỆN LẠNH BÁCH KHOA HÀ NỘI
GIÁO TRÌNH  KHỐI NGUỒN CÁC LOẠI - ĐIỆN LẠNH BÁCH KHOA HÀ NỘIGIÁO TRÌNH  KHỐI NGUỒN CÁC LOẠI - ĐIỆN LẠNH BÁCH KHOA HÀ NỘI
GIÁO TRÌNH KHỐI NGUỒN CÁC LOẠI - ĐIỆN LẠNH BÁCH KHOA HÀ NỘIĐiện Lạnh Bách Khoa Hà Nội
 
powerpoint lịch sử đảng cộng sản việt nam.pptx
powerpoint lịch sử đảng cộng sản việt nam.pptxpowerpoint lịch sử đảng cộng sản việt nam.pptx
powerpoint lịch sử đảng cộng sản việt nam.pptxAnAn97022
 
3-BẢNG MÃ LỖI CỦA CÁC HÃNG ĐIỀU HÒA .pdf - ĐIỆN LẠNH BÁCH KHOA HÀ NỘI
3-BẢNG MÃ LỖI CỦA CÁC HÃNG ĐIỀU HÒA .pdf - ĐIỆN LẠNH BÁCH KHOA HÀ NỘI3-BẢNG MÃ LỖI CỦA CÁC HÃNG ĐIỀU HÒA .pdf - ĐIỆN LẠNH BÁCH KHOA HÀ NỘI
3-BẢNG MÃ LỖI CỦA CÁC HÃNG ĐIỀU HÒA .pdf - ĐIỆN LẠNH BÁCH KHOA HÀ NỘIĐiện Lạnh Bách Khoa Hà Nội
 
Các điều kiện bảo hiểm trong bảo hiểm hàng hoá
Các điều kiện bảo hiểm trong bảo hiểm hàng hoáCác điều kiện bảo hiểm trong bảo hiểm hàng hoá
Các điều kiện bảo hiểm trong bảo hiểm hàng hoámyvh40253
 
sách sinh học đại cương - Textbook.pdf
sách sinh học đại cương   -   Textbook.pdfsách sinh học đại cương   -   Textbook.pdf
sách sinh học đại cương - Textbook.pdfTrnHoa46
 
1.DOANNGOCPHUONGTHAO-APDUNGSTEMTHIETKEBTHHHGIUPHSHOCHIEUQUA (1).docx
1.DOANNGOCPHUONGTHAO-APDUNGSTEMTHIETKEBTHHHGIUPHSHOCHIEUQUA (1).docx1.DOANNGOCPHUONGTHAO-APDUNGSTEMTHIETKEBTHHHGIUPHSHOCHIEUQUA (1).docx
1.DOANNGOCPHUONGTHAO-APDUNGSTEMTHIETKEBTHHHGIUPHSHOCHIEUQUA (1).docxTHAO316680
 

Último (12)

Đề cương môn giải phẫu......................
Đề cương môn giải phẫu......................Đề cương môn giải phẫu......................
Đề cương môn giải phẫu......................
 
PHÁT TRIỂN DU LỊCH BỀN VỮNG Ở TUYÊN QUANG
PHÁT TRIỂN DU LỊCH BỀN VỮNG Ở TUYÊN QUANGPHÁT TRIỂN DU LỊCH BỀN VỮNG Ở TUYÊN QUANG
PHÁT TRIỂN DU LỊCH BỀN VỮNG Ở TUYÊN QUANG
 
chuong-7-van-de-gia-dinh-trong-thoi-ky-qua-do-len-cnxh.pdf
chuong-7-van-de-gia-dinh-trong-thoi-ky-qua-do-len-cnxh.pdfchuong-7-van-de-gia-dinh-trong-thoi-ky-qua-do-len-cnxh.pdf
chuong-7-van-de-gia-dinh-trong-thoi-ky-qua-do-len-cnxh.pdf
 
Campbell _2011_ - Sinh học - Tế bào - Ref.pdf
Campbell _2011_ - Sinh học - Tế bào - Ref.pdfCampbell _2011_ - Sinh học - Tế bào - Ref.pdf
Campbell _2011_ - Sinh học - Tế bào - Ref.pdf
 
Chuong trinh dao tao Su pham Khoa hoc tu nhien, ma nganh - 7140247.pdf
Chuong trinh dao tao Su pham Khoa hoc tu nhien, ma nganh - 7140247.pdfChuong trinh dao tao Su pham Khoa hoc tu nhien, ma nganh - 7140247.pdf
Chuong trinh dao tao Su pham Khoa hoc tu nhien, ma nganh - 7140247.pdf
 
GIÁO TRÌNH KHỐI NGUỒN CÁC LOẠI - ĐIỆN LẠNH BÁCH KHOA HÀ NỘI
GIÁO TRÌNH  KHỐI NGUỒN CÁC LOẠI - ĐIỆN LẠNH BÁCH KHOA HÀ NỘIGIÁO TRÌNH  KHỐI NGUỒN CÁC LOẠI - ĐIỆN LẠNH BÁCH KHOA HÀ NỘI
GIÁO TRÌNH KHỐI NGUỒN CÁC LOẠI - ĐIỆN LẠNH BÁCH KHOA HÀ NỘI
 
powerpoint lịch sử đảng cộng sản việt nam.pptx
powerpoint lịch sử đảng cộng sản việt nam.pptxpowerpoint lịch sử đảng cộng sản việt nam.pptx
powerpoint lịch sử đảng cộng sản việt nam.pptx
 
3-BẢNG MÃ LỖI CỦA CÁC HÃNG ĐIỀU HÒA .pdf - ĐIỆN LẠNH BÁCH KHOA HÀ NỘI
3-BẢNG MÃ LỖI CỦA CÁC HÃNG ĐIỀU HÒA .pdf - ĐIỆN LẠNH BÁCH KHOA HÀ NỘI3-BẢNG MÃ LỖI CỦA CÁC HÃNG ĐIỀU HÒA .pdf - ĐIỆN LẠNH BÁCH KHOA HÀ NỘI
3-BẢNG MÃ LỖI CỦA CÁC HÃNG ĐIỀU HÒA .pdf - ĐIỆN LẠNH BÁCH KHOA HÀ NỘI
 
Các điều kiện bảo hiểm trong bảo hiểm hàng hoá
Các điều kiện bảo hiểm trong bảo hiểm hàng hoáCác điều kiện bảo hiểm trong bảo hiểm hàng hoá
Các điều kiện bảo hiểm trong bảo hiểm hàng hoá
 
sách sinh học đại cương - Textbook.pdf
sách sinh học đại cương   -   Textbook.pdfsách sinh học đại cương   -   Textbook.pdf
sách sinh học đại cương - Textbook.pdf
 
1.DOANNGOCPHUONGTHAO-APDUNGSTEMTHIETKEBTHHHGIUPHSHOCHIEUQUA (1).docx
1.DOANNGOCPHUONGTHAO-APDUNGSTEMTHIETKEBTHHHGIUPHSHOCHIEUQUA (1).docx1.DOANNGOCPHUONGTHAO-APDUNGSTEMTHIETKEBTHHHGIUPHSHOCHIEUQUA (1).docx
1.DOANNGOCPHUONGTHAO-APDUNGSTEMTHIETKEBTHHHGIUPHSHOCHIEUQUA (1).docx
 
1 - MÃ LỖI SỬA CHỮA BOARD MẠCH BẾP TỪ.pdf
1 - MÃ LỖI SỬA CHỮA BOARD MẠCH BẾP TỪ.pdf1 - MÃ LỖI SỬA CHỮA BOARD MẠCH BẾP TỪ.pdf
1 - MÃ LỖI SỬA CHỮA BOARD MẠCH BẾP TỪ.pdf
 

DE PHAT TRIEN THEO CAU TRUC DE MINH HOA 2022 MON TIENG ANH DE 1 5.pdf

  • 1. Question 1: They offered her the job because she was very _______ on the design front. A. creatively B. creative C. create D. Creation 1 (TH) Kiến thức: Từ loại Giải thích: A. creatively (adv): một cách sáng tạo B. creative (adj): sáng tạo C. create (v): tạo ra D. creation (n): sự sáng tạo Sau động từ “was” và trạng từ “very” cần điền tính từ. Dấu hiệu: đuôi –tion thường là danh từ, -ly thường là trạng từ, -ive thường là tính từ. Tạm dịch: Họ đề nghị cô ấy làm việc vì cô ấy rất sáng tạo trong lĩnh vực thiết kế.
  • 2. Question 2: She’ll take the flight for the early meeting, ______? A. won’t she B. hasn’t she C. didn’t she D. doesn’t she (NB) Kiến thức: Câu hỏi đuôi Giải thích: Vế trước câu hỏi đuôi dạng khẳng định => câu hỏi đuôi dạng phủ định Vế trước: She'll take => câu hỏi đuôi: won’t she? Tạm dịch: Cô ấy sẽ đáp chuyến bay cho cuộc họp sớm phải khôn
  • 3. Question 3: The letters he would write were full of doom and _______. We still do not know what made him so depressed. A. drib B. gloom C. duck D. Tuck 3 (VDC) Kiến thức: Thành ngữ Giải thích: A. drib (không tồn tại từ này) B. gloom (n): u ám, buồn bã C. duck (n): con vịt D. tuck (n): nếp gấp (ở quần áo) => doom and bloom: cảm giác tuyệt vọng, bi quan Tạm dịch: Những lá thư anh ấy viết chứa đầy cảm giác tuyệt vọng. Chúng tôi vẫn không biết điều gì đã khiến anh ấy chán nản như vậy.
  • 4. Question 4: The restaurant is well-known ________ its friendly atmosphere and excellent service. A. for B. on C. in D. Off 4(TH) Kiến thức: Giới từ Giải thích: A. for: cho B. on: trên C. in: trong D. off: rời, nghỉ => be well-known for something: nổi tiếng về cái gì Tạm dịch: Nhà hàng nổi tiếng với bầu không khí thân thiện và dịch vụ tuyệt vời.
  • 5. Question 5: The course was so difficult that I didn’t ………any progress at all. A. do B. make C. produce D. create 5 (TH) Make progress ~ improve: tiến bộ. Dịch: Khóa học này quá khó đến nỗi tôi không tiến bộ lên được chút nào
  • 6. Question 6: John crashed his truck _________ he was driving too fast. A. despite B. although C. because D. because of 6(TH) Kiến thức: Liên từ Giải thích: A. despite + cụm danh từ/ V_ing: mặc dù … B. although + S + V: mặc dù … C. because + S + V: bởi vì … D. because of + cụm danh từ/ V_ing: bởi vì … Sau chỗ trống là một mệnh đề => loại A, D. Tạm dịch: John đã gặp va chạm với xe tải của mình vì anh ta lái xe quá nhanh.
  • 7. Question 7: She bought a _________ jacket as a present for her daughter. A. new Italian leather B. new leather Italian C. leather new Italian D. leather Italian new 7 (TH) Kiến thức: Trật tự tính từ Giải thích: Khi có nhiều tính từ cùng đứng trước 1 danh từ, sắp xếp chúng theo thứ tự: OSASCOMP + N. Trong đó: O – opinion:quan điểm S – size:kích thước A – age: độ tuổi S – shape: hình dạng C – colour:màu sắc O – origin: nguồngốc M – material: chất liệu P – purpose:mục đích N – noun: danh từ Nếu có số thứ tự => đứng trước tính từ & danh từ => Trật tự tính từ: “new” – chỉ độ tuổi + “Italian”– chỉ nguồn gốc + “leather”– chỉ chất liệu Tạm dịch: Cô ấy đã mua một chiếc áo khoác da mới của Ý để làm quà cho con gái.
  • 8. Question 8: Unless we do something now, hundreds of plant and animal species will _______. A. point outB. die out C. cut up D. make up 8 (TH)Kiến thức: Cụm động từ Giải thích: A. point out: chỉ ra B. die out: chết đi C. cut up: cắt nhỏ D. make up: chiếm (phần trăm), bịa (chuyện), … Tạm dịch: Nếu chúng ta không làm điều gì đó ngay bây giờ, hàng trăm loài động thực vật sẽ chết.
  • 9. Question 9: While she was taking her exams, she _______ a terrible headache. A. will haveB. has C. had D. is having 9(TH) Kiến thức: Thì quá khứ đơn, phối hợp thì Giải thích: Cấu trúc: While + S + was/ were + V_ing, S + V_ed/ cột 2 (quá khứ đơn) Cách dùng: diễn tả một hành động đang xảy ra trong quá khứ (chia quá khứ tiếp diễn) thì một hành động khác xen vào (chia quá khứ đơn). Tạm dịch: Trong khi cô ấy đang làm bài thi thì cô ấy bị đau đầu kinh khủng. Chọn C.
  • 10. Question 10: I hope these machines will have worked very well ______. A. as soon as you would come back next month B. when you came back next month C. by time you come back next month D. after you were coming back next month 10 (TH) Kiến thức: Sự kết hợp thì Giải thích: Dấu hiệu: “next month” => chia các thì tương lai Cấu trúc: S + will have P2 + by the time + S + V_(s/es): … vào thời điểm … Động từ trong mệnh đề thời gian chia thì hiện tại, động từ trong mệnh đề chính chia tương lai. => loại A, B, D. Tạm dịch: Tôi hy vọng những chiếc máy này sẽ hoạt động rất tốt vào thời điểm bạn quay lại vào tháng sau.
  • 11. Question 11: The popular press often contains a lot more _______ than hard facts. A. tolerance B. influence C. speculation D. Realism 11 (VD) Kiến thức: Từ vựng Giải thích: A. tolerance (n): khoan dung B. influence (n): ảnh hưởng C. speculation (n): sự đồn đoán, đầu cơ D. realism (n): chủ nghĩa hiện thực Tạm dịch: Báo chí phổ thông thường chứa đựng nhiều thông tin đồn đoán hơn là những sự thật trần trụi.
  • 12. Question 12: Researchers have ________ to the conclusion that personality is affected by your genes. A. gotB. reached C. arrived D. Come 12 (VD) Kiến thức: Từ vựng, sự kết hợp từ Giải thích: A. got (P2): nhận được B. . reached (P2): chạm tới, đạt tới C. arrived (P2): đến D. come (P2): tới, về => come to the conclusion: đưa ra kết luận Tạm dịch: Các nhà nghiên cứu đã đưa ra kết luận rằng tính cách bị ảnh hưởng bởi gen của bạn.
  • 13. Question 13: The flood victims ________ with food and clean water by the volunteers A. provided B. were provided C. were providing D. provide 13 (TH) Kiến thức: Bị động thì quá khứ đơn Giải thích: Chủ ngữ là “The flood victims” – “Những nạn nhân lũ lụt”,động từ “provide” – “cung cấp” => câu mang nghĩa bị động Cấu trúc câu bị động thì quá khứ đơn: S + was/ were + Vp2 (by + O). Tạm dịch: Những nạn nhân lũ lụt được cung cấp đồ ăn và nước sạch bởi những tình nguyện viên. Chọn B
  • 14. Question 14: _______ the promotion, I began to search for other goals. A. Having attained B. To attain C. Being attained D. Attained 14 (VDC) Kiến thức: Rút gọn mệnh đề trạng ngữ Giải thích: Khi mệnh đề trạng ngữ và mệnh đề chính có cùng một chủ ngữ, có thể lược bỏ chủ ngữ ở mệnh đề trạng ngữ và đưa động từ về dạng V_ing khi mệnh đề ở dạng chủ động Để nhấn mạnh hành động ở mệnh đề trạng ngữ xảy ra trướchành động ở mệnh đề chính ta dùng dạng: having+ P2. Câu đầy đủ: I had attainedthe promotion, I began to searchfor other goals. Câu rút gọn: Having attainedthe promotion,I beganto searchfor other goals. Tạm dịch: Sau khiđạt được sự thăng tiến, tôi bắt đầu tìm kiếm các mục tiêu khác.
  • 15. Question 15: _______ the person is, the more privilege he enjoys. A. The rich B. The richest C. As rich as D. The richer 15(TH) Kiến thức: So sánh lũy tiến/ so sánh kép Giải thích: Cấu trúc so sánh lũy tiến: The + so sánh hơn + S + V, the + so sánh hơn + S + V Dạng so sánh hơn của tính từ “rich” là “richer”. Tạm dịch: Người càng giàu thì càng được hưởng nhiều đặc quyền.
  • 16. Question 16: Peter is talking to Mary about eating habits. - Peter: “You should eat more fruits and vegetables.” – Mary: “___________” A. Yes, I will, Thanks B. No, thanks C. My pleasure D. You’re welcome 16 (TH) Kiến thức: Ngôn ngữ giao tiếp Giải thích: Peter đang nói chuyện với Mary về thói quen ăn uống. Peter: “Bạn nên ăn nhiều trái cây và rau quả hơn.” Mary: “ ________ ” A. Ừ, tớ sẽ làm vậy. Cảm ơn nhé. B. Không, cảm ơn. C. Rất hân hạnh. D. Không có chi.
  • 17. Question 17: Mrs Brown and Mr Smith are talking about teaching soft skills at school. - Mrs Brown: “Some soft skills should be taught to children.” - Mr Smith: “______. They are necessary for them.” A. I don’t either B. I agree with you C. You’re quite wrong D. You’re welcome 17. (TH) Kiến thức: Ngôn ngữ giao tiếp Giải thích: Bà Brown và ông Smith đang nói về việc dạy các kỹ năng mềm ở trường. Bà Brown: “Một số kỹ năng mềm nên được dạy cho trẻ em.” Ông Smith: “ ________ . Chúng cần thiết cho họ.” A. Tôi cũng không B. Tôi đồng ý với bạn C. Bạn khá sai rồi D. Không có chi
  • 18. Question 18 A. document B. holiday C. location D. journalist 18 (NB) Kiến thức: Trọng âm từ có 3 âm tiết Giải thích: A. document /ˈdɒkjumənt/ B. holiday /ˈhɒlədeɪ/ C. location /ləʊˈkeɪʃn/ D. journalist /ˈdʒɜːnəlɪst/
  • 19. Question 19 A. listen B. agree C. escape D. deny 19 (NB) Kiến thức: Trọng âm từ có 2 âm tiết Giải thích: A. listen /ˈlɪsn/ B. agree /əˈɡriː/ C. escape /ɪˈskeɪp/ D. deny /dɪˈnaɪ/ Phương án A có trọng âm rơi vào âm tiết 1, còn lại là âm tiết 2. Chọn A.
  • 20. Question 20: A. concerned B. devoted C. renewed D. improved 20 (NB) Kiến thức: Cách phát âm đuôi “ed” Giải thích: A. concerned /kənˈsɜːnd/ B. devoted /dɪˈvəʊtɪd/ C. renewed /rɪˈnjuːd/ D. improved /ɪmˈpruːvd/ Quy tắc phát âm động từ đuôi –ed dựa vào chữ cái kết thúc: Những động từ có chữ cái kết thúc tận cùng là: TH 1: t, d => đuôi –ed được phát âm là /id/ TH 2: p, pe; k, ke; ff, ph, gh; ss, ce, se, x; ch; sh => đuôi –ed được phát âm là /t/ TH 3: còn lại => đuôi –ed được phát âm là /d/ Phần gạch chân phương án B được phát âm là /ɪd/, còn lại phát âm là /d/.
  • 21. Question 21: A. thrill B. hide C. prize D. crime Kiến thức: Cách phát âm “i” Giải thích: A. thrill /θrɪl/ B. hide /haɪd/ C. prize /praɪz/ D. crime /kraɪm/ Phần gạch chân phương án A được phát âm là /ɪ/, còn lại phát âm là /aɪ/. Chọn A.
  • 22. Question 22: Please give me some recommendation to buy suitable books for my ten-year-old girl A. advice B. defense C. interest D. question 22 (TH) Kiến thức: Từ vựng Giải thích: recommendation (n): sự giới thiệu, tiến cử A. advice (n): lời khuyênB. defense (n): sự che chở, phòng thủ C. interest (n): mối quan tâm D. question (n): câu hỏi => recommendation = advice Tạm dịch: Xin hãy cho tôi một vài giới thiệu để mua những cuốn sách phù hợp cho con gái mười tuổi của tôi.
  • 23. Question 23: He has been jobless and often has to ask his parents for money A. out of fashion B. out of practice C. out of order D. out of work 23 (TH) Kiến thức: Từ đồng nghĩa Giải thích: jobless (adj): không có việc làm, thất nghiệp A. out of fashion: lỗi thời B. out of practice: không rèn luyện C. out of order: bị hỏng (máy móc) D. out of work: thất nghiệp => jobless = out of work Tạm dịch: Anh ấy thất nghiệp và thường xuyên phải xin tiền bố mẹ.
  • 24. Question 24: Aren’t you putting the cart before the horse by deciding what to wear for the wedding before you’re invited to it? A. knowing the horse cart B. do things in the right order C. doing things in the wrong order D. upsetting the horse cart Kiến thức: Thành ngữ Giải thích: putting the cart before the horse: làm việc sai trình tự A. knowing the horse cart: biết đến xe ngựa B. do things in the right order: làm mọi việc theo đúng trình tự C. doing things in the wrong order: làm những việc không đúng trình tự D. upsetting the horse cart: làm đổ xe ngựa => putting the cart before the horse >< do things in the right order Tạm dịch: Bạn có đang làm sai trình tự khi quyết định mặc gì cho đám cưới trước khi bạn được mời đến dự không đó?
  • 25. Question 25: He is writing a letter of acceptance to the employer with the hope to get his favorite position in the company. A. confirmation B. refusal C. agree D. admission 25 (TH) Kiến thức: Từ vựng Giải thích: acceptance (n): sự chấp nhận A. confirmation (n): sự xác nhận B. refusal (n): sự từ chối C. agree (v): đồng ý D. admission (n): sự thu nạp, kết nạp => acceptance >< refusal Tạm dịch: Anh ấy đang viết một lá thư chấp nhận cho nhà tuyển dụng với hy vọng sẽ có được vị trí yêu thích trong công ty.
  • 26. Question 26: The book was interesting. I’ve read it three times. A. Such was the interesting book that I have read it three times. B. So interesting was the book that I have no time to read it. C. Only if it is an interesting book have I read it three times. D. Should the book be interesting, I have read it three times. 26 (VDC) Kiến thức: Đảo ngữ Giải thích: Đảo ngữ với “such … that …”: Such + be + cụm danh từ + that + S + V Đảo ngữ với “so … that …”: So + adj + be + S1 + that + S2 + V2 Đảo ngữ với “only if”: Only if + S1 + V1 + trợ động từ + S2 + V2 Đảo ngữ câu điều kiện loại 1: Should + S + V_infinitive, S + will/ can/ may + V_infinitive Tạm dịch: Cuốn sách thật thú vị. Tôi đã đọc nó ba lần. A. Đó là cuốn sách thú vị đến mức tôi đã đọc nó ba lần. B. Cuốn sách thú vị đến mức tôi không có thời gian để đọc nó. => sai nghĩa C. Chỉ khi nó là một cuốn sách thú vị, tôi mới đọc nó ba lần. => sai nghĩa D. sai ngữ pháp, sai nghĩa
  • 27. Question 27: She bought an old TV. She has regrets about it now. A. Provided she bought an old TV. She wouldn’t have regrets. B. If she hadn’t bought an old TV. She would have regrets. C. If only she had bought an old TV D. She wishes she hadn’t bought an old TV. Kiến thức: Câu ước Giải thích: Provided that + S + V_(s/es), S + will/ can/ may + V_infinitive: Miễn là Câu điều kiện hỗn hợp 3 và 2: If + S + had + P2, S + would/ could/ might + V_infinitive Diễn tả 1 điều kiện trái với quá khứ, dẫn đến 1 kết quả trái với hiện tại Cấu trúc: If only + S + had + P2: Giá mà ai đó đã làm gì trong quá khứ (nhưng đã không làm) Câu ước trái với quá khứ: S + wish(es) + S + had + P2 Tạm dịch: Cô ấy đã mua một chiếc TV cũ. Cô ấy hối hận về điều đó bây giờ. A. sai ngữ pháp: “provided” dùng cho câu điều kiện loại 1 B. Nếu cô ấy không mua một chiếc TV cũ, cô ấy sẽ hối tiếc. => sai nghĩa C. Giá mà cô ấy đã mua một chiếc TV cũ. => sai nghĩa D. Cô ấy ước cô ấy đã không mua một chiếc TV cũ.
  • 28. Question 28: Last night, she stays up so late to study for her exams. A B C D 28(NB) Kiến thức: Thì quá khứ đơn Giải thích: Dấu hiệu: “Last night” – tối hôm qua => chia thì quá khứ đơn Thì quá khứ đơn: S + V_ed/ cột 2 Diễn tả một hành động đã xảy trong quá khứ có thời điểm xác định Sửa: stays => stayed Tạm dịch: Đêm qua, cô ấy thức rất khuya để học bài cho kỳ thi.
  • 29. Question 29: She is a confidential and practiced speaker who always impresses her audience. Kiến thức: Từ vựng Giải thích: confidential (adj): bí mật, thân tín confident (adj): tự tin Sửa: confidential => confident Tạm dịch: Cô ấy là một diễn giả tự tin và có kinh nghiệm, người luôn gây ấn tượng với khán giả của mình. Chọn A.
  • 30. Question 30: My sister finally got his own favorite piano to practice every day. Kiến thức: Tính từ sở hữu Giải thích: Chủ ngữ “My sister” – danh từ chỉ người là phái nữ => dùng tính từ sở hữu “her” Sửa: his => her Tạm dịch: Em gái tôi cuối cùng đã có được cây đàn piano yêu thích của riêng mình để luyện tập mỗi ngày. Chọn B.
  • 31. Question 31: Every student is required to write an essay on the topic. A. Every student might write an essay on the topic. B. Every student must write an essay on the topic. C. Every student can’t write an essay on the topic. D. Every student needn’t write an essay on the topic. Kiến thức: Động từ khuyết thiếu Giải thích: be required to: được yêu cầu phải làm gì S + might + V_infinitive: Ai đó có thể làm gì S + must + V_infinitive: Ai đó phải làm gì S + can’t + V_infinitive: Ai đó không thể làm gì S + needn’t + V_infinitive: Ai đó không cần làm gì Tạm dịch: Mỗi học sinh được yêu cầu viết một bài luận về chủ đề này. A. Mỗi học sinh có thể viết một bài luận về chủ đề này. => sai nghĩa B. Mỗi học sinh phải viết một bài luận về chủ đề này. C. Mọi học sinh không thể viết một bài luận về chủ đề này. => sai nghĩa D. Mọi học sinh không cần phải viết một bài luận về chủ đề này. => sai nghĩa
  • 32. Question 32: “If I were you, I would rent anotherroom” saidmy friend. A. My friend threatenedme to rent anotherroom. B. My friend was thinking about renting another room. C. My friend insistedon renting anotherroom. D. My friend advisedme to rent anotherroom. Kiến thức: Câu tường thuật Giải thích: S + threatened + O + to V_infinitive: Ai đó đe dọa ai làm gì S + insisted on + V_ing: Ai đó nài nỉ làm gì S + advised + O + to V_infinitive: Ai đó khuyên ai làm gì Tạm dịch: “Nếu tôi là bạn, tôi sẽ thuê một phòng khác” bạn tôi nói. A. Bạn tôi đe dọa tôi phải thuê phòng khác. => sai nghĩa B. Bạn tôi đã nghĩ đến việc thuê một căn phòng khác. => sai nghĩa C. Bạn tôi đòi thuê phòng khác. => sai nghĩa D. Bạn tôi khuyên tôi nên thuê phòng khác
  • 33. Question 33: He last cooked for the whole family five months ago. A. He didn’t cook for the whole family five months ago. B. He has cooked for the whole family for five months. C. He hasn’t cooked for the whole family for five months. D. He would cook for the whole family in five months. Kiến thức: Thì hiện tại hoàn thành Giải thích: Cấu trúc: S + last + V_ed + khoảng thời gian + ago = S + have/ has + (not) + P2 + for + khoảng thời gian Tạm dịch: Lần cuối cùng anh ấy nấu cho cả gia đình là cách đây 5 tháng. A. Anh ấy đã không nấu ăn cho cả gia đình năm tháng trước. => sai nghĩa B. Anh ấy đã nấu ăn cho cả gia đình trong năm tháng. => sai nghĩa C. Anh ấy đã không nấu ăn cho cả gia đình trong năm tháng. D. Anh ấy sẽ nấu ăn cho cả gia đình trong năm tháng. => sai nghĩa
  • 34. Question 34: Read the following passage and mark the letter A, B, C or D on your answer sheet to indicate the correct word or phrase that best fits each of the numbered blanks from 1 to 5. A stinky gym bug in your kitchen? Who will be more upset by the smell - the men or the women in your family? (34) _______ scientists suggest that women not only smell, but feel, taste, and hear more accurately than men. A. Each B. Some C. Another D. Every Kiến thức: Lượng từ Giải thích: A. Each + danh từ đếm được dạng số ít: Mỗi … B. Some + danh từ đếm được dạng số nhiều: Một vài … C. Another + danh từ đếm được dạng số ít/ of + danh từ đếm được dạng số nhiều: … khác D. Every + danh từ đếm được dạng số ít: Mỗi … Sau chỗ trống là danh từ số nhiều, đếm được => loại A, C, D.
  • 35. Question 35: Take colors, for example. One study suggests that men are not as good as women at distinguishing between (35) _______ of color, although they focus well on rapidly changing images. A. smells B. senses C. shadows D. shades Kiến thức: Từ vựng, sự kết hợp từ Giải thích: A. smells (n): mùi B. senses (n): giác quan C. shadows (n): bóng tối D. shades (n): sắc thái => shades of color: sắc thái màu sắc Tạm dịch: Một nghiên cứu cho thấy rằng đàn ông không giỏi bằng phụ nữ trong việc phân biệt giữa các sắc thái màu sắc, mặc dù họ tập trung tốt vào những hình ảnh thay đổi nhanh chóng.
  • 36. Question 36: Evidence shows that boys hear as well as girls at birth, but with age, a man's hearing may soon deteriorate. ………………, environmental factors could play a role in this. A. Moreover B. ThereforeC. However D. Due to 36 (TH) Kiến thức: Liên từ Giải thích: A. Moreover + S + V, …: Hơn thế nữa … (thêm ý) B. Therefore, …: do đó … (chỉ kết quả) C. However, …: tuy nhiên … (chỉ ý đối lập) D. Due to + cụm danh từ/ V_ing, …: vì …(chỉ nguyên nhân) Tạm dịch: Bằng chứng cho thấy rằng các bé trai nghe tốt như các bé gaí khi mới sinh, nhưng theo tuổi tác, thính lực của đàn ông có thể sớm bị suy giảm. Tuy nhiên, các yếu tố môi trường có thể đóng vai trò trong việc này.
  • 37. Question 37 Women may also be better at identifying different flavors …………….. need both taste and smell to experience, as studies have shown that they have more taste buds on their tongue. A. who B. which C. where D. whom Kiến thức: Đại từ quan hệ Giải thích: A. who + V: người mà … => bổ sung thông tin cho từ chỉ người trước nó B. which + V: cái mà … => bổ sung thông tin cho từ chỉ vật trước nó C. where + S + V: khi mà … => bổ sung thông tin cho từ chỉ địa điểm trước đó D. whom + S + V: người mà … => bổ sung thông tin cho từ chỉ người trước nó “different flavors” – danh từ chỉ vật => loại A, C, D. Tạm dịch: Phụ nữ cũng có thể giỏi hơn trong việc xác định các hương vị khác nhau những thứ cần cả vị giác và khứu giác để trải nghiệm, vì các nghiên cứu đã chỉ ra rằng họ có nhiều nụ vị giác hơn trên lưỡi.
  • 38. Question 38: A. taken B. sprung C. mazed D. freaked Going back to the smelly gym bag - yes, it's likely that Mom will be the most ……………..out by it. Kiến thức: Từ vựng Giải thích: A. taken (P2): lấy B. sprung (P2): nhảy ra, bật lên C. mazed (không tồn tại từ này) D. freaked (P2): sốc, sợ hãi, ngạc nhiên => freaked out: sốc, sợ hãi, ngạc nhiên Tạm dịch: Quay trở lại với chiếc túi tập thể dục bốc mùi - vâng, có khả năng mẹ sẽ là người sốc nhất với nó.
  • 39. Question39:Which of the followingbest states the main idea of the reading? A. No one knows for sure where dogs firstappearedwith humans. B. There are two theories about the evolutionof dogs. C. Evolutionand breeding have made dogs what they are today. D. Human beings used dogs for many jobs, so dogs are the first tame animals. Kiến thức: Đọc hiểu Giải thích: Câu nào sau đây thể hiện đúng nhất ý chính của bài đọc? A. Không ai biết chắc nơi đầu tiên loài chó xuất hiện cùng với con người. B. Có hai giả thuyết về sự tiến hóa của loài chó. C. Sự tiến hóa và lai tạo đã tạo nên những con chó D. Loài người đã sử dụng chó vào nhiều công việc, vì vậy chó là con vật được thuần hóa đầu tiên. Thông tin: - Dogs originally came from wolves. - At some point, some wolves changed through evolution. Some of them were smaller and gentler. These nicer wolves were the first dogs. - Some scientists believe that dogs mixed with other animals from the Canidae family. This includes coyotes, foxes and jackals. Most scientists also believe that evolution and breeding influenced the variation.
  • 40. Question 40: The word “they” in the first paragraph refers to _______. A. people B. years C. dogs D. wolves Kiến thức: Đọc hiểu Giải thích: Từ “they” trong đoạn đầu tiên đề cập đến ________ . A. people (n): người B. years (n): năm C. dogs (n): những con chó D. wolves (n): những con chó sói Thông tin: Wolves look a lot like dogs, but they are bigger. Tạm dịch: Chó sói trông rất giống chó, nhưng chó sói lớn hơn.
  • 41. Question 41: According to some scientists, when did the first tame dogs appear with humans? A. About 15,000years ago B. Between 15,000and 50,000years ago C. Over 50.000years ago D. About 13,000years ago Kiến thức: Đọc hiểu Giải thích: Theo một số nhà khoa học, những con chó thuần hóa đầu tiên xuất hiện cùng với con người khi nào? A. Khoảng 15.000 năm trước B. Từ 15.000 đến 50.000 năm trước C. Hơn 50.000 năm trước D. Khoảng 13.000 năm trước Thông tin: Scientists who study humans say the first tame dogs appeared with humans about 13,000 years ago in the Middle East. Tạm dịch: Các nhà khoa học nghiên cứu con người cho biết những con chó đầu tiên được thuần hóa đã xuất hiện cùng với loài người vào khoảng 13.000 năm trước ở Trung Đông.
  • 42. Question 42: The word “companions” in the second paragraphmostly means ______. A. messengers B. co-workers C. travelingfriends D. hunting animals Kiến thức: Đọc hiểu Giải thích: Từ “companions” trong đoạn thứ hai chủ yếu có nghĩa là ________ . companions: bạn đồng hành A. messengers (n): sứ giả, người đưa tin B. co-workers (n): đồng nghiệp C. traveling friends (n): bạn cùng đi du lịch D. hunting animals (n): săn bắn động vật => companions = traveling friends Thông tin: They were also good companions. Tạm dịch: Chúng cũng là những người bạn đồng hành tốt.
  • 43. Question 43: According to the reading, which of the following is TRUE? A. Most people like to name their dogs Reliable, Blacky, and Useless. B. The Roman always used dogs as messengers. C. The first tame dogs appeared in ancient Egypt. D. According to some studies, dogs mixed with coyotes and jackals. Kiến thức: Đọc hiểu Giải thích: Theo bài đọc, điều nào sau đây là ĐÚNG? A. Hầu hết mọi người đều thích đặt tên cho những chú chó của họ là Reliable, Blacky và Useless. B. Người La Mã luôn dùng chó làm người đưa tin. C. Những chú chó được thuần hóa đầu tiên xuất hiện ở Ai Cập cổ đại. D. Theo một số nghiên cứu, chó lai tạp với chó sói đồng cỏ và chó rừng. Thông tin: Some scientists believe that dogs mixed with other animals from the Canidae family. This includes coyotes, foxes and jackals. Tạm dịch: Một số nhà khoa học tin rằng những loài chó đã phối ngẫu với các động vật khác từ họ Canidae. Bao gồm sói đồng cỏ, cáo và
  • 44. Question 44: Which of the following could be served as the best title for the passage? A. Urban cities - The new opportunity for communitydevelopment B. Urbanization - Pros and cons C. Urbanization - How people’s health is impacted? D. Developing countries - The fastest urbanization Kiến thức: Đọc hiểu Giải thích: Điều nào sau đây có thể được coi là tiêu đề hay nhất cho đoạn văn? A. Thành phố đô thị - Cơ hội mới để phát triển cộng đồng B. Đô thị hóa - Ưu và nhược điểm C. Đô thị hóa - Sức khỏe của con người bị ảnh hưởng như thế nào? D. Các nước đang phát triển - Đô thị hóa nhanh nhất Thông tin: Despite its positive things, there are also negatives from urbanization on the physical health of humans living. Tạm dịch: Mặc dù có nhiều điều tích cực của việc này, nhưng cũng có những tiêu cực từ việc đô thị hóa đối với sức khỏe thể chất của con người.
  • 45. Question 45: The word “agrarian” in paragraph 2 mostly means ________. A. farming B. industry C. city D. modernizing Kiến thức: Đọc hiểu Giải thích: Từ “agrarian” trong đoạn 2 chủ yếu có nghĩa là ________ . agrarian (n): nông nghiệp A. farming (n): nông nghiệp B. industry (n): công nghiệp C. city (n): thành phố D. modernizing (n): hiện đại hóa => agrarian = farming Thông tin: China is a country that in the past 30-40 years went from being an agrarian based society to a significant industrialized country. Tạm dịch: Trung Quốc là một quốc gia mà trong vòng 30 - 40 năm qua đã từ một xã hội nông nghiệp trở thành một nước công nghiệp với sự phát triển đáng kể.
  • 46. Question 46: The word “congested” in paragraph 3 is closest in meaning to _______. A. fresh B. overcrowded C. sparse D. contaminated Kiến thức: Đọc hiểu Giải thích: Từ “congested” trong đoạn 3 gần nghĩa nhất với ________ . congested (adj): đông đúc, mật độ giao thông cao A. fresh (adj): tươi B. overcrowded (adj): đông đúc C. sparse (adj): thưa thớt D. contaminated (adj): bị ô nhiễm => congested = overcrowded Thông tin: One very common and fairly obvious negative aspect of highly congested urban areas is air pollution. Tạm dịch: Một khía cạnh tiêu cực khác rất phổ biến và khá rõ ràng của các khu đô thị vô cùng đông đúc là ô nhiễm không khí.
  • 47. Question 47: The following are the air pollution sources mentioned in paragraph 3, EXCEPT _______. A. industrial plants B. sewage C. refineries waste D. chemicals Kiến thức: Đọc hiểu Giải thích: Sau đây là các nguồn ô nhiễm không khí được đề cập trong đoạn 3, NGOẠI TRỪ ________ . A. industrial plants: nhà máy công nghiệp B. sewage: nước thải C. refineries waste: chất thải nhà máy lọc dầu D. chemicals: hóa chất Thông tin: It could include particulate matter, most commonly attributed to industrial plants and refineries waste, or chemicals like CO2 or Methane. Tạm dịch: Nó có thể bao gồm các chất dạng hạt thường được cho là do chất thải của các nhà máy công nghiệp và nhà máy lọc dầu, hoặc các chất hóa học như CO2 hoặc Metan.
  • 48. Question 48: The word “it” in the last paragraph refers to ________? A. quality B. sugar C. food D. sodium Kiến thức: Đọc hiểu Giải thích: Từ “it” trong đoạn cuối đề cập đến ________ . A. quality (n): chất lượng B. sugar (n): đường C. food (n): thực phẩm D. sodium (n): natri Thông tin: Because this food is so accessible, people tend to eat it more. Tạm dịch: Bởi vì thực phẩm này rất dễ tiếp cận, mọi người thường có xu hướng ăn thực phẩm này nhiều hơn.
  • 49. Question49:Why are urban populations easy to get diseasesfrom food, accordingto the last paragraph? A. Becauseof the change in people’s diet. B. Because this food is so deliciousthat people have a tendency to eat more than normal. C. Becausethe way people get this food is ratheraccessible, quick and easy. D. Becauseof the low qualityand the high proportionof sodiumand sugar in this food. Kiến thức: Đọc hiểu Giải thích: Tại sao dân thành thị dễ mắc bệnh từ thức ăn, theo đoạn cuối? A. Do chế độ ăn của con người thay đổi. B. Vì thực phẩm này rất ngon nên mọi người có xu hướng ăn nhiều hơn bình thường. C. Vì cách mọi người có được thực phẩm này khá dễ tiếp cận, nhanh chóng và dễ dàng. D. Vì thực phẩm này có chất lượng thấp và tỷ lệ natri và đường cao. Thông tin: This food is also more than likely not as high quality as well as contains a large amount of sodium and sugar. Tạm dịch: Các loại thực phẩm này nhiều khả năng không có chất lượng cao cũng như chứa một lượng lớn natri và đường.
  • 50. Question 50: What can be inferred fromthe passage? A. One of the negativehealth effects comes from the overpopulationin industrializedcountries. B. Living in urban areas for a long time will certainlymake the life expectancy of inhabitantsshorten. C. Peoplein developedcountriessuffer less harmful health effects from urbanizationthan thosein developingnations. D. The bad health effects from urbanizationare not greater than the benefits it brings to people in Kiến thức: Đọc hiểu - Điều gì có thể được suy ra từ đoạn văn? A. Một trong những ảnh hưởng tiêu cực đến sức khỏe là do dân số quá đông ở các nước công nghiệp phát triển. B. Sống ở thành thị trong thời gian dài chắc chắn sẽ làm cho tuổi thọ của cư dân bị rút ngắn. C. Người dân ở các nước phát triển ít bị ảnh hưởng sức khỏe do đô thị hóa hơn người dân ở các nước đang phát triển. D. Những ảnh hưởng xấu đến sức khoẻ do đô thị hoá không lớn hơn những lợi ích mà nó mang lại cho người dân ở các thành phố đô thị. Thông tin: As it would be expected, developing countries tend to see more negative physical health effects than modern countries in regard to urbanization. Tạm dịch: Đúng như dự đoán, các nước đang phát triển đều có xu hướng nhận thức được việc xuất hiện nhiều ảnh hưởng tiêu cực đến sức khỏe thể chất con người hơn là các nước hiện đại liên quan đến quá trình đô thị hóa.
  • 51. Question 1. You may find doing this job very__________. Try it! A. relaxing B. relaxed C. relax D. relaxation Kiến thức: Từ loại Giải thích: A. relaxing (a): thoải mái, dễ chịu (tính từ đuôi ing, có yếu tố chủ động, thể hiện bản chất, đặc điểm của sự vật, sự việc, tự thân nó có) B. relaxed (a): thoải mái, dễ chịu (tính từ đuôi ed có yếu tố bị động, nó không tự có tính chất này mà là bên ngoài tác động vào khiến nó có cảm giác như vậy) C. relax (v): thư giãn D. relaxation (n): sự thư giãn, thoải mái Ta có cấu trúc “Find sb/ sth ADJ’: thấy ai/ cái gì/ việc gì như thế nào. Vì thế vị trí chỗ trống ta cần một tính từ. Và tính từ này để nói rõ tính chất của “sb/sth” phía trước nên phải dùng tính từ đuôi ing.. Tạm dịch: Có lẽ bạn sẽ thấy công việc này rất thoải mái. Hãy thử nó đi.
  • 52. Question 2. No one on the plane was alive in the accident last night, __________? A. wasn’t he B. weren’t they C. were they D. was he Kiến thức: Câu hỏi đuôi Giải thích: Chủ ngữ là “No one” thì ở câu hỏi đuôi ta dùng đại từ “they”. Các đại từ bất định chỉ người như “everyone, everybody, someone, somebody,anyone, anybody, no one, nobody” ta dùng “they” cho vế câu hỏi đuôi. Các đại từ bất định chỉ vật như “everything, something, anything, nothing” ta dùng “it” cho vế câu hỏi đuôi. Vế trước chủ ngữ “no one”: không một ai, chứa yếu tố phủ định, nên câu hỏi đuôi sẽ ở thể khẳng định, do đó đáp án phù hợp là “were they”. Chọn C. Tạm dịch: Không ai trên máy bay còn sống trong vụ tai nạn đêm qua, phải không?
  • 53. Question 3. We had better keep on our__________ while were walking along the dark portions of this street. A. figures B. nails C. toes D. knees Kiến thức: Idiom Giải thích: Thành ngữ: keep on our toes: thận trọng, đề phòng Chọn C Tạm dịch: Chúng ta nên cảnh giác đề phòng khi chúng ta đi bộ ở những đoạn đường tối ở thành phố này.
  • 54. Question 4. He regretted spending too much time _____ computer games. A. to B. for C. in D. on Kiến thức: Giới từ Giải thích: Ta dùng cấu trúc: S + spend time/money on sth: tiêu xài thời gian/ tiền bạc vào cái gì. Chọn D. Ta cũng có: S + spend time/ money on sth: tiêu xài thời gian/ tiền bạc vào cái gì. Chọn D. Ta cũng có: S + spend time/ money Ving: tiêu xài thời gian/ tiền bạc vào việc gì. Tạm dịch: Anh ta thấy hối hận vì đã dùng quá nhiều thời gian chơi điện tử.
  • 55. Question 5. He looks for any excuse he can to blow off his__________ to do housework. A commitment B. obligation C. assignment D. responsibility Kiến thức: Từ vựng cùng trường nghĩa Giải thích: A. commitment (n): lời cam kết B. obligation (n): nghĩa vụ C. assignment (n): nhiệm vụ được giao D. responsibility (n): trách nhiệm Ta có cụm thường gặp “blow off the responsibility”: rũ bỏ trách nhiệm. chọn D Tạm dịch: Anh ta tìm mọi lý do có thể để rũ bỏ trách nhiệm phải làm việc nhà
  • 56. Question 6. __________ what she prepared for the job interview, Megan didn’t pass it. A. Despite of B. In spite of C. Though D. However Kiến thức: Liên từ Giải thích: Despite/ In spite of + N/ Ving = Though + S + V: mặc dù However S+V: Tuy nhiên Ta có: “What + s + V” là mệnh đề danh từ → Dùng “Despite/ In spite of”. Loại A do despite không theo cùng với of, đồng thời loại C và D vì sau Though là một mệnh đề và However không đứng đầu để nối hai mệnh đề với nghĩa “tuy nhiên” như này, However chỉ nối câu với câu hoặc đoạn với đoạn hoặc đứng ở giữa được ngăn cách bởi dấu chấm phảy và dấu phảy. Tạm dịch: Bất chấp những gì cô ấy chuẩn bị cho buổi phỏng vấn xin việc, Megan đã không vượt qua nó.
  • 57. Question 7. There is __________ table in my bedroom. A. an old square wooden B. a square wooden old C. an old wooden square D. a wooden old square Kiến thức: Trật tự tính từ Giải thích: Trật tự tính từ: OSASCOMP 1. Opinion and general description (Ý kiến hoặc miêu tả chung) Ví dụ: nice, awesome, lovely... 2. Size / Weight(Kích cỡ, cân nặng) Ví dụ: big, small, heavy... 3. Age (Tuổi, cũ-mới) Ví dụ: old, new, young, ancient... 4. Shape (Hình dạng) Ví dụ: round, square, oval... 5. Color (Màu sắc) Ví dụ: green, red, blue,... 6. Origin (Xuất xứ) Ví dụ: Swiss, Italian, English.... 7. Material (Chất liệu) Ví dụ: woolly, cotton, plastic... 8. Purpose (Mục đích) Ví dụ: walking (socks), running (shoes).... Ta có sắp xếp đúng: old (cũ) - age → square (hình vuông) - shape → wooden(bằng gỗ) – material Chọn đáp án A Tạm dịch: Có một chiếc bàn gỗ cũ hình vuông trong phòng của tôi.
  • 58. Question 8. The new airport has__________ a lot of changes on this island. A. brought about B. taken to C. counted in D. turned up Kiến thức: Phrasal verbs Giải thích: A. bring about: mang lại B. take to: bắt đầu thích C. count in: hoạt động cùng ai D. turn up: xuất hiện Tạm dịch: Sân bay mới đã mang lại nhiều thay đổi trên hòn đảo này.
  • 59. Question 9. They__________ the bridge by the time you come back. A. will finish B. will have finished C. will be finished D. have finished Kiến thức: sự phối hợp thì Giải thích: Thì tương lai hoàn thành dùng để diễn tả một hành động hoặc một sự việc xảy ra và hoàn thành trước một hành động hoặc một thời điểm khác trong tương lai. Công thức: S + will + have + P2 by the time + S + V(s/es). Chú ý: By the time là dấu hiệu để chia thì tương lai hoàn thành nếu sau “by the time” có yếu tố hiện tại hoặc tương lai, còn by the time sẽ là dấu hiệu chia quá khứ hoàn thành nếu sau cụm này có yếu tố của quá khứ. Câu này có yếu tố tương lai nên chọn B. Tạm dịch: Họ sẽ hoàn thành xong cây cầu trước khi anh quay lại.
  • 60. Question 10. She won’t come home __________. A. as soon as she had finished all the paperwork B. until she has finished all the paperwork C. by the time she finished all the paperwork D. when she was finishing all the paperwork Kiến thức: Mệnh để trạng ngữ Giải thích: Ta sử dụng kiến thức phổi hợp thì và kiến thức về liên từ để giải quyết. Mệnh đề cho trước đang là tương lai đơn nên không thể dùng A (quá khứ hoàn thành); không dùng C (quá khứ đơn), và không dùng D (quá khứ tiếp diễn) do tương lai không kết hợp với quá khứ.
  • 61. Question 11. The Youth Union in our school has decided to launch a/an__________ to raise funds for local charities. A. activity B. announcement C. campaign D. decision Kiến thức: Từ vựng A. activity: hoạt động B. announcement:thông báo C. campaign: chiến dịch, phong trào D. decision: quyết định Ta có: launch a campaign: phát động một phong trào Tạm dịch: Hội thanh niên trường tôi đã phát động phong trào để gây quỹ cho các tổ chức từ thiện địa phương.
  • 62. Question 12. Being helpful is good, but don’t allow others to__________ advantage of your generosity. A. getB. take C. useD. make Kiến thức: Cụm từ Giải thích: Ta có: take advantage of: tận dụng/ lợi dụng = make use of: sử dụng/ lợi dụng. Chọn B. Tạm dịch: Có ích thì tốt, nhưng đừng để người khác lợi dụng sự hào phóng của bạn.
  • 63. Question 13. I demand to know how this vase __________, and no one is leaving till I find out. A. got broken B. was breaking C. has broken D. is broke Kiến thức: Câu bị động. Giải thích: Ngoài cấu trúc bị động quen thuộc là be + P2 thì ta còn có cách diễn đạt bị động khác nữa với cấu trúc: Get + P2. Loại B và C là 2 phương án viết ở chủ động, loại D do cách thể hiện “is broke” sai ngữ pháp.
  • 64. Question 14. Although __________ by the bravery of his fellow soldiers, Bloch had harsh words for the army leadership. A. was impressed B. impressed C. having impressed D. impressing Kiến thức: Giản lược mệnh đề cùng chủ ngữ: Giải thích: Though/ Although + S + V, S + V: Mặc dù ... Nếu chủ ngữ của 2 mệnh đề giống nhau thì ta có thể rút gọn mệnh đề nhượng bộ theo cấu trúc: Although+ ADJ/ P2, S + V. Câu này cùng chủ ngữ là Bloch, có thể lược bỏ chủ ngữ ở mệnh đề đầu tiên và giữ lại phần P2 (vì là câu bị động). Ta chọn B. Tạm dịch: Mặc dù bị ấn tượng bởi lòng dũng cảm của những người lính, Bloch vẫn có những từ ngữ khá khắc nghiệt cho việc lãnh đạo quân đội. Tạm dịch: Tôi yêu cầu được biết làm sao mà cái lọ hoa này lại bị vỡ, và sẽ không ai rời đi cho đến khi nào tôi tìm ra.
  • 65. Question 15. The more you practice speaking in public, ____. A. the more you become confident B. the more you become confidently C. the greater confidence you become D. the more confident you become Kiến thức: Câu so sánh đồng tiên: Càng... càng Cấu trúc: The ...er/ more...S + V, the...er/ more...S + V. Đáp án D đúng cấu trúc yêu cầu. Các phương án khác sai cấu trúc nên loại. Chọn D. Tạm dịch: Bạn càng chịu khó luyện tập nhiều trước đám đông thì bạn càng trở nên tự tin hơn. Đáp án B sử dụng hiện tại hoàn thành kết hợp hoàn hảo với thì tương lai như câu đã cho. Tạm dịch: Cô ta sẽ không quay trở về nhà cho tới khi nào cô ấy hoàn thành xong tất cả các công việc giấy tờ.
  • 66. Question 16: Tom is inviting Linda to his birthday party. Tom: “Would you like to come to my birthday party next week?" Linda: “……………………..’’ A. Why not? B. Yes, I'd love to. C. I don’t think so. D. N 0, I‘d love to. Tình huống giao tiếp Tạm dịch: Tom đang mời Linda tới bữa tiệc sinh nhật của mình. Tom: “cậu có muốn tham gia bữa tiệc sinh nhật vào tuần sau của tớ không?" Linda: " ." Tại sao không nhỉ? (đáp lại khi ai đó đưa ra ý kiến) Có, tớ rất muốn đi (đáp lại lời mời) Tớ không nghĩ vậy đâu Không, tớ rất muốn
  • 67. Question 17: Laura is asking Tom for his idea about a vacation at the beach. Laura: “Do you think a vacation at the beach will do me good?” Tom: " ……………" A. Sure. Have a good time there. B. Yes, I think. C. Could you bye me something? D. Yes, it does. Tình huống giao tiếp Tạm dịch: Laura đang hỏi Tom về ý tưởng của anh ấy cho kì nghỉ ở bãi biển. Laura: "Cậu có nghĩa rằng đi nghỉ ở biển là tốt cho tớ không?" Tom : “” A. chắc chắn rồi. Chúc đi vui vẻ nha. B. có. Tớ nghĩ vậy C. cậu có thể mua cho tớ vài thứ được không? D. có. Nó sẽ như vậy mà.
  • 68. Question 18. A. equip B. secure C. vacant D. oblige Đáp án C đúng vì đáp án C có trọng âm rơi vào âm thứ nhất. Các phương án còn lại có trọng âm rơi vào âm tiết thứ hai. equip (v) /ɪˈkwɪp/: trang bị secure (adj) /sɪˈkjʊə/: an toàn vacant (adj) /ˈveɪkənt/: còn trống oblige (v) /əˈblaɪʤ/: bắt buộc
  • 69. Question19. A. encounter B. agency C. influence D. memory Đáp án A đúng vì đáp án A có trọng âm rơi vào âm thứ hai. Các phương án còn lại có trọng âm rơi vào âm tiết thứ nhất. encounter (v) /ɪnˈkaʊntə/: đương đầu agency (n) /ˈeɪʤənsi/: đơn vị, đại lí influence (v/n) /ˈɪnflʊəns/: ảnh hưởng memory (n) /ˈmɛməri/: kí ức, trí nhớ
  • 70. Question 20. A. asked B. danced C. cashed D. studied Kiến thức: Cách phát âm đuôi “ed” Giải thích: - Đuôi “ed” được phát âm là /id/: Khi động từ có phát âm kết thúc là /t/ hay /d/ VD: wanted, needed, invited, decided - Đuôi “ed” được phát âm là /t/: Khi động từ có phát âm kết thúc là: /s/: CE, X, SS; /f/: GH, PH, /p/, /ʃ/:SH, /tʃ/: CH, /k/ VD: danced, fixed, crossed, laughed, photographed, washed, watched, booked,... - Đuôi “ed” được phát âm là /d/ với những trường hợp còn lại. VD: considered, received, stayed,... Đáp án D đúng vì phần gạch chân của đáp án D được đọc là /d/. Các phương án còn lại phẩn gạch chân được đọc là /t/. A. asked /ɑːskt/: hỏi B. danced /dɑːnst/: nhảy, khiêu vũ C. cashed /kæʃt/: đổi tiền mặt D. studied /ˈstʌdɪd/: học
  • 71. Question 21. A. profile B. stomach C. postpone D. cyclone Kiến thức: Cách phát âm nguyên âm “o” Giải thích: Đáp án B đúng vì phần gạch chân của đáp án B được đọc là /ʌ/. Các phương án còn lại phần gạch chân được đọc là /əʊ/. profile (n) /ˈprəʊfaɪl/: hồ sơ, sơ yếu lí lịch stomach (n) /ˈstʌmək/: dạ dày, bụng postpone (v) /pəʊstˈpəʊn/: trì hoãn cyclone (n) /ˈsaɪkləʊn/: lốc. gió xoáy
  • 72. Question 22: Such problems as haste and inexperience are a universal feature of youth. A. marked B. shared C. hidden D. separated Kiến thức: Từ đồng nghĩa Giải thích: universal (adj): phổ biến, chung shared: được chia sẻ, chung marked (adj): rõ rệt hidden: ẩn, bị giấu đi separated (adj): ly thân => universal = shared Tạm dịch: Những vấn đề như sự vội vàng và thiếu kinh nghiệm là một đặc điểm chung của giới trẻ.
  • 73. Question 23: If that was done on a national scale, we would wipe out this infectious disease. A. establish B. retain C. maintain D. eliminate
  • 74. Question 24: Fee-paying schools, often called “independent schools”, "private schools" or "public schools“ A. college B. primary schools C. secondary schools D. state schools Từ trái nghĩa college /'kɔliddʒ/ (n): trường Đại học, Cao đẳng primary school (n): trường Tiểu học secondary school (n): trường Trung học phổ thông state school (n): trường công lập Tạm dịch: Những trường học phải trả phí thường được gọi là trường dân lập hay là trường tư thục. => independent schools >< state schools
  • 75. Question 25: Each time you turn it on, with appropriate hardware and software, it is capable of doing almost anything you ask. A. unsuitable B. unimportant C. ill-prepared D. irregular Từ trái nghĩa unsuitable /ʌn’su:təbl/ (a): không phù hợp, không thích hợp unimportant /,ʌnim'pɔ:tənt/ (a): không quan trọng ill-prepared (a): thiếu sự chuẩn bị irregular /i'regjulə/ (a): không đều, bất quy tắc Tạm dịch: Mỗi khi bạn bật nó lên, với phần cứng và phần mềm phù hợp, nó có thể làm gần như bất cứ thứ gì mà bạn yêu cầu. => appropriate >< unsuitable
  • 76. Question26. Hans told US about his investingin the company. He did it on his arrivalat the meeting. A. Only after investing in the company did Hans inform US of his arrival at the meeting. B. Not until Hans told US that he would invest in the company did he arrive at the meeting. C. Hardly had he informed US about his investing in the company when Hans arrived at the meeting. D. No sooner had Hans arrived at the meeting than he told US about his investing in the company. Kiến thức: Câu đảo ngữ Câu đê' bài cho: Hans nói với chúng tôi về đầu tư của mình trong công ty. Ông đã làm điều đó ngay khi đến cuộc họp. A. Chỉ sau khi đầu tư vào công ty Hans mới bảo cho chúng tôi khi ông ấy đến cuộc họp. B. Mãi cho đến khi Hans nói với chúng tôi rằng ông sẽ đầu tư vào công ty ông mới đến cuộc họp. C. Ngay khi ông thông báo cho chúng tôi về việc đầu tư của mình trong công ty thì Hans đến cuộc họp. D. Ngay khi Hans đến tại cuộc họp thì ông ấy nói với chúng tôi về đầu tư của mình trong công ty
  • 77. Question 27. The driver in front stopped so suddenly. Therefore, the accident happened. A. If the driver in front didn’t stop so suddenly, the accident wouldn’t happen. B. If the driver in front hadn’t stopped so suddenly, the accident would have happened. C. If the driver in front hadn’t stopped so suddenly, the accident wouldn’t have happened. D. If the driver in front had stopped so suddenly, the accident would have happened. Kiến thức: Câu điều kiện Câu đề bài cho: Người lái xe phía trước đã dừng đột ngột. Vì thế, vụ tai nạn đã xảy ra. Đề bài cho một tình huống đã xảy ra trong quá khứ nên ta nhận định sẽ viết về câu điều kiện loại 3 diễn tả những điều không có thật trong quá khứ với công thức: If S + had P2, S + would have P2. A loại do A viết về câu điều kiện loại 2 diễn tả những điều không có thật ở hiện tại. B loại do sai nghĩa: Nếu người lái xe phía trước không dừng đột ngột thì vụ tai nạn sẽ đã xảy ra. C đúng: Nếu người lái xe phía trước không dừng đột ngột thì vụ tai nạn sẽ đã không xảy ra. D loại do sai nghĩa: Nếu người lái xe phía trước dừng đột ngột thì vụ tai nạn sẽ đã xảy
  • 78. Question 28: What I told him a few days ago is not the solution to most of his problems. Kiến thức ngữ pháp: ago → was Chủ ngữ là mệnh đề danh từ động từ chia số it : were – was
  • 79. Question 29: Tom’s jokes are inappropriate but we have to put up with it just because he’s the boss. Kiến thức: từ vựng Giải thích: “jokes” là danh từ ở dạng số nhiều nên phải dùng tân ngữ “them” để thay th ế. it => them Tạm dịch: Những câu chuyện cười của Tom không phù hợp nhưng chúng ta phải chịu đựng chúng chỉ vì ông ấy là ông chủ.
  • 80. Question 30: Modern office buildings have false floors under which computer and phone wires can be lain. Kiến thức về cặp từ dễ gây nhầm lẫn lay và lie lay - laid - laid : đặt, xếp thứ gì đó nằm ở vị trí tĩnh lie – lay - lain: tựa lên, nằm nghỉ trên một vị trí bằng phẳng => Đáp án là D(can be lain => can be laid) Tạm dịch: Những tòa văn phòng hiện đại có những sàn nâng, bên dưới chúng, dây điện máy vi tính và dâ
  • 81. Question 31. It’s possible that she didn’t hear what I said. A. She might not hear what I said. B. She might have not heard what I said. C. She may not hear what I said. D. She may not have heard what I said. Kiến thức: Modal Verb Câu đề bài cho: Có khả năng là cô ta đã không nghe thấy những gì tôi nói. Khi nói lại các tình huống đã xảy ra trong quá khứ với Modal Verb (động từ khuyết thiếu) mà cụ thể ở đây là việc có lẽ cô ta đã không nghe thấy điều tôi nói, chúng ta phải dùng cấu trúc: S + Modal Verb + HAVE + P2, duy nhất đáp án D làm được điều này nên D đúng. Câu A và C sai do hai câu này đang viết sử dụng công thức S + Modal Verb + V0 nói về một phỏng đoán ở Hiện tại hoặc Tương lai. Câu B sai do: Viết sai cấu trúc, phải là she might not have heard...
  • 82. Question 32. “Stop smoking or you’ll be ill,” the doctor told me. A. The doctor advised me to give up smoking to avoid illness. B. The doctor suggested smoking to treat illness, C. I was warned against smoking a lot of cigarettes. D. I was ordered not to smoke to recover from illness. Kiến thức: Câu chuyển Trực tiếp- Gián tiếp Câu đề bài cho: “Ngừng hút thuốc hoặc anh sẽ bị bệnh”, bác sĩ đã nói với tôi. A. Bác sĩ khuyên tôi nên bỏ hút thuốc để tránh bệnh tật. B. Bác sĩ đề nghị hút thuốc để điểu trị bệnh. C. Tôi đã được cảnh báo không hút nhiều thuốc lá. D. Tôi được lệnh không hút thuốc để khỏi bệnh.
  • 83. Question 33. I haven’t heard from Mike for several months. A. I didn’t hear from Mike several months ago. B. Mike didn’t hear from me several months ago. C. Mike heard from me several months ago. D. I last heard from Mike several months ago. Kiến thức: Viết lại cấu chuyển đổi Thì của động từ. Câu đề bài cho: Tôi đã không nghe tin tức gì từ Mike vài tháng nay rồi. A. Tôi đã không nghe gì từ Mike vài tháng trước. B. Mike đã không nghe gì từ tôi vài tháng trước. C. Mike đã nghe tin tức từ tôi vài tháng trước. D. Lần cuối cùng tôi nghe tin tức từ Mike là vài tháng trước đây.
  • 84. Question 34. We also use a great …………….of water daily in our homes, in factories, and in power stations. Most of this water is fresh water and it comes to US from reservoirs, rivers and lakes. A. number B. much C. many D. amount Kiến thức: Lượng từ Giải thích: “water” là danh từ không đếm được, cho nên ta dùng cụm “a great amount of”. Chọn D. Các phương án còn lại: A. number: số lượng B. much: nhiều (dùng với danh từ không đếm được) C. many: nhiều (dùng với danh từ đếm được số nhiều) Tạm dịch: Chúng ta cũng sử dụng một lượng nước lớn hàng ngày trong nhà, trong các nhà máy và trong các nhà máy điện. Hầu hết nước này là nước ngọt và nó đến với chúng ta từ các hồ chứa, sông hồ.
  • 85. Question 35. The Earth’s surface is ………. by large areas of water which we call oceans and seas A. covered B. reserved C. constructed D. included Kiến thức: Từ vựng Giải thích: A. cover (v): bao phủ B. reserve (v): dự trữ, để dành C. construct(v): xây dựng D. include (v): bao gồm Ta có cách dùng: be covered: được/bị bao phủ Tạm dịch: Bề mặt Trái đất được bao phủ bởi những vùng nước rộng lớn mà chúng ta gọi là đại dương và biển.
  • 86. Question 36. If you have tasted the water from the sea, you will know that, unlike fresh water, seawater tastes salty. This is due to the …….. of sodium chloride which comes from the land. A. attraction B. presence C. advantage D. realization Kiến thức: Từ vựng Giải thích: A. attraction (n): sự thu hút, sự lôi cuốn B. presence (n): sự hiện diện C. advantage (n): lợi thế, ưu điểm D. realization (n): sự nhận biết Tạm dịch: Nếu bạn đã nếm nước từ biển, bạn sẽ biết rằng, không giống như nước ngọt, nước biển có vị mặn. Điều này là do sự hiện diện của natri clorua đến từ đất.
  • 87. Question 37. The reason is …………. sunlight is made up of many colors. A. what B. whoC. which D. that Giải thích: Câu đã đầy đủ các thành phần: the reason (S) is (V).., và phía sau là một mệnh đề nên ta dùng “that” đứng trước mệnh đề đó. Đây là mệnh đề danh ngữ bắt đầu bằng THAT. Không có yếu tố thay thế cho danh từ đứng trước nên không dùng các phương án còn lại. Tạm dịch: Lý do là ánh nắng mặt trời được tạo thành từ nhiều màu sắc.
  • 88. Question 38. Some colors disappear quickly in the sea but blue light bounces back or is reflected, to the surface. This makes the sea look blue. …………, a stormy sky will make the sea look grey. A. Moreover B. Hence C. Although D. However Kiến thức: Liên từ Giải thích: A. Moreover: Hơn nữa B. Hence: do đó, cũng vì lý do đó C. Although: mặc dù D. However: tuy nhiên Ta chọn B theo ý nghĩa của bài. Tạm dịch: Một số màu biến mất nhanh chóng trên biển nhưng ánh sáng xanh bị dội ngược lại hoặc bị phản xạ lên bề mặt. Điều này làm cho biển trông xanh. Cũng bởi lý do này, một bầu trời giông bão sẽ làm cho biển trông xám xịt.
  • 89. Question 39. Whatis the main idea of the passage? A. Satoru Iwata is a man with amazing ideas. B. Gaming is not just for dedicated gamers anymore. C. Satoru Iwata is an important man for dedicated gamers D. The gaming industry is making educational games now. Câu hỏi về nội dung chính của bài:Ý chính của bài đọc này là gì? A. Satoru Iwata là một người đàn ông có những ý tưởng đáng ngạc nhiên. B. Việc chơi game không chỉ dành cho đối tượng chơi game chuyên nghiệp nữa. C. Satoru Iwata là một người đàn ông quan trọng cho những người chơi game chuyên nghiệp. D. Nền công nghiệp game đang tạo ra nhiều trò chơi mang tính giáo dục. Xuyên suốt bài đọc chúng ta thấy nói về những sáng kiến và ý tưởng mới của ông Satoru Iwata làm thay đổi ngành công nghiệp game. Đoạn 1: Khi ngành game đang chững lại và những sự cải tiến bị dập khuôn thì “Iwata didn’t just want to attract dedicatedgamers. He wanted to bring in new kinds of playersto video gaming.” - Iwata không muốn chỉ thu hút những người chơi giỏi. Ông ấy còn muốn thu hút nhiều kiểu loại người chơi mới vào chơi game. Đoạn 2: Iwata muốn trò chơi phù hợp với cuộc sống của mọi người hơn nữa, vì thế ông ta đã tạo ra game mà: “People interacted more directly with the game by using a touch screen instead of just a set of buttons” - Mọi người tương tác trực tiếp hơn với game bằng màn hình cảm ứng thay vì chỉ với mỗi những nút bấm. Đoạn 3: Những thay đổi đi cùng với những game mới do Iwatatạo ra: “Some of these new games, like Brain Age, improvedthinking abilities.Others, like Wii Fit, improvedfitness. This has brought in a new age of gaming. Everyone from grandparentsto their grandchildren
  • 90. Question 40. What first motivated Iwata to change the strategy of Nintendo? A. He wanted to make useful consoles. B. He wanted to lower sales. C. He wanted to attract more players. D. He wanted a new concept for gaming. Điều gì đầu tiên đã thúc đẩy Iwata thay đổi chiến thuật của Nintendo? A. Ông ta muốn tạo ra những máy chơi game hữu ích hơn. B. Ông ta muốn giảm doanh số bán hàng. C. Ông ta muốn thu hút nhiều người chơi game nữa. D. Ông ta muốn có một khái niệm chơi game mới. Thông tin ở đoạn 1: However, Iwata didn’t just want to attract dedicated gamers. He wanted to bring in new kinds of players to video gaming. Tuy nhiên, Iwata không chỉ muốn thu hút những người chơi game chuyên nghiệp. Ông ta còn muốn thu hút nhiêu người chơi game khác nữa.
  • 91. Question 41. The word “that” in paragraph 2 refer to . A. Bringing in new kinds of players to video gaming. B. Attracting dedicated gamers C. Making the same kinds of games D. Improving games’ power and complexity. Từ “that” trong đoạn 2 liên hệ với____. A. Thu hút nhiều người chơi game mới. B. Thu hút những người vốn đã thích chơi game. C. Sáng tạo ra những loại game tương tự. D. Cải thiện sức mạnh và độ phức tạp của game. Thông tin đoạn 1: He wanted to bring in new kinds of players to video gaming. How was he going to do that? Ông ta còn muốn thu hút nhiều người chơi game khác nữa. Ông ta sẽ định làm điều đó như thế nào? Chọn A.
  • 92. Question 42. Which is TRUE about changes Iwata make about the world of gaming? A. More games for children and girls. B. More dedicated games. C. Created a new style of gaming. D. Less expensive game systems. Điều gì là đúng về những thay đổi mà Iwata tạo ra về thế giới game? A. Có thêm nhiều trò chơi cho trẻ em và phụ nữ. B. Thu hút thêm nhiều người lúc nào cũng thích chơi game. C. Tạo ra một kiểu chơi game mới. D. Trò chơi ít đắt đỏ hơn. Thông tin: - Iwata wanted to make video games easier to pick up and more relevant to people’s lives. - The kinds of games produced changed as well. - Ông ta mong muốn khiến cho các trò chơi điện tử dễ hiểu và dễ chơi hơn, liên quan, phù hợp nhiều hơn đến cuộc sống của con người. - Loại hình game cũng thay đổi. Chọn C.
  • 93. Question 43. The word “groundbreaking” in the last paragraph is closest in meaning to A. innovative B. bad C. beautiful D. natural Từ “groundbreaking” ở đoạn cuối gần nghĩa với_____. A. innovative (adj): sáng tạo, cải tiến, đổi mới B. bad (adj): tệ hại C. beautiful (adj): đẹp D. natural (adj): tự nhiên Thông tin: With all that we have seen so far, we can only wonder what is next for Iwata. Surely it will be something groundbreaking. Với tất cả những thứ mà chúng ta nhìn thấy, chúng ta chỉ có thể hỏi là liệu Iwata sẽ làm gì tiếp theo. Chắc chắn đó sẽ là những thứ mang tính đột phá. Chọn A
  • 94. Question44. What topicdoes the passagemainly discuss? A. The way how to write the resume for job application. B. The mistakes people make when applying for a job. C. The commonway to make impression in a job interview. D. The necessary skills for job application. Bài văn chủ yếu thảo luận về chủ đề gì? A. Cách viết bản sơ yếu lý lịch để xin việc. B. Những lỗi mọi người thường gặp phải khi nộp đơn xin việc. C. Cách thông thường để tạo ấn tượng tốt trong một cuộc phỏng vấn xin việc. D. Những kĩ năng cần thiết để xin việc. Đoạn đầu tiên của bài đọc này đóng vai trò như phần mở bài giới thiệu nội dung chính của cả bài, và các đoạn tiếp theo đi làm rõ ý cho đoạn 1. Căn cứ thông tin: “There are many mistakes that people make when writing their resume (CV) or completing a job application. Here are some of the most common and most serious.” Có nhiều lỗi mà mọi người thường gặp khi viết bản sơ yếu lí lịch hay hoàn thành hồ sơ xin việc. Dưới đây là một vài lỗi thường gặp và nghiêm trọng nhất.
  • 95. Question 45. The word “executing” in paragraph 2 is closest in meaning to . A. enumerating B. determining C. completing D. implementing Từ “executing” trong đoạn 2 gần nghĩa nhất với từ______. A. liệt kê B. xác định, tìm ra C. hoàn thành D. thực hiện, thi hành Căn cứ thông tin: “They do not necessarily know the specific skills you used in executing them, nor do they know what results you achieved” - Họ không cần thiết phải biết những kĩ năng cụ thể bạn đã sử dụng để thực hiện các nhiệm vụ đó, họ cũng không cần biết kết quả bạn đã đạt được là gì. Từ đồng nghĩa: executing (thực hiện) = implementing. Chọn D.
  • 96. Question 46. The word “concrete” in paragraph 3 could be best replaced by . A. indeterminate B. specific C. substantial D. important Từ “concrete” trong đoạn 3 được thay thế tốt nhất bởi từ______. A. mơ hồ, không rõ B. cụ thể, rõ ràng C. chủ yếu, thiết yếu D. quan trọng Căn cứ thông tin: “The more concrete information you can include, the better.” - Thông tin bạn cung cấp càng cụ thể càng tốt Từ đồng nghĩa: concrete (cụ thể) = specific. Chọn B.
  • 97. Question 47. What does the word “it” in paragraph 3 refer to ? A. organization money B. information C. productivity D. percentage Từ “it” trong đoạn 3 đề cập đến từ nào? A. tiền của tổ chức B. thông tin C. năng suất D. tỉ lệ phần trăm Căn cứ thông tin đoạn 3: “If any innovations you introduced saved the organizationmoney, how much did they save? If you found a way of increasing productivity, by what percentage did you increase it?” Nếu bạn đã đưa ra được ỷ tưởng đổi mới nào giúp tiết kiệm được tiền cho tổ chức, thì họ đã tiết kiệm được bao nhiêu tiền? Nếu bạn đề xuất được một cách tăng năng suất, vậy bạn đã làm tăng nó lên bao nhiêu phần trăm? Vậy “it” ở đây thay thế cho “productivity” → Chọn C.
  • 98. Question48. Accordingto the passage, what informationshould candidatesincludein their resume? A. specific skills for previous jobs B. the past achievements C. previous positions D. future objective Theo đoạn văn, thông tin nào những người xin việc nên bao hàm trong bản sơ yếu lí lịch của họ? A. các kĩ năng cụ thể cho các công việc trước đây B. các thành tựu đạt được trong quá khứ C. các chức vụ, vị trí công việc trước đây D. mục tiêu trong tương lai Từ khóa: information/include in their resume Cản cứ các thông tin trong đoạn văn: “The biggest problem is perhaps listing the duties for which you were responsible in a past position:all this tells your potential employers is what you were supposed to do. They do not necessarily know the specific skills you used in executing them, nor do they know what results you achieved” - Vấn đề lớn nhất có lẽ là liệt kê các công việc mà bạn đã làm trong chức vụ trước đây: tất cả những điều này nói cho nhà tuyển dụng tiềm năng của bạn biết những gì bạn có thể làm được. Họ không cần thiết phải biết những kĩ năng cụ thể bạn đã sử dụng để thực hiện các nhiệm vụ đó.→ Loại A ...họ cũng không cần biết kết quả bạn đã đạt được là gì. -→ Loại B “Writing what you are trying to achieve in life - your objective - is a waste of space.” - Viết về những điều bạn đang cố gắng đạt được trong cuộc sống - mục tiêu của bạn - là một sự lãng phí giấy. → Loại D Chọn C: chúng ta liệt kê các chức vụ trước đây đã làm nhưng không nên nhắc các kĩ
  • 99. Question 49 According to the passage, which of the following is NOT true? A. The abilityto negotiate effectively is as significant as technicalskills. B. Candidatesmust study the job they are applyingcarefullybefore writing the CV C. Applicantsshould not apply for a distinct job from what they are doing. D. The information interviewees present should be related to the job they are applying. Theo đoạn văn, câu nào sau đây là không đúng? A. Khả năng đàm phán hiệu quả cũng quan trọng như các kĩ năng thực hành. B. Người xin việc cần phải nghiên cứu công việc họ đang nộp đơn xin một cách kĩ càng trước khi viết bản sơ yếu lí lịch. C. Người xin việc không nên nộp đơn xin một công việc khác với công việc mà họ đang làm. D. Những thông tin mà người đi phỏng vấn trình bày nên có liên quan đến công việc mà họ đang xin. Từ khóa: NOT TRUE Cản cứ vào các thông tin trong đoạn văn: “However, yourabilityto negotiate effectively, for example, can be just as important as your technicalskills.” Tuy nhiên, khả năng đàm phán hiệu quả của hạn cũng quan trọng như các kĩ năng thực hành. Loại A. “All information you give should be relevant, so carefully consider the job for which you are applying.If you are applying for a job that is somewhat different than your current job, it is up to you to draw a connectionfor the resume reviewer, so that they will understandhow your skillswill fit in their organization.” - Tất cả các thông tin bạn cung cấp nên có mối liên
  • 100. Question50. It can be inferred from the last paragraphthat A. you should write accurately about your ability for the vacant position. B. you should be modest about what you can do. C. a resume reader is good enough to understand what you imply about your ability in the CV. D. you are allowed to exaggerate the truth of your competence if possible. . Có thể suy ra từ đoạn văn cuối rằng______. A. bạn nên viết một cách chính xác về khả năng của bạn cho vị trí còn trống. B. bạn nên khiêm tốn về những gì bạn có thể làm. C. người đọc bản sơ yếu lý lịch đủ giỏi để hiểu những gì bạn hàm ý về khả năng của bạn trong bản sơ yếu lịch. D. bạn được phép phóng đại sự thật về khả năng của bạn nếu có thể. Căn cứ vào thông tin đoạn cuối: “If you are modest about the skills you can offer, or the results you have achieved, a resume reader may take what you write literally, and be left with a low opinion of your ability: you need to say exactly how good you are. On the other hand, of course, never stretch the truth or lie.” Nếu bạn khiêm tốn vê những kĩ năng bạn có thể làm hay kết quả bạn đã đạt được, người đọc bản sơ yếu lí lịch sẽ hiểu theo đúng nghĩa đen bạn viết, và sẽ có ấn tượng không tốt về khả năng của bạn: bạn cần phải nói chính xác bạn giỏi đến mức nào. Mặc khác, tất nhiên, không bao giờ được phóng đại sự thật hay nói dối.
  • 101. Question 1: His______of the generator is very famous. A. invent B. inventive C. invention D. inventor Căn cứ bằng tính từ sở hữu "his". Sau tính từ sở hữu + N Trong đó: invent (v): phát minh, sáng chế inventive (a): có tài sáng chế, đầy sáng tạo invention (n): sự phát minh, sự sáng chế inventor (n) : người phát minh, người sáng tạo Dịch nghĩa: Phát minh về máy phát điện của ông ấy rất nổi tiếng.
  • 102. Question 2: They rarely let her stay out late, _______? A. do they B. don’t they C. does she D. doesn’t she - câu hỏi đuôi Câu hỏi đuôi luôn hỏi cho động từ và chủ ngữ ở mệnh đề chính. Trong trường hợp này, chủ ngữ của mệnh đề chính là “they” nên hai đáp án C và D (hỏi cho She) bị loại. Câu khẳng định có câu hỏi đuôi dạng phủ định còn câu phủ định có câu hỏi đuôi dạng khẳng định. Câu đề bài cho có “rarely” là trạng từ phủ định nên nó là câu phủ định. Vì vậy, chọn đáp án A - do they (có phần hỏi đuôi khẳng định). Tạm dịch: Họ không cho cô ấy ra ngoài quá muộn đâu, có đúng không?
  • 103. Question 3: The commission estimates that at least seven companies took___________ of the program. A. advantage B. useC. benefit D. dominance câu hỏi thành ngữ Thành ngữ “take advantage of sb/sth”: tận dụng/ lợi dụng ai hoặc cái gì đó. Xét 4 đáp án, chỉ chọn được A - advantage. A. advantage (n): lợi thế B. use (n)/ (v): công dụng (n) - thường đi trong “Make use of sth - tận dụng cái gì đó/ sử dụng (v) C. benefit (n): lợi ích D. dominance (n): sự thống trị Tạm dịch: Ủy ban này ước tính rằng có ít nhất 7 công ty đã lợi dụng chương trình này
  • 104. Question 4. Both husband and wife should be responsible ________ doing the household chores. A. with B. to C. for D. of Kiến thức: Giới từ Giải thích: be responsible for st/doing st: có, chịu trách nhiệm về cái gì/làm gì Tạm dịch: Cả hai vợ chồng đều nên có trách nhiệm làm việc nhà.
  • 105. Question 5: Mary was clearly nervous; she was sitting right on the______ of his chair. A. outside B. edge C. tip D. border Outside: bên ngoài Edge: mép Tip: đầu Border: biên giới Câu này dịch như sau: Mary rõ ràng đang lo lắng; cô ây ấy đang ngồi ngay ở mép ghế.
  • 106. Question 6: I will stand here and wait for you you come back. A. because B. though C. so D. until A. Because: bởi vì C. So: vì thế B. Though: mặc dù D. Until: mãi đến khi Dịch nghĩa: Anh sẽ vẫn đứng đây chờ em cho đến khi em quay lại.
  • 107. Question 7. She bought a _________ lunchbox that she could carry lunch to work. A. new red plastic B. red plastic new C. new plastic red D. plastic new red Kiến thức: Trật tự tính từ Giải thích: Khi có nhiều tính từ cùng đứng trước 1 danh từ, sắp xếp chúng theo thứ tự: OSASCOMP+ N. Trong đó: O – opinion: quan điểm S – size: kích thước A – age: độ tuổi S – shape: hình dạng C – colour: màu sắc O – origin: nguồn gốc M – material: chất liệu P – purpose: mục đích N – noun: danh từ Nếu có số thứ tự => đứng trước tính từ & danh từ Tạm dịch: Cô ấy đã muamột hộp cơm bằng nhựa màu đỏ mới để có thể mang đi ăn trưa để đi làm.
  • 108. Question 8: The government hopes to______its plans for introducing cable TV. A. turn out B. carry out C. carry on D. keep on + turn out: hoá ra + carry on = keep on = go on = continue: tiếp tục + carry out: tiến hành Dịch nghĩa: Chính phủ hi vọng thực hiện được kế hoạch áp dụng cáp quang ti vi.
  • 109. Question 9: When she came home from school yesterday, her mother _______in the kitchen. A. cooked B. was cooking C. is cooking D. cooks Kiến thức: Thì trong tiếng Anh. Giải thích: Ta dùng thì quá khứ đơn và thì quá khứ tiếp diễn để diễn tả một hành động đang xảy ra trong quá khứ thì có một hành động khác xen vào. Tạm dịch: Hôm qua khi cô ấy đi học về, mẹ cô đang nấu ăn trong bếp.
  • 110. Question 10: ______, he went straight home. A. While he would finish his work B. When he has finished his work C. After he had finished his work D. Before he has been finishing his work Kiến thức: Thì quá khứ hoàn thành. Giải thích: Cấu trúc: After S + had +Ved/ Vp2, S + Ved/ V2 Tạm dịch: Sau khi anh ấy hoàn thành xong công việc, anh ấy đi thẳng về nhà.
  • 111. Question 11: The jury______her compliments on her excellent knowledge of the subject A. paid B. gave C. made D. said Câu này dịch như sau: Bồi thẩm đoàn khen ngợi cô ấy kiến thức tuyệt vời về chủ đề này. Pay compliment /ˈkɒmplɪmənt/ (n) on sth = compliment/ˈkɒmplɪment/ (v) on sth: khen ngợi ai về việc gì
  • 112. Question 12: I took the children to the _______park last weekend. They really enjoyed going on all the rides. A. wildlife B. amusement C. national D. entertainment Kiến thức: từ vựng Wildlife (n): động vật hoang dã Amusement(n): sự giải trí National (a): thuộc về quốc gia Entertainment(n): sự giải trí Cụm danh từ: amusemnet park [ khu vui chơi giải trí] Câu này dịch như sau: Tôi dẫn lũ trẻ đến khu vui chơi cuối tuần trước. Chúng thật sự thích cả chuyến đi.
  • 113. Question 13. This villa ———————- in 1975 by my grandfather. A. built B. was builtC. was build D. has built Kiến thức: Câu bị động: Giải chi tiết: Câu bị động thì Quá khứ đơn. Hành động được nhấn mạnh là ngôi nhà được xây dựng, trạng ngữ chỉ thời gian là in 1970. S + động từ tobe + V-ed/V3 Tạm dịch : Căn biệt thự được xây vào năm 1975 bởi ông tôi.
  • 114. Question 14. ________ this movie last week, I don’t want to see it again. A. Having seen B. Being seen C. Having been seen D. Seeing Kiến thức: Mệnh đề phân tử / Rút gọn mệnh đề đồng ngữ Giải thích: Khi 2 mệnh đề có cùng chủ ngữ (I) thì có thể rút gọn 1 trong 2 mệnh đề về dạng: - V-ing / Having P2: nếu mệnh đề được rút gọn mang nghĩa chủ động - P2 (quá khứ phân từ): nếu mệnh đề được rút gọn mang nghĩa bị động Chủ ngữ “I” có thể làm chủ (tự thực hiện hành động”see” => nghĩa chủ động. hành động đã xảy ra trong quá khứ. Having+V3/ed Tạm dịch: Đã xem phim này tuần trước, tôi không muốn xem lại.
  • 115. Question 15: The richer you are, _______. A. you may become more worried B. you more worried may become C. the more worried you may become D. the more worry you may become become Xét các đáp án: A. you may become more worried => Sai cấu trúc so sánh kép B. you more worried may become => Sai cấu trúc so sánh kép C. the more worried you may become → Cấu trúc so sánh kép khi nói về 2 người hoặc sự vật: The + so sánh hơn + S + V, the + so sánh hơn + S + V D. the more worry you may become → sau more là adj/adv, không phải động từ
  • 116. Question 16: “How fashionable a pair of trainers you have!” - “______________” A. Do you want to know where I bought them? B. Thanks for your compliment. C. I know it’s fashionable. D. Yes, of course. Tình huống: “Bạn có đôi giày thể thao thật thời trang!” A. Bạn có muốn biết tôi mua chúng ở đâu không? B. Cảm ơn vì lời khen. C. Tôi biết nó hợp thời trang. D. Đúng rồi, tất nhiên là vậy. => Khi được khen, chúng ta nói cảm ơn
  • 117. Question 17: “What can I do for you?” - “_______.” A. No need to help. B. Thank you. C. Thanks, I’m just looking. D. Sorry for not buying anything “Tôi có thể giúp gì cho bạn?” - “__________” A. Không cần giúp. B. Cảm ơn. C. Cảm ơn, tôi chỉ đang xem thôi. D. Xin lỗi vì đã không mua thứ gì. => Khi được hỏi có cần giúp gì không => chúng ta nói cảm ơn
  • 118. Question 18: A. admit B. suggest C. remind D. manage Kiến thức: Trọng âm từ có 2 âm tiết Giải thích: A. admit /ədˈmɪt/ B. suggest /səˈdʒest/ C. remind /rɪˈmaɪnd/ D. manage /ˈmænɪdʒ/
  • 119. Question 19: A. approval B. applicant C. energy D. influence Kiến thức: Trọng âm từ có 3 âm tiết Giải thích: A. approval /əˈpruːvl/ B. applicant /ˈæplɪkənt/ C. energy /ˈenədʒi/ D. influence /ˈɪnfluəns/
  • 120. Question 20: A. watched B. cleaned C. stopped D. picked Kiến thức: Phát âm “ed” Giải thích: Cách phát âm đuôi “ed”: + Đuôi “ed” được phát âm là /id/ khi động từ có phát âm kết thúc là /t/ hay /d/ + Đuôi “ed” được phát âm là /t/ khi động từ có phát âm kết thúc là /s/,/f/,/p/,/ʃ/,/tʃ/,/k/ + Đuôi “ed” được phát âm là /d/ với các trường hợp còn lại A. watched /wɒtʃt/ B. cleaned /kliːnd/ C. stopped /stɒpt/ D. picked /pɪkt/
  • 121. Question 21: A. hole B. home C. come D. hold Kiến thức: Phát âm “o” Giải thích: A. hole /həʊl/ B. home /həʊm/ C. come /kʌm/ D. hold /həʊld/
  • 122. Question 22: I will not stand for your bad attitude any longer. A. like B. sit C. tolerate D. care Kiến thức: Từ đồng nghĩa Giải thích: stand for: chịu đựng A. like (v): thích B. sit (v): ngồi C. tolerate (v): chịu đựng D. care (v): quan tâm => stand for = tolerate Tạm dịch: Tôi sẽ không chịu đựng thái độ xấu của bạn nữa.
  • 123. Question 23. The repeated commercials on TV distract many viewers from watching their favorite films .A. advertisements B. contests C. business D. economics Tạm dịch: Những quảng cáo lặp đi lặp lại trên ti vi sẽ làm cho người ta bị xao nhãng khỏi việc xem các bộ phim yêu thích của họ. commercials = advertisements: những mẩu quảng cáo
  • 124. Question 24: In some countries, so few students are accepted by the universities that admission is almost a guarantee of a good job upon graduation. A. a promise B. an uncertainty C. an assurance D. a pledge Từ trái nghĩa - kiến thức về thành ngữ Tạm dịch: Ở một số quốc gia, rất ít sinh viên được các trường đại học chấp nhận mà việc nhập học gần như là một sự đảm bảo cho một công việc tốt khi tốt nghiệp. => guarantee /ˌɡærənˈtiː/ (n): lời cam kết, đảm bảo Xét các đáp án: A. promise (n): lời hứa B. uncertainty /əˈʃʊərəns/ (n): tính không chắc chắn C. assurance /ʌnˈsɜːrtnti/ (n): lời cam kết, hứa hẹn D. pledge /pledʒ/ (n): lời cam kết, lời nguyện => Do đó: a guarantee >< an uncertainty
  • 125. Question 25: The burglar crept into the house without making any noise. That's why no one heard anything. A. inaudibly B. boisterously C. shrilly D. hurly-burly Từ trái nghĩa - kiến thức về thành ngữ Tạm dịch: Tên trộm lẻn vào nhà mà không hề tạo ra chút tiếng ồn nào. Đó là lý do không ai nghe thấy gì. => without making any noise: không hề tạo ra chút tiếng ồn nào Xét các đáp án: A. inaudibly /ɪnˈɔːdəbli/ (adv): một cách vô thanh B. boisterously /ˈbɔɪstərəsli/ (adv): một cách ầm ĩ, náo nhiệt (tràn đầy năng lượng) C. shrilly /ˈʃrɪlli/ (adv): một cách the thé, inh tai (gây khó chịu) D. hurly-burly /ˈhɜːrli bɜːrli/ (adv): một hoạt động, tình huống ồn ào, náo nhiệt => Do đó: without making any noise >< boisterously
  • 126. Question 26: Seth informed us of his retirement from the company. He did it when arriving at the meeting. A. Only after his retiring from the company did Seth tell us about his arrival at the meeting. B. Not until Seth said to us that he would leave the company did he turn up at the meeting. C. Hardly had Seth notified us of his retiring from the company when he arrived at the meeting D. No sooner had Seth arrived at the meeting than we were told about his leaving the company Kiến thức: Đảo ngữ Giải thích: No sooner + had + S + PP than + S + Ved/ V2: Ngay khi... thì...
  • 127. Question 27: They were late for the meeting. The heavy was heavy. A. If it snowed heavily, they would be late for the meeting. B. Had it not snowed heavily, they would have been late for the meeting. C. But for the heavy snow, they wouldn't have been late for the meeting. D. If it didn't snow heavily, they wouldn't be late for the meeting. Tạm dịch: Họ đến muộn buổi gặp mặt. Tuyết rơi nhiều. They were late for the meeting. The snow was heavy. (nguyên nhân và kết quả) ( Quá khứ đơn) Tình huống ở quá khứ nên viết lại bằng câu điều kiện loại 3. A. Sai vì là câu điều kiện loại 2 B. Sai vì là câu điều kiện loại 3 nhưng không phù hợp về nghĩa với câu đã cho (Nếu không có tuyết rơi nhiều, họ đã đến muộn buổi gặp mặt) C. Đúng vì phù hợp là câu điều kiện loại 3 (rút gọn) và phù hợp về nghĩa. D. Sai vì là câu điều kiện loại 2
  • 128. Question 28: Mrs. Hoa and her friends from Vietnam plan to attend the festival now * Chia thì theo đúng trạng ngữ chỉ thời gian: now Mrs. Hoa and her friends from Vietnam (plan) to attend the festival now. Ta phải chia động từ plan thành thì hiện tại tiếp diễn
  • 129. Question 29: Some manufacturers are not only raising their prices but also decreasing the production of its products. Kiểm tra kiến thức: Sự hoà hợp giữa chủ ngữ và tính từ sở hữu cách Some manufacturers số nhiều → their Dịch: Một số nhà sản xuất không những tăng giá thành mà còn giảm năng suất của các sản phẩm.
  • 130. Question 30: The whole matter is farther complicated by the fact that Amanda and Jo refuse to speak to each other. Cả “farther” và “further” đều là dạng so sánh hơn của từ “far”. Tuy nhiên, chúng có sự khác nhau như sau: + Farther: xa hơn (về khoảng cách địa lí) + Further: xa hơn, sâu hơn (về mức độ, tính chất) Tạm dịch: Toàn bộ vấn đề phức tạp hơn bởi thực tế là Amanda và Jo từ chối nói chuyện với nhau. => Đáp án là B (farther -further) Cấu trúc khác: Refuse to do st: từ chối làm gì
  • 131. Question 31: A supermarket is more convenient than a shopping centre. A. A shopping centre is not as convenient as a supermarket. B. A shopping centre is more convenient than a supermarket. C. A supermarket is not as convenient as a shopping centre. D. A supermarket is as inconvenient as a shopping centre. Kiến thức: diễn đạt câu với Câu so sánh Giải thích: Một siêu thị thuận tiện hơn một trung tâm mua sắm. A. Một trung tâm mua sắm không thuận tiện như siêu thị. B. Một trung tâm mua sắm thuận tiện hơn siêu thị. C. Một siêu thị không thuận tiện như một trung tâm mua sắm. D. Một siêu thị bất tiện như một trung tâm mua sắm.
  • 132. Question 32: "It was your fault. You broke my windows, "said the woman to him. A. The woman insisted him on breaking her windows. B. The woman advised him to break her windows. C. The woman told him to break her windows. D. The woman blamed him for having broken her windows. Tạm dịch: “ Đó là lỗi của cháu. Cháu đã làm vỡ cửa kính nhà bác” , Người phụ nữ nói với cậu bé. A. Người phụ nữ khăng khăng bắt cậu bé làm vỡ của kính nhà bà ta. B. Người phụ nữ khuyên cậu bé làm vỡ của kính nhà bà ta. C. Người phụ nữ bảo cậu bé làm vỡ của kính nhà bà ta. D. Người phụ nữ đổ lỗi cho cậu bé vì đã làm vỡ của kính nhà bà ta.
  • 133. Question 33. It was a mistake for Tony to buy that house. A. Tony couldn’t have bought that house. B. Tony can’t have bought that house. C. Tony needn’t have bought that house. D. Tony shouldn’t have bought that house. Kiến thức: Động từ khuyết thiếu Giải thích: couldn’t have P2: không thể làm gì trong quá khứ can’t have P2: không thể nào needn’t have P2: đáng lẽ ra không cần shouldn’t have P2: đã không nên làm gì Tạm dịch: Đó là lỗi của Tony khi mua căn nhà đó. D. Tony đã nên không mua căn nhà đó.
  • 134. Question 34: “His sister learned the household skills that would prepare her to become ..........wife and mother.” A. many B. the C. an D. a Giái thích: vì his sister là số ít nên loại bỏ A; Cụm từ “become” trở thành….không xác định, số ít nên dùng mạo từ “a” (Chị gái cậu ấy đã học các kĩ năng nội trợ sẽ chuẩn bị cho tương lai cô ấy trở thành người vợ và người mẹ.)
  • 135. Question 35: “Nowadays young people grow up in a much freer society ........ they enjoy almost unlimited career opportunities” A. where B. when C. why D. whom where: trạng từ quan hệ, thay thế cho từ/ cụm từ nơi chốn when: trạng từ quan hệ, thay thế cho từ/ cụm từ chỉ thởi gian why: trạng từ quan hệ, thay thế cho từ/ cụm từ chỉ lý do whom: đại từ quan hệ, thay thế cho tân ngữ chỉ người Trong câu này từ cần điền vào là đại từ quan hệ thay thế cho cụm từ nơi chốn “in a society”. (Ngày nay, những người trẻ lớn lên trong một xã hội tự do hơn nhiều, nơi mà họ có được hầu như vô số cơ hội nghề nghiệp.)
  • 136. Question 36: “In recent years, there ............an enormous increase in the kinds of vocations from which it is possible to choose” A. had been B. has been C. will be D. was In recent years: trong những năm gần đây => động từ chia ở thì hiện tại hoàn thành (Trong những năm gần đây, đã có sự tăng mạnh về các loại nghề nghiệp để cho mọi người lựa chọn.)
  • 137. Question 37: “In addition, many of the barriers to career opportunity that existed only a few decades ago, such as ............. based on sex or religion or ethnic origins...” religion or ethnic origins...” A. judgement B. perception C. devotion D. discrimination judgment (n): sự đánh giá, óc phán đoán perception (n): sự tiếp nhận, sự nhận thức devotion (n): sự tận tụy, sự hiến dâng discrimination (n): sự phân biệt đối xử (Thêm vào đó, nhiều rào cản đối với cơ hội nghề nghiệp đã tồn tại một vài thập kỉ trước đây như sự phân biệt đối xử dựa trên giới tính hoặc tôn giáo hoặc nguồn gốc dân tộc...)
  • 138. Question 38: “In addition, many of the barriers to career opportunity that existed only a few decades ago, such as discrimination based on sex or religion or ethnic origins, are ............. disappearing.” A. rapidly B. incessantly C. categorically D. vigilantly rapidly (adv): nhanh chóng incessantly (adv): không ngừng, không dứt categorically (adv): rõ ràng, minh bạch vigilantly (adv): thận trọng, cảnh giác (Thêm vào đó, nhiều rào cản đối với cơ hội nghề nghiệp đã tồn tại chỉ một vài thập kỉ trước đây như sự phân biệt đối xử dựa trên giới tính hoặc tôn giảo hoặc nguồn gốc dân tộc đang nhanh chóng biến mất.)
  • 139. Question39. What is the main focus of this passage? A. Jobs on Wall Street B. Types of graduate degrees C. Changes in enrollment for MBA schools D. How schools are changing to reflect the economy Trọng tâm chính của bài đọc này là gì? Các công việc trên phố Wall Các loại bằng tốt nghiệp Những thay đổi trong việc tuyển sinh của các trường MBA Cách thức các trường đang thay đổi để phản ánh nền kinh tế Dẫn chứng: Đoạn 1: After twenty years of growing student enrollments and economic prosperity, business schools in the United States have started to face harder times... (Sau 20 năm gia tăng số sinh viên đăng kí học và sự phát triển thịnh vượng của nền kinh tế thì các trường kinh doanh ở Mỹ đã bắt đầu phải đối mặt với những khó khăn...) Đoạn 2: There are two factors causing this decrease in students seeking an MBA degree. (Có 2 yếu tố dẫn đến tình trạng giảm số lượng sinh viên theo đuổi bằng MBA.)
  • 140. Question 40. The word “prosperity” in the first paragraph could be best replaced by which of the following? A. success B. surplus C. nurturing D. education success (n): sự thành công surplus (n): số dư nurture (n): sự nuôi dưỡng education (n): giáo dục “After twenty years of growing student enrollments and economic prosperity, business schools in the United States have started to face harder times... (Sau 20 năm gia tăng số sinh viên đăng kí học và sự phát triển thịnh vượng của nền kinh tế thì các trường kinh doanh ở Mỹ đã bắt đầu phải đối mặt với những khó khăn...)” Do đó: prosperity: ~ success: sự thịnh vượng, sự thành công
  • 141. Question 41. Which of the following business schools has shown an increase in enrollment? A. Princeton B. Harvard C. Stanford D. Yale Trường kinh doanh nào sau đây cho thấy sự gia tăng trong việc tuyển sinh? Princeton Harvard Stanford Yale Dẫn chứng: Only Harvard’s MBA School has shown a substantial increase in enrollment in recent years. Both Princeton and Stanford have seen decreases in their enrollments. (Chỉ có trường kinh doanh MBA ở Harvard đã cho thấy việc tuyển sinh tăng đáng kể trong những năm gần đây. Cả Princeton và Standford đều cho thấy việc tuyển sinh giảm xuống.)
  • 142. Question 42. Which of the following descriptions most likely applies to Wall Street? A. a center for international affairs B. a major financial center C. a shopping district D. a neighborhood in New York Miêu tả nào sau đây có thể thích hợp nhất với Phố Wall? Một trung tâm ngoại giao Một trung tâm tài chính lớn Một khu vực mua sắm Một vùng lân cận ở New York Dẫn chứng: The first one is that many graduates of four-year colleges are finding that an MBA degree does not guarantee a plush job on Wall Street, or in other financial districts of major American cities... (Yếu tố đầu tiên là nhiều sinh viên tốt nghiệp đại học 4 năm nhận thấy rằng bằng MBA không đảm bảo công việc tốt ở phố Wall hoặc các khu vực tài chính khác ở các thành phố lớn của nước Mỹ.)
  • 143. Question 43: As used in the second paragraph, the word “struggling” is closest in meaning to_____. A. evolving B. plunging C. starting D. striving evolving: tiến hóa plunging: lao xuống, đẩy vào starting: bắt đầu striving: cố gắng, đấu tranh Business needs are changing, and MBA schools are struggling to meet the new demands. (Nhu cầu kinh doanh đang thay đổi và các trường MBA đang cố gắng hết sức để đáp ứng các yêu cầu mới.)” Do đó: struggling ~ striving: cố gắng phấn đẩu